Sie sind auf Seite 1von 262

16

6. While demand is ----, land is still selling because


1. - 16.sorularda, cmlede bo braklan yerlere
the new homes market remains strong..
uygun den szck ya da ifadeyi bulunuz.

A) getting on

1. Most scientists agree that the human history of B) slowing down


North America began when the early ---- of
modern Native Americans made their way across C) taking after
a land bridge that once connected north-eastern
Asia to North America.. D) showing off

E) keeping away
A) inhabitants
7. At the end of the First World War, the leaders of
B) ancestors
victorious countries gathered at Versailles, and
C) colleagues there, they ---- to decide what penalties
Germany, Austria and other allies ----..
D) counterparts
A) tried / would have to pay
E) descendants
B) had tried / must have paid
2. Even during its early phases, the French
Revolution aroused ---- conflict throughout C) were trying / were paying
Europe..
D) used to try / might have paid

A) ignorant E) could try / should have paid

B) accessible
8. Migraine-patients who go untreated for too long
C) significant ---- structural changes in their brains, so they ----
proper therapy..
D) durable
A) are incurring / insisted on
E) fertile
B) incurred / would insist on
3. A lack of skills or of higher education are issues
that ---- many South Africans from making a C) incur / used to insist on
decent living..
D) must incur / would have insisted on

A) prevent E) may incur / should insist on

B) promote
9. The chestnut tree bears no fruit ---- about 15
C) complain years after plantation..

D) offer
A) since
E) help
B) still

4. Today, Pompeii has become a popular tourist C) yet


destination; with ---- 2.5 million visitors a year..
D) when

A) conversely E) until

B) legally
10. ---- the Kyoto Protocol is signed by all nations
C) randomly and strictly followed, the harm done to the
environment will be irreversible..
D) approximately
A) In order that
E) mainly
B) If only
5. A partial recovery a few days later raised hopes,
but ---- to be deceptive.. C) In the event of

D) In case
A) turned out
E) Unless
B) fell out

C) pulled through

D) made up

E) took over
11. ---- half of the budget of the US National Institute 16. Freud, ---- was continually worried about his poor
on Ageing is spent on research into Alzheimers health, feared that he would die before his
disease.. mother..

A) As much as A) that

B) More of B) whom

C) Rather than C) whose

D) Other D) who

E) The most E) how

12. The essays taken together tell the story of how


the US became an economic power ---- a
scale unprecedented ---- history..

A) for / with

B) at / by

C) with / from

D) through / at

E) on / in

13. ---- its vast forests, Myanmar was thought


to possess the largest number of tigers ----
India..

A) For / with

B) In / for

C) At / to

D) With / after

E) Through / by

14. An adolescent's discovery that he or she thinks


and feels ---- way ---- someone else becomes an
important basis of friendship and helps in the
development of a sense of identity..

A) as / as

B) not only / also

C) the same / as

D) more / than

E) both / and

15. The main functions of the kidneys are ---- the


volume, composition, and pH of body fluids and -
--- metabolic wastes from the blood and excrete
them to the outside..

A) regulating / to be removed

B) to be regulated / having removed

C) having been regulated / removed

D) to regulate / to remove

E) regulated / remove
21. V.
17. - 21.sorularda, aadaki parada
numaralanm yerlere uygun den szck ya
da ifadeyi bulunuz. A) which

B) both

Scientists have long suspected thatthe vibrant colours and C) certain


flashy features of male birds show their prospectivemates
that they are fit and healthy. Now research has (I) ---- the D) the
theory, byconnecting the brightness of the beak colour in
zebra finches (II) ---- thehealth of their immune systems. E) a
The secret (III) ---- chemicals calledcarotenoids. (IV)---- are
the red and yellow pigments found in the beaks of (V)----
species, including male zebra finches and black birds.

17. I.

A) determined

B) disproved

C) reduced

D) exhausted

E) confirmed

18. II.

A) into

B) with

C) for

D) over

E) through

19. III.

A) refers to

B) belongs to

C) leaves out

D) lies in

E) links with

20. IV.

A) Some

B) Such

C) Any

D) What

E) These
26. V.
22. - 26.sorularda, aadaki parada
numaralanm yerlere uygun den szck ya
da ifadeyi bulunuz. A) how

B) as

Older people suffer more brokenbones than younger C) such


people because the mass and strength of bones decrease
withage. There is no single reason why this occurs, or why D) so
some individuals andpopulations are more (I) ---- than
others. Like other complex traits, age-relatedchanges in E) most
bones result (II) ---- interactions between environmental
andgenetic factors. Scientists have linked changes in bone
strength to (III) ----in physical activity, the levels of dietary 27. - 36.sorularda, verilen cmleyi uygun
calcium and vitamin D, and alcoholand tobacco use. ekilde tamamlayan ifadeyi bulunuz.
However, among (IV) ----, physical activity is the
variable(V) ---- likely to account for the geographic
heterogeneity in the incidence offractures.
27. ---- as soon as they get him into the ambulance..

A) We were still waiting for the news of him


22. I.
B) It was your job to find out
A) considerate
C) His condition deteriorated rapidly
B) equal
D) Be sure to give me a call
C) sensible
E) The situation has proved urgent
D) negligible
28. ---- so that it can get close to hostile
E) vulnerable forces without being detected..

23. II. A) The precision navigation systems are what


impressed most of us
A) to
B) Ideally the periscope would have been eliminated
B) of
C) The new-style submarine has been specifically
designed
C) from
D) This is just one of a number of smaller, smarter,
D) over
stealthier submarines
E) at
E) Previously submarines were not expected to work
in shallow, coastal waters
24. III.
29. Just before a flash of lightning lights the sky, -
A) circulations ---..

B) settlements
A) the phenomenon means physicists may have to
C) variations rethink how lightning is made

D) tendencies B) past studies have seen occasional hints of X-ray


bursts
E) similarities
C) a mysterious property of lightning has been
confirmed
25. IV.
D) an electromagnetic noise accompanies lightning

A) these E) a huge blast of X-rays or other energy particles is


released
B) that

C) which 30. ---- whether or not life is an accident unique


to our planet..
D) whom

E) this A) We cannot be sure

B) It is hardly surprising

C) The evidence clearly suggests

D) The chances are

E) The risks cannot be calculated


31. Although NASA's budget has risen by 7% 36. Small states in Africa like Senegal would
ever the past two years, ----.. probably integrate with the global production
chains of multinationals ----..

A) its responsibilities have grown much faster


A) although Mali and Cameroon could be industrialized
B) the space station programmes are not targets for by processing foodstuffs
cutting costs
B) while countries with large populations like Nigeria
C) the programme of unmanned missions could have might develop their own national industries
been discontinued
C) if ever South African firms expand aggressively
D) Earth-observing satellites are of less importance across the region

E) in fact, the Voyager probes may be cancelled D) whereas such obstacles explain why sub- Saharan
countries have only a marginal share in
manufactured goods
32. Animals are particularly vulnerable ----..
E) in that most African industries are classified as
small-to-medium sized enterprises
A) when they are in their infancy

B) whether they could fend for themselves


37. - 42.sorularda, verilen ngilizce cmleye
C) as soon as they had left their mothers anlamca en yakn Trke cmleyi, Trke
cmleye anlamca en yakn ngilizce cmleyi
D) so that their offspring will survive bulunuz.

E) unless every effort has been made by the parents

33. You may increase your risk of contact dermatitis, 37. ABD Ticaret Bakanl, hassas teknolojik bilgilere
especially on sensitive skin, ----.. ulam snrlandrmaya ilikin politikalarn
gzden geirilmesi gerekip gerekmediini
incelemek zere bir grup uzman
A) on condition that you have satisfied hydration and grevlendirmitir..
radiance concerns

B) just because your skin feels smoother A) A group of experts commissioned by the US
Department of Commerce are examining how the
C) if you keep changing your skin-care products policies concerning limited access to sensitive
technological information should be reformulated.
D) as you have a problem limited only to the skin
B) A group of experts have been appointed by the US
E) before you become worried about wrinkles Department of Commerce to review the policies
that limit access to confidental technological
information.
34. Cars entail a great many expenses ----..
C) The US Department of Commerce has recruited a
group of experts to find out to what extent the
A) so long as one doesnt use them excessively policies for the limitation of access to delicate
technological information can be revised.
B) unless one could get a sufficient loan from the bank
D) The US Department of Commerce has
C) in case a number of people cannot afford them commissioned a group of experts to examine
whether policies on limiting access to sensitive
D) so its worth thinking carefully before buying one technological information should be reviewed.
E) however essential they were thought to be in E) The policies on limited access to confidental
business life technological information are being reviewed by a
group of experts, appointed by the US Department
35. Do not attempt to cut down or prune large trees of Commerce.
----..

A) just as branches that appeared healthy had in fact


been rotten inside

B) unless you are absolutely confident that you can do


it properly

C) by making sure that your neighbours were not


opposed to it at all

D) if the weather conditions had not been taken into


account

E) in case the manager rejected the findings


38. spanyada on dokuzuncu yzyl sonlar, yaygn 41. The Old English language, also called Anglo-
siyasal yolsuzluklara bir tepki olarak gelien Saxon, can only be read today by those who have
anarizmle birlikte ulusal bir k dnemiydi.. made a special study of it..

A) The later decades of the nineteenth century in A) Anglo-Sakson ad verilen eski ngiliz dili, bugn,
Spain were a period of national decline as well as zel renim grm kiiler tarafndan kolayca
anarchism, which emerged as a reaction against okunabilmektedir.
extensive political malpractice.
B) Anglo-Sakson ad da verilen ngiliz dili, bugn
B) The late nineteenth century in Spain was a period sadece zel renim grm ok az kiinin
of national decline, along with anarchism okuyabildii bir dil durumuna gelmitir.
developing as a response to widespread political
corruption. C) Ancak zel renim grenlerin anlayabildii eski
ngiliz diline, bugn Anglo-Sakson da denilmektedir.
C) As a response to extreme political decadence,
anarchism developed in Spain in the late D) Bugn Anglo-Sakson ad verilen eski ngiliz dilini
nineteenth century, which was a period of national okuyabilmek iin, bu dilde zel renim grm
decline. olmak gerekmektedir.

D) Anarchism in Spain came into being in the late E) Anglo-Sakson ad da verilen eski ngiliz dili, bugn
nineteenth century, also known as a period of sadece, bu dilin zel renimini grm olanlar
national decline, as a response to widespread tarafndan okunabilmektedir.
corruption in politics.

E) The end of the late nineteenth century in Spain 42. When animals are used for research purposes, it
marked a period of national decline, which was also is not the taking of life that people object to, but
noted for anarchism developing as a struggle the suffering that is caused..
against extreme political corruption.
A) Hayvanlar aratrma amacyla kullanldnda
39. teki Avrupallara kyasla, Almanlarn daha uzun insanlarn kar kt, can alnmas deil, sebep
yllk tatilleri olduu ve yurt d gezilere daha olunan acdr.
ok para harcadklar bir gerektir..
B) nsanlar, aratrmalarda kullanlan hayvanlarn
ektii acdan ok, onlarn cannn alnmasna kar
A) It is true that, compared with other Europeans, the karlar.
Germans have longer annual holidays and spend
more money on foreign travel. C) Aratrmalarda kullanlan hayvanlarn canlarnn
alnmasndan ok, insanlar onlarn ac ekmesi
B) It is admitted that, comparatively, the Germans zer.
enjoy longer annual holidays than other Europeans
and save a lot of money for their foreign travels. D) nsanlarn aratrmalarda hayvanlarn
kullanlmasnda kar ktklar ey, onlarn cannn
C) The fact that, unlike other Europeans, the Germans ac ektirilerek alnmasdr.
are entitled to longer holidays each year and put
aside more money for their travels abroad is E) nsanlar, aratrma amacyla hayvanlar
undeniable. kullanldnda, hem onlarn canlarnn alnmasna
hem de ac ekmelerine kar karlar.
D) Truly, contrary to other Europeans, the German
people tend to have longer annual holidays and are
prepared to spend more money on their travels
abroad.

E) When the Germans are compared with other


Europeans, it is clear that their annual holidays are
longer and the money they spend for their travels
abroad is much more.

40. A great many inventions have been made


through history, but the invention of the
computer has probably been more influential on
our way of life than any other..

A) Tarih boyunca birok bulu yaplmtr; ancak,


bilgisayarn icad, yaam biimimiz zerinde
muhtemelen dierlerinden daha fazla etkili
olmutur.

B) Tarihte birok bulu yaplmtr; ancak, bilgisayarn


icad, yaammzda ok byk bir etki yapmtr.

C) Tarih boyunca ok sayda keif vardr; ancak


aslnda, yaam biimimizi muhtemelen en fazla
etkileyen yenilik, bilgisayarn icad olmutur.

D) Tarihte yaplm olan bulularn says oktur;


ancak, hibiri yaam biimimiz zerinde
bilgisayarn icad kadar etkili olmamtr.

E) Tarihte eitli bulular yaplmtr; ancak,


bilgisayarn icadnn yaam biimimizi dierlerinden
ok daha fazla etkiledii grlmtr.
45. According to the passage, Washington University
43. - 46.sorular aadaki paraya gre
researchers ----..
cevaplaynz.

A) have been harshly criticized by their colleagues


around the world
Asthma is a life-threatening, allergy-driven lung disease
common in wealthy countries. But exactly what causes it B) are strongly of the opinion that there is a link
is unknown. Researchers at Washington University believe between eczema and asthma
the direct cause of asthma is a chemical distress signal
produced in skin that is damaged by another hazard of C) feel that they still have to carry out new
modern life:eczema. Unlike asthma, it is not dangerous, so experiments as regards asthma
people rarely worry about it. Nevertheless, 17% of
D) have already come up with a set of solutions to
children in America have it, and similarly high figures are
reduce the rate of eczema among American
found in Australia, Britain, and New Zealand. What is children
particularly intriguing is that many people with eczema go
on to develop asthma (in America, the figure is 70%). That E) have collaborated with their British, Australian and
compares with an asthma prevalence of 4-8% in the New Zealander colleagues to study eczema and
general population. The Washington University group asthma
theorizes that the link between the two conditions is
formed by thymicstromal lympho-poietin (TSLP), a
46. It is clearly pointed out in the passage that
signalling molecule secreted by damaged skin cells that asthma ----..
elicits a strong immune response from the body to fight
off invaders.Thus, eczema-induced TSLP enters the
bloodstream and, when it arrives at the lungs, sensitizes A) is most widespread in countries with prosperous
them so that they react to allergens that would not populations
previouslyhave bothered them. In other words, they
B) always causes serious damage in the lungs, which
become asthmatic. Several experiments carried out by the
try to fight it off
researchers, only on mice, have confirmed that skin
damage creates susceptibility to asthma by releasing C) has only recently become a serious concern among
TSLP. researchers in English-speaking countries

D) causes a great deal of TSLP secretion in the body,


which leads to skin-allergy
43. It is clear from the passage that a causative
relationship between skin damage and asthma - E) is generally ignored among the people who are not
---.. wealthy enough to seek medical aid

A) has been proven only in English-speaking countries

B) would mean that allergens would no longer play a


role in asthma attacks

C) has been established in mice but not yet in human


beings

D) means over-exposure to the sun may hurt the


lungs

E) does not provide immunity to sufferers of both


conditions

44. The passage points out that the prevalence of


eczema among children in America ----..

A) is related to their distress levels

B) is much higher than it is in other English speaking


countries

C) may be due to peoples failure to worry about it

D) results from the abundance of TSLP in that country

E) is much higher than that of asthma in the general


population
50. As is understood from the passage, in the past,
47. - 50.sorular aadaki paraya gre
football was not a good example of globalization
cevaplaynz. because ----..

A) there werent many national football clubs playing


Not long ago, football was not a good example of internationally
globalization. The labour market in international club
football was highly protected. National leagues like Italys B) most players didnt want to leave their native
Serie A and Spains La Liga imposed quota son their country
teams, allowing them to import only a limited number of
players. Some teams could have only two foreign players C) most teams werent allowed to have many foreign
on the field. This arrangement, however, began to players
crumble in 1995, when the European court ruled that the
D) the number of clubs throughout Europe was limited
difference of treatment of nationals from other EU
countries was anti constitutional.This permitted players to E) foreign players could not cross the borders to play
move freely within the EU, and made the club teams much in other countries
more multi-national. Now it is not unusual for a majority of
the players on a successful league team to beforeign
nationals.

47. It can be understood from the passage that the


quotas imposed in the past by national leagues -
---..

A) pushed their teams toward a more global approach

B) allowed many foreign nationals to enter the labour


market

C) created a closed labour market in international club


football

D) were approved by the European court

E) made the club teams quite multi-national

48. An example of footballs globalization given in


the passage is ----..

A) the large number of foreign players on many teams


today

B) the ruling by the European court in 1995

C) the quotas that many national leagues still impose


on their teams

D) that the Italian teams exchange their players more


frequently than the other teams do

E) that the number of football teams in the EU


countries has increased since 1995

49. According to the passage, the European court


ruling of 1995 ----..

A) created a new constitution

B) resulted in a complete reorganisation of the


European football leagues

C) restricted the number of foreign players a team


could import

D) made mandatory the inclusion of at least two


foreign players from outside the EU on every
national team

E) made it illegal for EU teams to refuse to employ


foreign players simply because they were foreign
54. This passage is mainly concerned with ----. .
51. - 54.sorular aadaki paraya gre
cevaplaynz.
A) the illegal construction of a new archaeological
park in Agrigento, Sicily
Each year in the touristic town of Agrigento, Sicily, B) the beautiful ancient temples found in Agrigento,
hundreds of illegally-built houses are bulldozed by the Sicily
local government. New construction in Agrigento, home to
many ancient temples which tourists come to see, has C) the pollution of the archaeological park in
been banned since 1968. In spite of this, hundreds of new Agrigento, Sicily, which began in 1968
and half-built houses can be seen in the hills surrounding
the archaeological park. Not only do these buildings spoil D) the reasons why illegal construction is still
continuing in Agrigento, Sicily
the landscape, but many are also unsafe and unsanitary.
Some of the people living in these buildings pour sewage E) the problem of illegally-built houses in Agrigento,
into the sea and pile garbage on roadsides since their Sicily
houses are illegal and they arent allowed to use the city
sewage system and garbage service. Several of these
houses are also built on dangerous cliffs, sites that would
never be allowed by Italys strict building codes.

51. It is made clear in the passage that pouring


sewage into the sea and piling garbage on
roadsides are examples of ----. .

A) the reason these houses are built in the hills

B) compliance with Italys strict building codes

C) why the illegally-built houses in Agrigento are


unsanitary

D) the beautification of Agrigentos archaeological


park

E) a protest against the Italian government

52. According to the passage, the building


prohibition in Agrigento, Sicily, ----..

A) has meant that no new houses have been built


there since 1968

B) has not put a stop to the building of new houses

C) has meant that there are no houses on the


surrounding hills

D) has caused the destruction of many ancient


temples

E) has led to the strict control of new construction

53. It can be inferred from the passage that the local


government in Agrigento, Sicily, does not want
new houses to be built near the archaeological
park because ----..

A) the local government wants to use the land to build


houses for its officials

B) the local government cannot collect taxes from the


owners of these houses as they are built on public
land

C) there is no water, sewage or garbage service


available near the temples

D) new houses spoil the landscape around the ancient


temples

E) the local government is planning to build several


new hotels for tourists on the land
57. We learn from the passage that in the opinion
55. - 58.sorular aadaki paraya gre
ofHalley, ----..
cevaplaynz.

A) the mystery surrounding comets could never be


cleared up
For almost 200 years, the idea of cosmic eventsaffecting
life on Earth was viewed as heretical by thechurch, which B) Newtons laws of motion and gravitation needed to
regarded catastrophe as proof ofdivine intervention, and be further clarified and elaborated
as nonsense by the scientificestablishment, which
dismissed it as superstition. Yetin the end, the sheer C) the earlier appearances of the comet Halley had
weight of evidence has sweptaway all doubt about the not been properly recorded
reality of globalcatastrophes. Attempts to make scientific
D) global catastrophes could be prevented through
sense ofthe many legends of global catastrophes date
new scientific developments
back tothe dawn of modern science itself, in the 17th
century.Following the publication of Newtons laws of E) a comet may, at some point in the future, strike
motionand universal gravitation in 1687, Edmond Earth
Halleydecided to apply them to the mystery of comets.
Bystudying records of their appearance, Halley arguedthat
58. It is clear from the passage that, in the past,
the bright comets of 1456, 1531, 1607 and 1682were in
thechurch ----..
fact one comet, later known as the Halleycomet, that
followed a vast elliptical orbit around theSun in agreement
with Newtons laws. But Halleynoted something else as A) regarded global catastrophes as acts of God
well: a comet crossing theorbit of the Earth might one day
collide with us withdevastating consequences. B) was particularly interested in the movements of
comets

C) consistently banned any research into cosmic


55. According to the passage, Newtons laws events
ofmotion and gravitation ----..
D) encouraged scientists to find ways of preventing
global catastrophes
A) had no impact whatsoever on the rise of modern
science E) wished to suppress all thoughts of cosmic events

B) were approached skeptically by Halley and other


contemporary scientists

C) helped Halley to identify the comet that bears his


name

D) convinced Halley that catastrophes were in fact


acts of divine intervention

E) were dismissed right away by the scientific


establishment of his time

56. It is pointed out in the passage that, prior to


therise of modern science in the 17th century, -
---..

A) various studies had been made of comets, but


Halley disregarded them all

B) the way people viewed cosmic events varied


greatly

C) all kinds of learning had been subject to the


approval of the church

D) the Earth had experienced several collisions with


cosmic objects

E) attempts had been made to explain certain


catastrophes with reference to gravitational laws
62. This passage is concerned with ----..
59. - 62.sorular aadaki paraya gre
cevaplaynz.
A) the exhibits of a wireless museum which cover a
considerable variety
The Wireless Museum has several of the earliest crystal B) the governments war efforts and production
wireless sets from the 1920s which ran on policies regarding radios
electromagnetic waves with no external power source,
and were easily made at home. Valve radios, which came C) the technical features of transistor radios
along in the 1930s, needed electricity to heat up the
valves and the museum has both main sand battery- D) the way a valve radio works
powered valve radios on display. The collection also has
E) why the wireless museum was originally set up
some rare war time civilian receivers the only type of
valve radio manufactured during the Second World War.
This was by order of the government, because at this time
63. - 67.sorularda, karlkl konumann bo
most manufacturing was focused on the war effort.There
braklan ksmn tamamlayabilecek ifadeyi
are also plenty of modern day transistor radios including a
bulunuz.
collection of novelty radios dating from the sixties and
seventies.

63. Sharon : I need a memory card that is compatible


with my laptop.
59. It is pointed out in the passage that, during
Shop Assistant : ----
World War II, ----..
Sharon : I think two gigabytes will be enough to
store all my files.
A) transistor radios began to replace traditional valve Shop Assistant : Then I suggest this one. It has
radios the memory volume you want, and it is quite
user-friendly..
B) the production of wireless sets was almost entirely
for military purposes
A) Do you have enough money for a two-gigabyte
C) various types of radios requiring no external power card?
source were developed
B) What is the operating system on your laptop?
D) the government banned all kinds of civilian
C) How much memory capacity do you need?
receivers
D) Where do you store your files?
E) the efficiency of valve radios was upgraded through
the introduction of new designs
E) What do you think about this memory card?

60. We learn from the passage that the very early


64. Senior Doctor :- I hear youve been doing
crystal wireless sets ----..
research on alcoholism in Turkey.
Junior Doctor :- Yes. I feel the findings are not all
A) were the models out of which transistor radios were that alarming.
later developed Senior Doctor :- ----
Junior Doctor :- In a way, yes. For instance,
B) were manufactured in large quantities before the nearly 8 per cent of adults in the US have a
arrival of valve radios serious problem with alcohol use..

C) are among the museums most precious exhibits


A) Really? You say that in comparison with other
D) were still in use during World War II, though in developed countries?
limited numbers
B) Arent they? Its a fact that alcohol produces both
E) were unconnected to an outside power system psychological and physical dependence.

C) In your report, you claim that men are four times


61. It is clear from the passage that valve radios - more likely than women to become alcoholics.
---..
D) In general, alcoholics often cant manage their
behaviour and tend to drive while drunk.
A) were still in widespread use in the sixties and
seventies E) Well, alcohol is rapidly absorbed from the small
intestine into the blood stream.
B) originally operated on electromagnetic waves

C) consumed more electricity than one might expect

D) are of two types: mains and battery-powered

E) were costly products and the government


disapproved of them
65. Marian :-Do you remember Chinas one child 67. George: This summer, I spent my holiday in
policy, where each family was allowed to produce Spain. I was particularly impressed by Madrid.
only one child in order to reduce the countrys Mark : I am sure you were. While there, you must
population? have found time enough to visit the Prado
Helen :-Yes, of course I do. The policy has Museum.
created a huge imbalance in the male-to-female George : ----
population ratio, due to the fact that many Mark : Originally, all these paintings, especially
couples abort their female foetuses. the major works of Velsquez and Goya, were in
Marian :---- the royal collection..
Helen :-It did, but only recently, and many
couples are still doing it even though its against
the law. . A) Of course, certainly, because it houses the worlds
greatest collection of Spanish paintings from the
12th to the 19th centuries.
A) I think that abortion should be allowed only in
medical emergencies. B) Actually, the origins of the city date back to A.D.
852, when the Moors built a fortress near the
B) How can they bring themselves to do that? Manzanares river.

C) Why dont they want to have a girl? C) In fact, it was under the Habsburgs that the city
acquired some of its most notable landmarks,
D) Didnt the couples think before deciding to have a including its splendid palaces.
child?
D) Let me tell you that the museum was designed in
E) But I thought the Chinese government made 1719 by the royal architect Juan de Villanueva.
abortion illegal.
E) In Spanish culture, the 18th century was an era of
French influence, following the Bourbon accession
66. Sarah : The book you are reading is about Brazil, to the Spanish throne.
isnt it? I know a little bit about this countrys
economy.
Henry : It is, indeed. It says, since 1930,
68. - 71.sorularda, verilen cmleye anlamca en
successive governments have persistently
pursued policies for economic growth and yakn cmleyi bulunuz.
Amazon area development.
Sarah : ----
Henry : That explains partly why by the 1990s
Brazil has become one of the worlds largest 68. Despite major reforms, small businesses still find
economies.. it difficult to raise capital, and banks lend mostly
to established companies..

A) Unfortunately, development has destroyed much of


the ecosystem that has characterized the Amazon A) Due to the fact that banks lend mostly to
basin. established companies, small businesses are still
facing problems and going through important
B) This is not surprising, since Brazil has vast natural changes.
resources and a huge labour force.
B) Major reforms did not help with the fact that small
C) Geographically, the north eastern region of Brazil is businesses still experience problems in raising
semiarid scrubland, heavily settled and poor. capital, and banks lend mostly to bigger
companies.
D) However, it is true that today more than one out of
four Brazilians survive on less than $1 a day. C) Small businesses find it problematic to raise
capital, and banks usually lend to bigger companies
E) Well, Brazils narrow coastal belt includes most of as a result of some certain regulations.
the major cities, of which Rio de Janeiro is the most
famous one. D) Because of major reforms, banks lend mostly to
established companies, making it hard for small
businesses to raise capital.

E) Although there have been vital improvements,


banks do not lend to all types of companies, as it is
difficult for some of them to raise capital.
69. The changing climate will have negative effects
72. - 75.sorularda, bo braklan yere, parada
on all parts of the world; depending on peoples
location and lifestyles, however, there will be anlam btnln salamak iin
great differences in the subsequent health getirilebilecek cmleyi bulunuz.
hazards that human populations face..

A) No matter how and where people live, the 72. Cloning could be crucial when a species is
subsequent health hazards will be terribly great threatened with extinction. For instance, a few
after the varying climate negatively affects all years ago, the last remaining bucardo mountain-
regions of the world. goats in Spain were rounded up for a captive
breeding programme. ----. This species is now
B) Based on their lifestyles and geographical location, extinct but could have been saved by the cloning
human populations all over the world will technology we have today..
experience health risks to be brought about by
adverse effects of the changing climate.
A) Unfortunately, however, they were wiped out by
C) Whether all regions of the world will be negatively disease
affected by the incremental climate change largely
depends on peoples location and ways of life, yet B) Habitat protection is the cornerstone of
human populations will end up with health r conservation

D) Since all parts of the world are likely to be C) Scientists made the world's first healthy clone of an
adversely influenced by the globally changing endangered species in 2003
climate, human populations have been subject to
resultant health risks, regardless of how and where D) Literally a hundred species become extinct every
they day

E) All regions of the world will be adversely affected E) There are cases in which cloning may prevent
by the changing climate, but the resulting health extinction
risks to human populations will vary greatly,
depending on where and how people live.
73. The turn of the twentieth century brought a
series of crises to the Western empires. Those
70. Everyone in our class is doing something at the crises did not end European imperial rule in the
end-of-term concert, but Mary alone is staying colonies. ---- The crises also drove these nations
away.. to expand their economic and military
commitments in what they called their territories
overseas. However, because of these crises, the
A) At the concert at the end of term, Mary is going to colonial confidence of these nations was much
represent our class. shaken..
B) No one in our class but Mary, is taking part in the
end- of-term concert. A) In recent years historians have become
increasingly interested in colonial cultures and the
C) Everyone in Marys class hopes to do something at results of the imperial encounter across the world.
the end-of-term concert.
B) Writers such as Joseph Conrad believed that
D) Mary is the only one in our class who isnt taking imperialism signalled deeply rooted prejudices in
part in the endof- term concert. European culture.

E) The class wants Mary to play in the concert at the C) They did, however, create sharp tensions between
end of term, but she wont. Western imperial nations which had already got
into fierce rivalry for territorial gains.
71. Even if you have never touched a cigarette in
D) As a result of colonialism, European and indigenous
your life, you are still at risk from smoking
institutions and cultures were transformed by their
related diseases if you live, work or travel with
contact with each other.
smokers..
E) Especially in China and India, there were fierce
debates about whether education should be
A) Among the people you have to live if there is a
westernized or continue on traditional lines.
person smoking, you are prone to catch a disease
connected with smoking just because youve
exposed to smoke.

B) Whether or not you have smoked so far, because a


comrade of yours smokes, you are likely to suffer
from smoking related illnesses.

C) Unless you give up smoking under this


circumstance, you may suffer from some illnesses
correlated with smoking.

D) Supposing that you havent smoked even a


cigarette in your life, how come you may catch a
serious illness just because you have been exposed
to smoke?

E) In order not to catch an illness linked to passive


smoking, you had better not contact with your
friends so frequently who smoke a lot.
74. The world literacy rate has risen sharply in
76. - 80.sorularda, cmleler srasyla
recent decades. Today a greater portion of the
worlds population is able to read and write than okunduunda parann anlam btnln
ever before. ----. This is true even in developed bozan cmleyi bulunuz.
countries such as Italy. In the United States,
demand for books is increasing among the
middle-aged population, but young Americans,
as studies indicate, dont spend much time 76. (I) Systemic anaphylaxis is a dangerous allergic
reading for leisure.. reaction that can occur when a person develops
an allergy to a specific drug such as penicillin.
(II) In serious allergic disorders, patients are
A) In addition, the population in most developed sometimes given a form of immunotherapy
countries is getting older and older by the day known as desensitization. (III) A widespread
allergic reaction takes place within minutes after
B) For example, the rate of literacy has gone up the drug enters the body. (IV) Mast cells release
considerably in the least developed parts of the large amounts of histamine and other
world, such as the sub-Saharan Africa compounds into the circulation, and these
compounds cause extreme vasodilatation and
C) Similarly, the rate of child births has been permeability. (V) So much plasma may be lost
decreasing over the past few decades from the blood that circulatory shock and death
can occur within a few minutes..
D) Still, many people have trouble reading
intermediate to upper-intermediate level texts in
their native languages A) I

E) On the contrary, in most developing and B) II


underdeveloped countries, education is compulsory
for all children C) III

D) IV
75. Before it was privatized, Trk Telekom was a
national monopoly with exclusive rights to all
E) V
fixed-line voice operations. ---- Reforms since the
early 1990s have led to the introduction of three
new mobile telephone companies. A series of
private companies that provide services such as
Internet access and cable television have also
been introduced..

A) Turkey has failed to make advances in


telecommunications.

B) It was a private enterprise owned by several rich


partners.

C) The Internet had not yet come into use.

D) The use of cable television has been very popular


in Turkey.

E) It also provided cable services for television.


77. (I) As forests are cut down, many wild animals
are finding it harder to survive. (II) Sadly the
range of the leopard, for instance, is but a
fraction of what it formerly was. (III) They are,
for instance, no longer to be found in Europe
proper. (IV) Fortunately, there are instances of
such endangered species managing to re-
establish themselves. (V) In the Caucasus and
northern Iran there are still some, but their
numbers are decreasing rapidly..

A) I

B) II

C) III

D) IV

E) V

78. (I) Boston High School is Americas oldest and


one of its finest public schools. (II) Indeed Boston
is a city with large black population. (III) Sarah
Wessman, who is 14, wants to go there. (IV) She
applied last autumn and was rejected. (V) That
might have been the end of the story except that
Sarah found she was one of ten white candidates
rejected by Boston High School while ten black
candidates with lower scores wore admitted..

A) I

B) II

C) III

D) IV

E) V
79. (I) Archeological evidence has shown that the
earliest libraries were built by the Sumerians. (II)
All libraries are classified to facilitate reference,
and the favorite system is the Dewey Decimal
System, which divides the whole field of
knowledge into ten main classes. (III) These are
General Works, Philosophy, Religion, Sociology,
Philology, Natural Science, Useful Arts,
Literature, and History. (IV) Each of these main
classes is again subdivided into ten main
divisions. (V) Then, each division is marked by
decimals within itself..

A) I

B) II

C) III

D) IV

E) V
80. (I) Today the worst problem facing the
government of South Africa is unemployment. (II)
This now affects a third of the population and is
rising rapidly. (III) Actually, the end of South
Africa's isolation from the world meant that
companies had to cut jobs to be competitive. (IV)
Moreover, in order to meet the government's
tight deficit targets, there are now cuts being
made in the number of the public-service staff,
which makes unemployment even worse. (V) In
the economic sector, the situation is even more
hopeful for there has been a wave of reforms and
mergers among banks and insurance companies..

A) I

B) II

C) III

D) IV

E) V
SORU CEVAP SORU CEVAP

1 B 41 E

2 C 42 A

3 A 43 C

4 D 44 E

5 A 45 B

6 B 46 A

7 A 47 C

8 E 48 A

9 E 49 E

10 E 50 C

11 A 51 C

12 E 52 B

13 D 53 D

14 C 54 E

15 D 55 C

16 D 56 B

17 E 57 E

18 B 58 A

19 D 59 B

20 E 60 E

21 C 61 D

22 E 62 A

23 C 63 C

24 C 64 A

25 A 65 E

26 E 66 B

27 D 67 A

28 C 68 B

29 E 69 E

30 A 70 D

31 A 71 B

32 A 72 A

33 C 73 C

34 D 74 D

35 B 75 E

36 B 76 B

37 D 77 D

38 B 78 B

39 A 79 C

40 A 80 B
17
5. A vitamin deficiency can cause normal body
1. - 16.sorularda, cmlede bo braklan yerlere
functions to ---- and render a person susceptible
uygun den szck ya da ifadeyi bulunuz. to disease..

A) break down
1. Some temperate environments have mild winters
with abundant ----, combined with extremely dry B) hold on
summers..
C) go with

A) heat D) keep out of

B) harvest E) rely on

C) rainfall

D) supply

E) growth

2. The lead from exhaust fumes enters the


atmosphere, mostly as simple lead compounds,
which are poisonous to children's ---- nervous
systems..

A) developing

B) domestic

C) exclusive

D) increasing

E) repeating

3. Each species has the capacity to produce more


offspring than will ---- to maturity..

A) conform

B) present

C) recognize

D) suggest

E) survive

4. Public health strategies regarding nutrition


are based largely on a biomedical model of diet
that requires individual consumers to comply ----
with dietary advice. .

A) regretfully

B) anxiously

C) voluntarily

D) adversely

E) coincidentally
6. The subject of the statistics is ---- the calculation
of the forces acting on and within structures that
are in equilibrium..

A) taken care of

B) turned up

C) kept off

D) given rise to

E) concerned with

7. When the Italian astronomer Giovanni


Schiaparelli ---- a map of Mars in 1877, he ---- a
large number of straight linear features, which
he called 'canali' that is 'channels'..

A) publishes / has noted

B) had published / was noting

C) has published / notes

D) published / noted

E) would have published / had noted

8. With the help of satellite images, the Indonesian


government ---- that it is the timber companies
which ---- the mass destruction of their rain
forests..

A) will show / would have caused

B) would have shown / had caused

C) had shown / were causing

D) shows / had been causing

E) has shown / are causing

9. On the night we went to the opera, Domingo was


not singing, but ---- we enjoyed ourselves
enormously..

A) in case

B) on the contrary

C) moreover

D) whatever

E) nevertheless
10. No one knows for certain whether or not people
today lie ---- they did in the past..

A) so that

B) so far as

C) as long as

D) for so long

E) more than
11. A series of gas discoveries in recent years in the 15. Species of mammals have developed varying
Western Desert of Egypt means that a range of adaptations ---- the different environments in
new export projects must be developed ---- the which they live..
country is to make full use of its new reserves..

A) such as
A) lest
B) by all means
B) although
C) including
C) after
D) due to
D) as if
E) in response to
E) if

12. I'm wondering what ---- in World War II if Britain -


--- to the Germans before Operation Barbarossa
took place..

A) will happened / surrenders

B) would have happened / had surrendered

C) could have happened / was to surrender

D) happened / surrendered

E) happens / is to surrender

13. The influence ---- Freud ---- the generation that


followed was even stronger..

A) through / over

B) from / to

C) off / towards

D) by / upon

E) of / on

14. Moreover, ---- its own statutes, the EU itself is


obliged to consult the trade unions ---- a number
of topics..

A) under / on

B) with / at

C) from / for

D) on / against

E) to / over
16. The Internet has recently become a global
common platform ---- organizations and
individuals communicate among each other to
carry out various activities..

A) where

B) that

C) which

D) when

E) why
21. V.
17. - 21.sorularda, aadaki parada
numaralanm yerlere uygun den szck ya
da ifadeyi bulunuz. A) was not

B) has not been

It has been nearly two years sincethe last cases of SARS C) must not be
were reported in China. (I) ----,a new affliction hasrisen to
take its place as a more deadly pandemic avian flu. Also D) is not
known asH5N1, this influenza virus is endemic to
waterfowl and has shown a disturbingpropensity (II) ----the E) ought not to be
past nine years to infect chickens and human
beings.Avian flu is the Ebola of the poultry world, a
haemorrhagic fever that (III)---- much bleeding from every
orifice of its winged victims. It leads toextensive
destruction of these animals in (IV) ---- two days. When
the virusjumps to humans, it (V) ---- very noticeable at
first, but in fact, has afatality rate as high as 33 per cent.

17. I.

A) As a result

B) Accordingly

C) Moreover

D) Likewise

E) Since then

18. II.

A) over

B) through

C) at

D) by

E) from

19. III.

A) upholds

B) spreads

C) causes

D) implements

E) consists

20. IV.

A) just

B) any

C) most

D) several

E) still
26. V.
22. - 26.sorularda, aadaki parada
numaralanm yerlere uygun den szck ya
da ifadeyi bulunuz. A) Even if

B) Though

Hans Eysenck was one of the mostcontroversial and C) Therefore


prolific psychologists (I) ---- the twentieth century. (II)---- in
Germany, he went to live in Britain in the 1930s. At the D) As if
time of hisdeath in1997, he was one of the most (III) -
---researchers in psychology, withhundreds of references E) In case
made to his work. Eysenck (IV) ---- the notion that
therewas a biological basis for personality. (V) ---- his work
on the biologicalbasis has been frequently criticized, it has 27. - 36.sorularda, verilen cmleyi uygun
also been increasingly validatedby research. ekilde tamamlayan ifadeyi bulunuz.

22. I. 27. ---- because they mark important milestones


in the search for planets orbiting other stars..

A) of
A) At present the number of known exoplanets
B) to exceeds 100

C) at B) One of the newly-discovered exoplanets is similar


to Jupiter in mass and orbit
D) on
C) The recent discovery of two new exoplanets has
E) among attracted a great deal of attention from
astronomers

23. II. D) One of these exoplanets has a mass of about 40


times that of Earth

A) Bearing E) The two exoplanets recently discovered are the


lightest yet found orbiting a solar-type star
B) To have been born

C) Having born 28. ---- that is open to the amateur as well as


the professional..
D) Born

E) To be born A) Today we know a lot about meteor showers and


planetary weather

24. III. B) Astronomy is one of the few sciences

C) The mapping of solar, lunar and planetary surfaces


A) cited has been accurately carried out
B) enforced D) Thanks to advances in modem astronomy, more
discoveries have been made about asteroids,
C) collected comets, novae and supernovae
D) perceived E) Through the use of computers and electronic
imaging devices, our knowledge of the celestial
E) explained
constellations has grown rapidly

25. IV.
29. Though humanoids are the most
appealing robots under development, ----..
A) set upon
A) the term \'robot\' first entered the English language
B) kept up
from the Czech word for \'slave\' or \'forced
laborer\'
C) took on
B) the majority of existing robots do not look even
D) got through
remotely human
E) put forth
C) in the early 1980s dozens of robotics firms were
founded and quickly went bust

D) robots need to have a human form and interact


benignly with humans

E) fire fighting is an exciting new area for robotics


30. Once the pollen season starts, ----.. 34. ----, he would have to face a great deal of
criticism from his own party..

A) in most people, allergic conjunctivitis is part of a


larger allergy syndrome, such as seasonal rhinitis A) Unless he decides to make a public apology for the
way he has behaved
B) the different types of allergic reactions are
generally categorized by what causes them B) However trivial the matter clearly seemed to you

C) antihistamines or decongestants are the usual C) If he were to object to these amendments to the
initial treatment for seasonal allergic rhinitis Housing Bill

D) people who have severe adverse effects from D) Until people started to forget this rather scandalous
taking drugs should consider allergen affair
immunotherapy
E) As no ones attention is presently centred upon the
E) the nose, the roof of the mouth, the back of the budget
throat, and the eyes start to itch

35. ----, scientists are entirely dependent on their


31. ---- until James Watson and Francis Crick instruments to hear the toothed whales clicks..
proposed a model for its structure that had
extraordinary explanatory power..
A) Since humans can hear only sounds between 20
and 20,000 hertz
A) Many genes encode proteins that are not enzymes
B) As the vessel has made both acoustic and visual
B) A great deal was known about the physical and observations of whales
chemical properties of DNA
C) While other whales use sound to hunt, orient
C) DNA was not widely accepted as the genetic themselves, and communicate
material
D) Given that sound travels easily through air and
D) DNA is made of two polynucleotide chains water
intertwined to form a double helix
E) Because the blue whale\'s vocalizations are very
E) The idea that genes and enzymes are related in easy to recognize
some way was first clearly stated in 1908

36. English and German are widely spoken in


32. Atoms have several properties ----.. Denmark, ----..

A) in case they have different numbers of protons and A) so visitors should have few problems in making
electrons themselves understood

B) in which they are the smallest particles of all B) whereas it is best known for its 20th century design
matter and craftsmanship

C) that help distinguish one type of atom from another C) as Danish is similar to Norwegian and Swedish

D) because physicists have split the atom into many D) just as there are some differences in meaning and
subatomic particles pronunciation

E) while the electrons in an atom are very much E) although in 1972, Denmark became the first
smaller than the protons and neutrons Scandinavian country to join the European
Community

33. The flow of private capital to eastern Europe and


Africa has decreased ----..

A) while to Asia and Latin America it has increased

B) because rich countries have started to suffer from


poverty

C) since eastern European and African countries do


not need it any more

D) as these countries have started transferring their


own private capital to richer countries

E) so long as the IMF has banned all help to


developing countries
39. ki galaksinin arpmas, evrenin ktlesine
37. - 42.sorularda, verilen ngilizce cmleye
hkmettii sanlan grnmez kara maddenin
anlamca en yakn Trke cmleyi, Trke bugne kadar elde edilen en iyi kantn salar..
cmleye anlamca en yakn ngilizce cmleyi
bulunuz.
A) Following the collision of two galaxies, there
appears the best evidence so far known of the
invisible dark matter which is believed to pervade
the mass of the universe.
37. Bebeklerin byme ve gelimesinin, yedikleri
yiyeceklerden etkilendiini hepimiz biliyoruz..
B) The collision of two galaxies provides the best
evidence yet obtained of the invisible dark matter
A) As far as we know, the kind of food babies eat has assumed to dominate the mass of the universe.
much influence on their growth and development.
C) The only evidence so far of the invisible dark
B) All of us know that the food babies eat can have matter thought to penetrate the mass of the
some adverse effect on their growth and universe is provided by the collision of two
development. galaxies.

C) We are all aware of the fact that babies growth D) It is from the collision of two galaxies that the best
and development depend a great deal on the kinds evidence yet of the invisible dark matter which is
of food they consume. assumed to hold together the mass of the universe
has been obtained.
D) We all know that the growth and development of
babies are influenced by the food they eat. E) The invisible dark matter which is thought to
dominate the mass of the universe is best
E) It is well known to us all that whatever food babies understood through the evidence provided by the
consume essentially affects their growth and collision of two galaxies.
development.

38. 930dan Norvele birletii 1262ye kadar


bamsz bir cumhuriyet olan zlanda,
bamszln1944 ylnda geri kazand. .

A) Though an independent republic since 930, Iceland


was taken over by Norway in 1262 and only
regained its independence in 1944.

B) Iceland, annexed by Norway in 1262, had been an


independent republic since 930, and became
independent again only in 1944.

C) Iceland had been an independent republic


between 930 and 1262 when it was invaded by
Norway, becoming a republic again in 1944.

D) Before it united with Norway in 1262, Iceland had,


as it is today and has been since 1944, been an
independent republic since 930.

E) Iceland, which was an independent republic from


930 to 1262 when it joined with Norway, regained
its independence in 1944.
40. Cutting down trees to build houses not only
damages the environment, but also threatens
human health..

A) Ev yapmak amacyla aalarn kesilmesi evreye


zarar vermekle kalmaz, insan saln da tehdit
eder.

B) Ev yaparken baz aalarn kesilmesi evreye zarar


verir, ayn zamanda insan saln tehdit eder.

C) Ev yapmak amacyla aalarn kesilmesi insan


saln tehdit etmese de evreye zarar verir.

D) Aalarn ev yapmak iin kesilmesi hem evreyi


hem de insan saln kt etkiler.

E) evreye zarar veren ve insan saln tehdit eden


nedenlerden biri aalarn ev yapmak iin
kesilmesidir.

41. No matter how much our level of technology


develops, the human race will always be at the
mercy of the forces of nature..

A) nsan rk, srekli doa glerinin insafna kald


iin teknoloji dzeyini srekli gelitirmeye
almaktadr.

B) Teknoloji dzeyimiz byk lde gelise de, insan


rk ou zaman doa glerinin insafna kalacaktr.

C) Teknoloji dzeyimiz ne kadar geliirse gelisin,


insan rk daima doa glerinin insafna kalacaktr.

D) nsan rk daima doa glerinin insafna


kalacandan, teknoloji dzeyimizin srekli
gelimekte olmas bir anlam tamaz.

E) Teknoloji dzeyimiz ne kadar geliirse gelisin,


doa gleri insan rkna hibir zaman insaf
etmeyecektir.
42. Lasting for 600 years, the Ottoman Empire was
not only one of the most powerful empires in the
history of the Mediterranean region, but it also
generated great works of art, architecture and
literature..

A) Akdeniz blgesinin tarihindeki gl


imparatorluklardan biri olan Osmanl mparatorluu
600 yl devam etmi ve sanat, mimarlk ve
edebiyatta byk eserler retmitir.

B) 600 yllk Osmanl mparatorluu, hem Akdeniz


blgesinin tarihindeki imparatorluklarn en gls
olmutur hem de sanat, mimarlk ve edebiyat
alanlarnda byk eserler retmitir.

C) 600 yl sren Osmanl mparatorluu, Akdeniz


blgesinin tarihinde en gl imparatorluklardan
biri olmasa da sanat, mimarlk ve edebiyatta byk
eserler vermitir.

D) Sanat, mimarlk ve edebiyat alanlarnda byk


eserler vermi olan Osmanl mparatorluu, 600 yl
devam etmi olsa da Akdeniz blgesinin tarihindeki
en gl imparatorluklardan biri deildir.

E) 600 yl devam eden Osmanl mparatorluu,


sadece, Akdeniz blgesinin tarihinde en gl
imparatorluklardan biri deildir, ayn zamanda
byk sanat, mimarlk ve edebiyat eserleri de
retmitir.
45. According to the passage, standard proceduresin
43. - 46.sorular aadaki paraya gre
a critical care unit ----..
cevaplaynz.

A) must be carried out as soon as a patient is


admitted
Patients are admitted to critical care units from a variety
of settings, including the emergency department, medical B) are primarily restricted to patients with
or surgical service, or operating room. Most critical care cardiopulmonary arrest
patients are acutely and severely ill, commonly with
dysfunction or failure of more than one organ system. The C) can begin only after the immediate initial
initial assessment must be rapid and focus on real or assessment has been completed
potentially life threatening processes that require
D) should never be limited even though the patient is
immediate intervention. An example is the resuscitation of
receiving immediate intervention
a patient with cardiopulmonary arrest. The pace of
resuscitation is necessarily quick; physical examination E) should be clearly defined before the patients case
may be restricted initially to the central nervous, is fully specified
cardiovascular, and respiratory systems, and interventions
may be limited to the essential ABCs of airway, breathing,
46. The passage states that critical care units ----..
and circulation. Later, continuous electrocardiographic
monitoring, measurement of blood pressure, and other
standard procedures should start. In general, A) only deal with patients having multi-organ
management of the critically ill patient should be based dysfunction or failure
on an understanding of physiology and
pathophysiology.Indeed, although the contributions of cell B) specialize in resuscitating those with
and molecular biology to critical care medicine are cardiopulmonary arrest
substantial, the critical care unit more resembles a
C) take into consideration the principles of physiology
physiology laboratory, since the effects of its interventions
when accepting patients
can be directly observed.
D) have benefited financially from cell and molecular
biology institutions

43. The passage explains that initial assessment in E) accept patients from several different areas within
the critical care unit ----.. a hospital

A) aims to resuscitate patients with cardiopulmonary


arrest

B) is only done for patients who are suffering from


various diseases

C) can itself become a life-threatening process if the


ABCs are not limited

D) must be immediate and centre on saving life

E) determines the ultimate survival of patients


suffering from heart disease

44. We learn from the passage that critical care


medicine ----..

A) rests primarily on the underlying disciplines of


physiology and pathophysiology

B) has received the greatest amount of support from


emergency departments

C) often must be based within an actual physiology


laboratory

D) revolves around the essential ABCs of airway,


breathing, and circulation

E) is necessarily concerned with the pace of


resuscitation in cardiopulmonary arrest
49. One understands from the passage that Pompeii
47. - 50.sorular aadaki paraya gre
----..
cevaplaynz.

A) was the only Roman city famous for its taverns and
shopping centres
The Roman city of Pompeii in A.D. 79 was a thriving
provincial centre, a few miles from the Bay of Naples, with B) was one of the Roman centres for the slave trade
a population of between 10, 000 and 20, 000 people. Its
narrow streets, made narrower by street vendors and C) had a very efficient water system
shops with cloth awnings for shade, were full of shoppers,
tavern-goers, slaves, and vacationers from the North. A D) was the second largest city in the Roman Empire
huge new aqueduct supplied running water from the
E) had been destroyed by volcanic eruptions several
Lower Apennine mountains, which flowed from fountains
times before A.D. 79
throughout the city, even in private homes. But the key to
Pompeiis prosperity, and that of smaller settlements
nearby like Oplont is and Terzigna, was the regions rich 50. According to the passage, what geologist Philip
black earth provided by Mount Vesuviusvolcanic Janey is actually saying in the part quoted is that
eruptions. One of the ironies of volcanoes is that they ----..
tend to produce very fertile soils, and that tends to tempt
people to live around them, says geologist Philip Janey. A) the city of Pompeii should have been founded on
Had Roman knowledge in the summer of A.D. 79 been less the other side of the Bay of Naples
mythological and more geological, the Pompeiians might
have recognized the danger signs from Mount Vesuvius B) volcanic terrain is most suitable for people to
and escaped the volcanic eruption that was to follow. settle and live on

C) people always prefer to settle in volcanic areas


since they believe the land there is more fertile
47. According to the passage, in A.D. 79, there had
been some geological indications that ----.. D) the Pompeiians knew that the area around Mount
Vesuvius was not a safe place to settle in, but they
settled there anyway
A) Mount Vesuvius was about to erupt, but the people
of Pompeii failed to understand them E) people attracted by the fertile lands around
volcanoes prefer to live there, ignoring the dangers
B) the Bay of Naples posed a serious danger to the of a volcanic eruption
city of Pompeii, but it was ignored by the
Pompeiians

C) the area in which the city of Pompeii was situated


was becoming less and less fertile

D) the water resources in the Lower Apennine


mountains were no longer adequate to supply
water to the city of Pompeii

E) Oplontis and Terzigna, the settlements near


Pompeii, were not safe to live in and, therefore, had
to be evacuated

48. It is emphasized in the passage that the


economic well-being of Pompeii ----..

A) reached its climax in A.D. 79, the year in which


there was a sharp increase in its population

B) primarily depended on the commercial activities of


its people as well as holidaymakers from the North

C) attracted all kinds of people with money, who


crowded its streets and led a carefree life

D) was essentially related to the fertility of its land,


which was due to the volcanic eruptions of Mount
Vesuvius

E) was the outcome of its exploitation of the


settlements around it such as Oplontis and
Terzigna
53. It is pointed out in the passage that the Angolan
51. - 54.sorular aadaki paraya gre
government ----..
cevaplaynz.

A) is working on multi-billion dollar deep water


investment programmes to be implemented in
Angolas emergence as a serious player in the global oil 2008
sector has been underlined by the publication of its latest
production figures. After several years of slow output B) feels that its 2008 oil production target may not be
growth, the fruits of the multi-billion dollar deep water realized due to the high costs of deepwater
investment programme are finally feeding through. Thus, investments
production reached 1.3 million barrels a day during the
final quarter of 2005. With the new deep water fields now C) is resolved to challenge only Libya for oil leadership
in Africa
coming into production, the governments target of
producing 2 million barrels a day by 2008 now seems D) has invested huge sums for the production of oil
eminently within reach. In the meantime, official Angolan from its deepwater fields
government figures indicate that oil production averaged
1.25 million barrels a day during the course of 2005, a E) announced that it has published its oil figures in
steep jump on the year before. In addition, the order to prove to the world that it is a serious
governments estimate of proven oil reserves has finally player in the global oil sector
beenincreased from 5.4 billion barrels to 12.4 billion
barrels. This shows that, as sub-Saharan Africas second 54. It is pointed out in the passage that, according to
biggest oil producer after Nigeria, Angola is in an excellent the Angolan government, ----..
position to overtake countries such as Libya and Algeria in
the table of oil powers on the continent as a whole.
A) the countrys verified oil reserves are more than
twice the original estimate

51. It is suggested in the passage that, with its rich B) the country owns the largest and most productive
oil reserves and rapidly increasing oil production, deepwater oil fields in sub-Saharan Africa
Angola ----. .
C) Angola has already become the largest oil producer
in all of Africa
A) has become one of the major oil powers in Africa
D) after 2008, Nigeria and other oil producing
B) has already completely left behind such major oil countries in Africa will no longer be major players in
producers as Nigeria, Libya and Algeria the global oil sector

C) has emerged as a threat in sub-Saharan Africa as E) the countrys deepwater oil reserves have finally
well as the global oil sector been verified but are too costly to be exploited

D) can be regarded as a long-time constructive player


in the African oil league

E) has always been in fierce competition with Nigeria


and other sub-Saharan African countries

52. One understands from the passage that Angolas


previously poor oil output ----. .

A) can only be improved through the exploitation of


its deepwater reserves

B) has improved dramatically and reached a record


level in 2005

C) is related to Nigerias dominant position in the sub-


Saharan oil sector

D) prevents it from competing efficiently with other oil


producing countries

E) does not make it a current major player in the


global oil sector
57. According to the passage, solid carbon dioxide -
55. - 58.sorular aadaki paraya gre
---..
cevaplaynz.

A) differs from liquid carbon dioxide in that it has a


less significant cooling effect
Carbon dioxide (CO2), like water and most other pure
substances, exists in solid, liquid, and gaseous statesand B) changes right away into the gaseous state at -
can undergo changes from one state to another.Solid CO2, 78C, without first melting into the liquid form
however, has an interesting property: at normal
pressures, it passes directly to the gaseous state without C) absorbs more heat than the liquid and gaseous
first melting to the liquid state. This property, together forms
with the fact that this change occurs at -78C, makes solid
D) is the most common form carbon dioxide takes as
CO2 useful for keeping materials very cold. Because solid
a substance
CO2 cools other objects and does not leave a liquid
residue, it is called dry ice. As for liquid CO2, it is E) has a wide range of properties that make it suitable
obtained by putting carbon dioxide gas under pressure. for various uses
When liquid CO2 evaporates, it absorbs large quantities of
heat, cooling as low as -57C. Because of this property, it
58. One can conclude from the passage that
is often used as a refrigerant. If the compressed gas from
carbondioxide ----..
the evaporating CO2 liquid is allowed to expand through a
valve, the rapidly cooled vapour forms solid carbon
dioxide snow. This CO2 snow is compacted into blocks A) changes into a series of states only when it is
and is the source of dry ice. subjected to unusual levels of pressure

B) is the most common substance used in the


production of dry ice
55. It is understood from the passage that liquid
carbon dioxide ----.. C) requires very high pressure in order to change
from one state to another

A) does not exist at normal pressures, but becomes D) is a substance which, similar to water, can be
available by pressurizing CO2 gas found in three different states

B) changes into the gaseous state at temperatures E) must be stored and used at very low temperatures
ranging from -57C to -78C

C) does not have as many different uses as the other


states of CO2 have

D) is used widely in obtaining solid carbon dioxide


under high pressure

E) never stays stable but soon changes into the solid


state

56. It is clear from the passage that solid


carbondioxide snow ----..

A) is the only form that solid CO2 usually takes under


normal pressures

B) is very effective in refrigeration if it is used in large


quantities

C) has almost the same properties as dry ice although


it leaves liquid residue on objects, making it a poor
regrigerant

D) is formed when the compressed gas obtained from


the evaporation of liquid CO2 expands and, hence,
rapidly cools

E) turns into liquid CO2 when it absorbs heat and,


consequently, melts
61. As one learns from the passage, the reason whya
59. - 62.sorular aadaki paraya gre
wildfire is followed by vegetation growth is that -
cevaplaynz. ---..

A) the minerals in the soil, especially potassium,


Wildfires are an important environmental hazard inmany phosphorus, and calcium, are preserved perfectly
geographical areas. Those areas most prone towildfires
have wet seasons followed by dry seasons.Vegetation that B) soil erosion takes a long time and, therefore, plants
grows and accumulates during thewet season dries out have time to grow up
enough during the dry seasonto burn easily. When
lightning hits the ground, itignites the dry organic C) plant seeds in the soil are unaffected by the fire
material, and a fire spreadsthrough the area. Actually fires and begin to germinate in the wet season
have several effectson the environment. First, burning
D) the ashes of the burnt organic matter contain many
frees the mineralsthat are locked in organic matter. The minerals indispensable for plants
ashesremaining after a fire are rich in potassium,
phosphorus, calcium, and other minerals essential forplant E) the area where the fires have taken place becomes
growth. Thus, vegetation flourishes following afire. suitable for shade-intolerant plants
Second, fire removes plant cover and exposesthe soil,
which stimulates the germination of seedsrequiring bare
62. One understands from the passage that, while
soil, and encourages the growth ofshade-intolerant plants.
thewet season is favourable for vegetation, ----..
Third, fire can causeincreased soil erosion because it
removes plantcover, leaving the soil more vulnerable to
wind andwater. A) the dry season ushers in the danger of fire

B) it causes a great deal of soil erosion

59. According to the passage, soil erosion ----.. C) it leads to the depletion of minerals in the soil

D) the dry season stimulates the germination of seeds


A) is undoubtedly the most adverse effect that
wildfires have on areas where plant cover is thick E) the growth of plants largely depends on the
due to heavy vegetation condition of the soil

B) can happen in an area which, due to wildfires, has


lost its plant cover, whereby the soil has become 63. - 67.sorularda, karlkl konumann bo
exposed to wind and water braklan ksmn tamamlayabilecek ifadeyi
bulunuz.
C) can be prevented in areas prone to wildfires just as
strict measures are taken to maintain adequate
plant cover

D) extensively undermines environmental 63. Dr Clark :- Let me stress that the most trouble
sustainability because it not only removes plant some problem of peritoneal dialysis is the risk of
cover, but also prevents plant growth infection.
Student :- We can use antibiotics to clear up the
E) is mainly caused by wind and water, which destroy infection.
the plant cover of an area and turn the area into an Dr Clark :- ----
arid land Student :- So its inevitable that in peritoneal
dialysis, complications can occur any time..

60. It is clear from the passage that the areas


withvegetation, where a wet season is followed A) Generally, peritoneal dialysis is not performed in
by adry season, ----.. people who have abdominal wall infections.

B) Yet thats not all. Other problems are also


A) are most suitable for the germination of plant associated with this type of dialysis.
seeds
C) In peritoneal dialysis, a catheter is inserted through
B) usually have a vast range of environmental a small incision in the abdominal wall into the
diversity peritoneal space.

C) maintain their plant cover and, therefore, are much D) Moreover, inflammation of the kidneys can also be
prone to soil erosion caused by an infection.

D) are usually rich in various minerals that are E) In fact, when the kidneys fail, waste products and
essential for plant growth excess water can be removed from the blood by
haemodialysis.
E) are most vulnerable to wildfires
64. Susan :- Hi, John! How are things going with you 66. Ruth:- What do you think of global warming and
and your family? humans alleged role in it?
John :- Not very well. Ive just lost my job and Larry:- Global warming is a problem that will
now all the financial responsibility is on my wife. largely have to be solved through energy
We have bills that are due and I feel terrible conservation, but ----
about it. Ruth:- What do you really mean?
Susan :- Im sorry to hear that, but surely your Larry:- I mean its environmental hypocrisy!
son is old enough to work. Ruth:- Hypocrisy?
John :- Actually, he has a good job, but he says Larry:- Yes. These people preach clean energy,
he doesnt get paid well and it is hard for him to simple living, and use of mass transportation
help out. while doing none of these things themselves..
Susan :- ----.

A) people who care about children, born or unborn,


A) Your son will always remember how you helped should be aware of the lurking environmental
save and redirect him during this difficult time. dangers.

B) You should withdraw your financial support from B) industrial agriculture has lowered the nutritional
him. value of staple vegetable crops far below what it
used to be.
C) Parents are sometimes fearful of the unknown and
different. C) these high-profile environmentalists who talk and
talk and do little else are worse than useless.
D) But all the family members should work together in
hard times. D) people must take scientific facts into account when
making important life choices.
E) So your expenses are piling up. Tell your wife to cut
down on her expenditures. E) there may be no solution to the coming nightmare
of world overpopulation.

65. John:- Hello! Im going to visit Buenos Aires.


Whats the best way to get around the city for
sightseeing?
Travel Agent:- The subway is a useful means of
public transport to help you get about.
John:- ----
Travel Agent:- Well, it has a good coverage of
most of the key tourist attractions, but you may
have to change stations or lines..

A) Can I travel to any part of the city by it?

B) From a central station to the museums downtown?

C) I have heard that the shuttle buses are usually very


crowded.

D) Is it cheaper to go by subway than by bus?

E) Do you think it is safe to travel late at night?


67. Okan: There are now almost a dozen studies
showing that mind power can be increased over
time by doing simple exercises.
Berkay: Really? Thats very interesting. What
kind of exercises are you talking about?
Okan: Things like sitting up straight instead of
lying down and brushing your teeth with your
less dominant hand.
Berkay: ----.

A) So, what youre saying is that we should keep


doing the same things over and over again.

B) Scientists always come up with fresh claims about


fitness and exercising for a healthy life.

C) Ive never realized such simple exercises could


contribute to the development of our brain.

D) Thats not new to me. Ive been trying to improve


my memorization skill for a very long time.

E) Studies on the brain tend to be quite popular


nowadays, but this one doesnt sound very
appealing.
68. - 71.sorularda, verilen cmleye anlamca en
yakn cmleyi bulunuz.

68. Before starting to write a test essay, read the


instructions carefully and be certain that you
know just what is wanted..

A) Unless you read the instructions carefully and be


certain about what is wanted you cannot start
writing a test essay.

B) However carefully you read the instructions and be


certain on what is wanted, these arent enough
before starting to write a test essay.

C) Should you read instructions carefully, the only


thing youll do is to know what is wanted from you
before writing a test essay.

D) Its widely believed that both reading the


instructions carefully and being certain about what
is wanted in writing a test essay is very important.

E) Youd better read the instructions carefully and be


certain that what is wanted to start writing a test
essay.

69. Over the past 40 million years, more than 600


species of elephants have roamed the earth..

A) Over the past 40 million years, it is estimated that


600 elephant species have become extinct.

B) Although there were more than 600 species


elephants on earth, most of them died out.

C) More than 600 kinds of elephants have become


extinct for more than 40 million years.

D) The number of elephant species has increased up


to 600 over the last 40 million years.

E) For more than 40 million years, there have been


over 600 sorts of elephants on earth.
70. The government should encourage more 71. Keen eyesight, an acute sense of smell and an
investment in rural areas so as to prevent excellent sense of hearing enable predators to
migration into big cities.. track down their prey..

A) Unless the authorities invest more in urban areas, A) Predators can easily catch their prey thanks to their
they wont be able to stop the population getting highly developed sense of smell, hearing and
scarce. excellent eyesight.

B) Much more invesments should have been done by B) In order to hunt their prey, all a predator needs is a
the authority to avoid people move from their strong sense of smell, hearing and eyesight, but
hometown to get better living conditions. hearing is the most important of all.

C) There has been so much migration into big cities C) Sense of smell, hearing and eyesight can become
that it is almost impossible to avoid even by very crucial for predators when they have difficulty
imposing some investments in rural regions. in finding their prey.

D) In order to stop migration to big cities, the D) Accurate eyesight and hearing abilities may help
government had better promote additional the predators to follow their prey, but the
investments in rural areas. importance of smell cannot be neglected.

E) To provide a high living standard for people living E) For predators, there are many senses that are
in rural areas, there is still more to do by the required to catch their prey easily such as sense of
government. smell, hearing and exceptional eyesight.

72. - 75.sorularda, bo braklan yere, parada


anlam btnln salamak iin
getirilebilecek cmleyi bulunuz.

72. Most people are familiar with one important


function of the kidneys to rid the body of waste
materials that are either ingested or produced by
metabolism. A second function that is especially
critical is to control the volume and composition
of the body fluids. For water and virtually all
electrolytes in the body, the balance between
intake and output is maintained in large part by
the kidneys. ----..

A) Each kidney in the human body is made up of


about 1 million nephrons, each capable of forming
urine

B) This regulatory function of the kidneys maintains


the stable environment of the cells necessary for
them to perform their various activities

C) Blood flow to the two kidneys is normally 21 per


cent of the cardiac output, or about 1,200ml/min

D) The two kidneys lie on the posterior wall of the


abdomen, outside the peritoneal cavity

E) These include urea, uric acid, creatinine and


metabolites of various hormones
73. Although difficulties with names are nothing
new, the nature of name-giving changed with the
introduction of computer technology. ----. In the
old days, a letter from overseas addressed to
theU.S. or the U.S.A. or even the E.U.
would stand a chance of being delivered, but an
e-mail for the corresponding geographic domain
must have the exact designation US; no
variation is tolerated. .

A) For example, the ticker symbols that identify


securities on the New York Stock Exchange can be
no more than three characters long

B) Broadcast radio stations in the US have call signs of


either three or four letters, and the first letter is
always either K or W

C) The names can get so long and intricate that only a


computer can identify them

D) It is not just the names that are scarce; we are


even running out of numbers

E) Place names and abbreviations for them offer a


good example of how names have changed

74. Culture is taken seriously in France: writers,


intellectuals, artists and fashion designers are
highly respected. ----. Other activities in various
areas from the music industry to the French
language itself are affected by the same
protectionist attitudes..

A) French social life, except between close friends,


has always been marked by formality

B) France is a country where tradition and progress


are found side by side

C) Although they are American in inspiration, they are


French in what they sell, with wonderful displays of
cheeses and a huge range of fresh vegetables and
fruit

D) Unemployment has led to growing racism against


Arab immigrants, many of whom are from former
French colonies in North Africa

E) In this regard, for example, the French are working


hard to defend their film activities against
pressures from Hollywood
75. The green leaves of a plant produce nutrients
using photosynthesis, a process that occurs in
chloroplasts located within the leaf cells. Water
and the sugar produced by photosynthesis are
distributed through the vascular bundles to
nourish cells. The vascular bundles can often be
clearly seen as the veins on the underside of a
leaf. The leaves draw carbon dioxide from the air
through slit-shaped openings called stomata.
These are usually found on the undersides of
leaves. ----.

A) The stem is an integral part of the plant and


supports the leaves, flowers and fruit, as well as
transporting water and nutrients.

B) Typically, stomata open in the daytime to release


excess water and the oxygen produced during
photosynthesis.

C) The basic components of a seed plant are the


roots, leaves, stems or shoots, and at certain times
of the year, flowers and fruit.

D) Green plants are an essential part of any garden


and provide shelter for numerous insects.

E) However, in an individual plant cell, there are


hundreds of lens-shaped sub-cellular organelles.

76. - 80.sorularda, cmleler srasyla


okunduunda parann anlam btnln
bozan cmleyi bulunuz.

76. (I) Mouth ulcers are painful sores on the inside of


the mouth or tongue. (II) The pain comes about
because the mucus membrane that forms the
delicate lining inside the mouth has been ripped
open, exposing nerves. (III) The most common
type is caused by accidental injury. (IV) Other
types can be caused by stress or fungal and viral
infection. (V) White areas can appear anywhere
in the mouth and often are simply food debris
that can be wiped away..

A) I

B) II

C) III

D) IV

E) V
77. (I) Christine Lagarde, Frances first female 79. (I) Any given place may have several different
minister for finance and the economy, says it is ecosystems that vary in size and complexity. (II)
time for French people to 'roll up their sleeves' Humans benefit from these smoothly-functioning
and stop thinking about holidays. (II) The former ecosystems in many ways. (III) A tropical island,
international lawyer, impressed by the work for example, may have a rain forest ecosystem
ethic she witnessed during her time in the US, that covers hundreds of square miles, a swamp
wants to instill the same spirit in her countrymen ecosystem along the coast, and an underwater
and women. (III) The French governments plans coral reef ecosystem. (IV) No matter how the size
to reorganize the retail industry have brought or complexity of an ecosystem is characterized,
fears that greater competition will threaten all ecosystems exhibit a constant exchange of
livelihoods. (IV) Her approach is calm and matter and energy between the living and non-
rational, bearing little resemblance to the living elements. (V) This constant exchange
harsher style of the French president, Nicolas between these elements makes ecosystems
Sarkozy. (V) Yet Christine Lagarde shares the highly interconnected..
presidents convictions when it comes to the
French people and the world of work..
A) I

A) I B) II

B) II C) III

C) III D) IV

D) IV E) V

E) V

78. (I) The mountain kingdom of Bhutan has decided


to build its national policies on improving its
gross national product. (II) The 'Easterlin
Paradox', which states that a nation's happiness
actually declines in relation to its economic
growth, was once thought to be limited to rich
Western countries. (III) However, researchers
have recently shown that it also applies to
developing countries. (IV) For example, even as
China experienced extraordinary economic
growth between 1990 and 2000, the percentage
of Chinese who described themselves as very
happy fell from 28per cent in 1990 to 12 per cent
in 2000. (V) Experts attribute this phenomenon
to the fact that peoples desires and
expectations change along with their material
fortunes..

A) I

B) II

C) III

D) IV

E) V
80. (I) The collapse of the economies of southeast
Asia in the early years of the decade was later
followed by that of Korea, the classic tiger
economy. (II) What happens next in a region that
is now strewn with the wrecks of so-called
economic miracles? (III) This leaves Europe and
United States as significant mass markets. (IV)
The assumption that recession will spread in the
region owing to capital flight and belt-tightening
is most plausible. (V) This will deepen as foreign
investors increasingly reduce their profile in
southeast Asia..

A) I

B) II

C) III

D) IV

E) V
SORU CEVAP SORU CEVAP

1 C 41 C

2 A 42 E

3 E 43 D

4 C 44 A

5 A 45 C

6 E 46 E

7 D 47 A

8 E 48 D

9 E 49 C

10 E 50 E

11 E 51 A

12 B 52 B

13 E 53 D

14 A 54 A

15 E 55 A

16 A 56 D

17 E 57 B

18 A 58 D

19 C 59 B

20 A 60 E

21 D 61 D

22 A 62 A

23 D 63 B

24 A 64 D

25 E 65 A

26 B 66 C

27 C 67 C

28 B 68 E

29 B 69 E

30 E 70 D

31 C 71 A

32 C 72 B

33 A 73 E

34 C 74 E

35 A 75 B

36 A 76 E

37 D 77 C

38 E 78 A

39 B 79 B

40 A 80 D
18
5. Historically, the development of capitalism has -
1. - 16.sorularda, cmlede bo braklan yerlere
--- several phases, following the period of feudal
uygun den szck ya da ifadeyi bulunuz. organization of society..

A) taken after
1. A small percentage of people have new or
recurring ---- of pain that feel like gall bladder B) called in
attacks even though they have no gall bladder..
C) brought forward

A) episodes D) gone through

B) removals E) turned down

C) deficits

D) responses

E) trials

2. About 60 million people had to leave Europe


before World War II, which caused an ----
migration to the US..

A) indifferent

B) explanatory

C) intuitive

D) ambiguous

E) immense

3. Patients diagnosed with identical cancers and


given similar therapies of radiation and
chemotherapy often ---- very differently..

A) correspond

B) approach

C) identify

D) respond

E) emerge

4. As we come to the last trading day of the third


quarter, it's hard to imagine things could get
worse- but they can and most ---- will..

A) voluntarily

B) ultimately

C) probably

D) profitably

E) generously
6. They had to call in troops to ---- the forest fire
which was spreading rapidly..

A) get out

B) put out

C) hold up

D) break down

E) hand out

7. In some ways, we know little more about


the planets than ---- the ancients who
worshipped them..

A) had done

B) have done

C) do

D) would do

E) did

8. The first evidence that there are genetic factors


in smoking ---- in the 1950s from studies which -
--- that identical twins tended to be more similar
in their choice to smoke or not than did fraternal
twins..

A) could appear / would indicate

B) has appeared / had indicated

C) had appeared / have indicated

D) would appear / were indicating

E) appeared / indicated

9. We know nothing at all about the companys new


chairman ---- that hes just had his fiftieth
birthday..

A) since

B) instead

C) in case

D) on condition

E) except
10. You will have to study for years...... you intend to
read the ancient Greek classics in the original..

A) if

B) unless

C) so

D) yet

E) nor
11. In their discussions concerning the proposed 15. It's human nature to get used to changes, so
defence strategies, NATO diplomats found they most of us have a tendency ---- how rapidly the
could not agree ---- what the alliance was ----, world has changed, and keeps ----..
what weapons it would threaten to use and in
what circumstances..
A) forgetting / having changed

A) on / for B) forget / changed

B) about / into C) having forgotten / change

C) with / over D) to forget / changing

D) by / about E) forgotten / to change

E) before / through

12. Infectious diseases are those which are caused -


--- an invasion of the body ---- organisms from
outside..

A) through / with

B) by / by

C) over / through

D) in / without

E) upon / within

13. The biggest accounting issue raised ----


the energy company is ---- accounting
standards..

A) by / over

B) to / to

C) for / under

D) through / within

E) from / into

14. Many people ---- know --- care about what the
new laws really mean..

A) whether / or

B) neither / nor

C) both / or

D) less / that

E) as / as
16. Items ---- display in this museum were obtained -
--- mosques, Turkish baths, fountains and
caravansarays dating back to the Ottoman
period..

A) on / from

B) in / by

C) for / out of

D) within / of

E) with / at
21. V.
17. - 21.sorularda, aadaki parada
numaralanm yerlere uygun den szck ya
da ifadeyi bulunuz. A) with

B) from

Ultraviolet (UV) light from the sunis classified into three C) into
types: ultraviolet A (UVA), ultraviolet B (UVB),
andultraviolet C (UVC), depending on its wavelength. UV D) without
light in small amounts is(I) ----, as it helps the body
produce Vitamin D. (II) ----, larger amounts ofUV light E) beside
damage DNA (the bodys genetic material) and alter the
amounts andkinds of chemicals (III) ---- the skin cells
make. UV light also (IV) ----folic acid, sometimes resulting
in deficiency of that essential vitamin in fair-
skinnedindividuals. Although UVA penetrates deeper (V) -
--- the skin, UVB isresponsible for at least three quarters of
the damaging effects of UV light,including tanning,
burning, premature skin aging, wrinkling, and skin cancer.

17. I.

A) neutral

B) beneficial

C) indeterminate

D) minimal

E) economical

18. II.

A) Additionally

B) Similarly

C) However

D) Indeed

E) Otherwise

19. III.

A) that

B) what

C) where

D) whatever

E) wherever

20. IV.

A) should break down

B) must break down

C) broke down

D) might have broken down

E) may break down


26. V.
22. - 26.sorularda, aadaki parada
numaralanm yerlere uygun den szck ya
da ifadeyi bulunuz. A) acquainted

B) sought

In the early republic, different groups of Romans (I) ---- for C) compelled
power. One group was the patricians, the
aristocraticlandowners who held most of the power. The D) acclimatize
other important group was theplebeians, the common
farmers, artisans, and merchants who made up the E) gained
majorityof the population. The patricians (II)---- their
power and social status. They claimed that their ancestry
gavethem the authority to make laws for Rome and its 27. - 36.sorularda, verilen cmleyi uygun
people. The plebeians werecitizens of Rome with the right ekilde tamamlayan ifadeyi bulunuz.
(III)----. They, (IV) ----, werebarred by law from holding
most important government positions. In time, theSenate
allowed them to form their own assembly and elect
representatives calledtribunes. Tribunes protected the 27. ---- so that future encounters with the same
pathogen are dealt with swiftly..
rights of the plebeians from unfair acts ofpatrician
officials. Eventually, plebeian pressure on the patricians
(V) ---- them additional politicalpower. A) Effective vaccines can be prepared in a number of
ways

B) Through immunization the body launches an


22. I. immune response, and develops memory cells

C) Most vaccines consist of the entire pathogen or of a


A) struggled protein from the pathogen

B) avoided D) Tetanus and botulism vaccines are made from


toxins secreted by the respective pathogens
C) rejected
E) Most persons contract measles or chickenpox only
D) abused once

E) pursued
28. ---- as they are today..

23. II.
A) Satellites are providing clear photographs

A) referred B) The fluctuating magnetic field lies deep in the


centre of Earth
B) inherited
C) Several other bodies in the solar system generate
C) handed their own magnetic fields

D) overwhelmed D) Earth's magnetic poles have not always been


oriented
E) abstained
E) Many intriguing explanations are being put forward
24. III.
29. By the time he boarded the plane, ----..

A) being voted
A) he had probably spent a lot more time on in-flight
B) voting games

C) vote B) the plane was flying at an altitude of 10 thousand


feet
D) to vote
C) he had already got to know eight important
E) voted business people

D) the discount business class ticket was still very


25. IV. expensive

E) changing planes in a busy airport was a headache


A) thus
for him
B) similarly

C) however

D) for example

E) moreover
30. Unless all countries in the developed world 34. Warsaw is rich in museums and historic sights -
reduce their fossil fuel consumption drastically, - ---..
---..

A) where the streets are less crowded than most


A) problems relating to global warming are far too European cities
numerous
B) because the inhabitants of the city are proud of
B) this was not enough to improve fuel efficiency their history

C) the price of petrol rises steadily C) although most of them were damaged during World
War II
D) there have been major innovations in personal
transportation vehicles D) when it finally became the capital city of Poland in
1596
E) the negative consequences of global warming will
increase E) because Warsaw is visited by thousands of tourists
every year

31. Since he ignores certain fundamental facts about


the past century, ----.. 35. With satellite digital radio, good reception is
guaranteed, ----..

A) there will be many different kinds of authoritarian


leaders A) before your receiver has been installed by the
mechanic
B) the conclusions had been unreliable
B) if you pay too much money for the digital radio
C) good and evil alike have to be judged
C) after you sign up for a monthly payment
D) he presents a distorted picture of reality
D) as long as you point your receiver at the satellite
E) someone should, nevertheless, still be blamed for
the unjustifiable deaths E) unless you purchase a good satellite digital radio

32. Charlie Chaplin, ----, always wore a black bowler 36. Many linguists advise students to master a
hat and carried a cane.. vocabulary of two or three thousand words in
Chinese ----..

A) as Limelight is the best known of his films


A) unless one had developed an adequate
B) that he acted and directed conversational ability for simple situations

C) in which the films were both tragic and comic B) when we hear words and expressions from a native
speaker
D) which was right at the beginning of cinema history
C) that one ought to try to write sentences on the
E) who was a popular film star in the time of the silent basis of the grammar rules
movies
D) if a knowledge of reading and writing may have
been acquired
33. Some politicians were asked to describe the
emotions their own demise would arouse and E) before they begin the difficult task of learning the
what would happen after they died, ----.. languages symbols

A) therefore all groups gave highly favourable


opinions on the measures that were taken by the
government

B) while others were given the rather less difficult task


of answering questions about their TV viewing
habits

C) when the study showed a politician can be a


charismatic leader advocating home security
measures

D) whereas a politician usually faced an uphill battle


to win the approval of voters in an impending
election

E) even if you could imagine an unscrupulous


politician having a quiet word in the ear of an
intelligence officer
39. Kuveyt'teki en parlak gelime, Krfez'de,
37. - 42.sorularda, verilen ngilizce cmleye
zellikle Dubai'de, yaygn bir uygulama
anlamca en yakn Trke cmleyi, Trke olan serbest blgenin yaratlmasdr..
cmleye anlamca en yakn ngilizce cmleyi
bulunuz.
A) In imitation of the Gulf and of Dubai, in particular,
Kuwait now has a free zone,which is a most
promising development.
37. Yllarca, gelimi pek ok lkede balca lm
B) The creation of a free zone in Kuwait is a very
nedeni, genellikle kardiyovaskler hastalklar
hopeful sign; the practice is common in the Gulf,
olarak bilinen kalp ve damar hastalklar
especially in Dubai.
olmutur..
C) The brightest development in Kuwait is the creation
A) Over the years, it has generally been known that of a free zone, a practice that is common in the
among the major causes of death in most Gulf, especially in Dubai.
developed countries is cardiovascular disease,
which refers to diseases of the heart and blood D) The creation of a free zone in Kuwait on the lines of
vessels. those in the Gulf and especially in Dubai is
regarded as a very hopeful sign.
B) Generally, diseases of the heart and blood vessels,
which for many years were known as E) The most striking development in Kuwait is the
cardiovascular disease, have been among the main creation of a free zone, as found commonly in the
causes of death in various developed countries. Gulf and especially in Dubai.

C) Cardiovascular disease, which as a whole includes 40. Although Plato loves Homer and regularly cites
the major diseases of the heart and blood vessels, from his epics, he insists on the censorship of
has over the years been the usual cause of death in those passages that represent morally
a number of developed countries. controversial behaviour..
D) For years, the major cause of death in many
developed countries has been diseases of the heart A) Platon, Homerosu sevmesine ve onun
and blood vessels, generally known as destanlarndan dzenli olarak alntlar yapmasna
cardiovascular disease. ramen, ahlk bakmdan tartmal davranlar
yanstan blmlerin sansr edilmesinde srar eder.
E) Numerous diseases of the heart and blood vessels,
which are as a whole called cardiovascular disease, B) Homerosu seven Platon, onun destanlarna srekli
have for years been the most common cause of atfta bulunsa da ahlk anlamda zararl davranlar
death in developed countries. tasvir eden blmlerin sansr edilmesi iin ok
aba gstermitir.
38. Yiyeceklerdeki iyot miktar deikendir ve
C) Platon, bir yandan Homerosu sevmi, bir yandan
genellikle, bitkilerin yetitii veya hayvanlarn
da onun destanlarndaki ahlka aykr davranlar
zerinde otlad topraktaki miktar yanstr. .
ieren blmlere iaret ederek, bunlarn sansr
edilmesi iin srekli talepte bulunmutur
A) The amount of iodine contained in foods varies
significantly and depends on the amount in the soil, D) Homerosu sevmi olmasna ramen, Platon, onun
needed for the growth of plants or the grazing of destanlarndaki ahlk bakmdan kabul edilemez
animals. blmleri eletirerek, bunlarn sansr edilmesi
gerektiini vurgular
B) Foods contain various amounts of iodine, and
these are usually related to the amount in the soil E) Platon, Homerosu sevmi olsa bile, onun
used for plant cultivation and animal grazing. destanlarn srekli eletirerek, ahlka aykr
davranlar ne karan blmlerin tamamen
C) The amount of iodine in foods is variable and sansr edilmesinin art olduunu ileri srer.
generally reflects the amount in the soil in which
plants are grown or on which animals graze. 41. As some columnists have also pointed out, the
year 2000 was a year in which the world shifted
D) The soil in which plants are grown or on which
its balance..
animals graze usually contains the same amount of
iodine as found in various foods.
A) Baz ke yazarlarnn belirttiine gre, dnyadaki
E) The amount of iodine which is contained in the soil dengeler 2000 ylnda olduka deimitir.
used for the cultivation of plants or the grazing of
animals is, on the whole, reflected by the amount B) 2000 ylnda dnyann dengesini deitirdii, baz
of iodine in foods. ke yazarlarnn ortak grdr.

C) Baz ke yazarlar, 2000 ylnda dnyann


dengesinin deitii konusunda gr birlii
iindedir.

D) Baz ke yazarlarnn da belirttii gibi, 2000 yl


dnyann dengesini deitirdii yl oldu.

E) Baz ke yazarlarnn da vurgulad gibi, dnya


dengesinin en ok deitii yl, 2000 yldr.
42. Despite the rapid industrialization of countries
such as China and India, climate change is
neither any one countrys fault nor any one
countrys responsibility..

A) in ve Hindistann hzl sanayilemesi nedeniyle


oluan iklim deiiklii, kesinlikle baka bir lkenin
suu veya sorumluluu deildir

B) in ve Hindistan gibi lkelerin hzl


sanayilemelerine ramen, iklim deiiklii, ne
herhangi tek bir lkenin hatasdr, ne de herhangi
tek bir lkenin sorumluluudur

C) in ve Hindistan dahil, pek ok lkenin hzla


sanayilemesi sonucu ortaya kan iklim deiiklii,
herhangi bir lkenin suu olmad gibi,
sorumluluu da deildir.

D) Tek bir lkenin suu veya sorumluluu olmayan


iklim deiiklii, in ve Hindistan dahil, pek ok
lkenin hzla sanayilemesi sonucunda ortaya
kmtr.

E) in ve Hindistann yan sra, eitli lkelerin hzla


sanayilemesi, iklim deiikliine yol am olup,
tek bir lke sulu veya sorumlu deildir.
46. One can understand from the passage that
43. - 46.sorular aadaki paraya gre
fluvaccines currently on the market ----..
cevaplaynz.

A) do not provide lifetime immunity to the type A


strain of flu
A new vaccine that may offer life time protectionagainst
the flu has shown promise in human trials. It works on the B) have proven rather promising in human trials
deadly type A strain, responsible for pandemics. Current
flu vaccines work by giving immunity against two proteins, C) eliminate the need to identify the most prevalent
called haemagglutinin and neurominidase, found on the strains of flu
surface of fluviruses. However, as these proteins
continually mutate, vaccines have to be reformulated D) give immunity against the M2 protein
every year to keep on working. The new vaccine, known
E) are subject to future trials on other strains
as ACAM-FLU-A, gets around this problem by homing inon
a protein called M2, found on all type A strains, that does
not mutate so readily. The vaccine could bequickly
produced in response to a flu out break. The doses can be
mass produced and used at any time, because there is no
need to identify the most prevalent strains. In theory, a
single injection could offer lifetime protection. Whether
this will work inpractice is a matter for future trials.

43. One can understand from the passage that the


new vaccine ----..

A) has not yet been tested on humans

B) focuses on a protein that is common to A strains

C) helps to identify the most prevalent strains

D) does not mutate as readily as predicted

E) cannot be produced in large quantities

44. According to the passage, the two proteins


haemagglutinin and neurominidase ----..

A) are the most prevalent strains of the disease

B) home in on a protein called M2 that is present in all


type A flu strains

C) cannot mutate as rapidly as other proteins do

D) are the active ingredients in the newly-developed


vaccine

E) require the development of new vaccines as they


incessantly mutate

45. It can be understood from the passage that the


protein M2 ----..

A) mutates only when haemagglutinin and


neurominidase mutate

B) is common to all types and strains of flu

C) does not mutate at the same rate as do other


proteins found on flu viruses

D) could be produced quickly in response to a flu


epidemic

E) can, in theory, be destroyed through forced


mutation
49. It is clear from the passage that Caxtons
47. - 50.sorular aadaki paraya gre
interest in the printing press ----..
cevaplaynz.

A) was originally aroused when he was visiting


Germany
The printing press was first introduced into England by
William Caxton in the last quarter of the fifteenth century. B) had always been strong ever since his early years
During his earlier travels in Europe, he had seen the newly
invented system of printing microwaveable type in C) was essentially related to his desire to make a lot
Germany. He set up his own pressin London in 1476. This of profit from book production
initiated a major change in English literature. Now books
did not have to be laboriously copied by hand. Soon, they D) was the main motive for his journey to Germany
would become relatively cheap. With books easily
E) became stronger as more and more people were
obtainable, more people could learn to read, anymore learning to read
books would be produced. The experience of literature
would soon shift from the breathless group of listeners
gathered in a hall or around a fire, hearing an old tale told
once more, to the solitary individual, alone with the
thoughts and feelings of another person speaking from
the printed page.

47. It is asserted in the passage that the


introduction of the printing press into England -
---..

A) made book production easy but caused a major


increase in their prices

B) caused the complete disappearance of oral


literature

C) was too late to have any constructive effect on


social and literary life

D) aroused a great deal of objection from copyists and


booksellers

E) had a very significant impact on peoples


relationship with literature

48. One understands from the passage that, with the


printing press, ----..

A) Caxton soon caught the attention of the general


public and was much respected

B) Caxton was able to produce a lot of books and


make a huge profit from their sale

C) London came to be a major centre of book


production in the fifteenth century

D) the number of books produced increased greatly


and to the benefit of the reading public

E) booksellers in London began to compete with each


other fiercely
50. It is pointed out in the passage that, after the
introduction of the printing press, a readers
easyaccess to books ----..

A) contributed enormously to the development of


different types of literature in fifteenth-century
England

B) was hindered by the fact that books were hardly


affordable

C) was what the London book publishers primarily


cared about

D) made storytelling obsolete and, hence, angered the


storytellers of the time

E) enabled the individual to share the thoughts and


experiences of another person through the printed
page
53. As is emphasized in the passage, there can be no
51. - 54.sorular aadaki paraya gre
limit to ----..
cevaplaynz.

A) the political laws that constitute the totality of a


countrys legal system
We commonly speak of both law and laws, and the
seterms, though not used with precision, point to two B) the number of the laws in a country that are
different aspects under which legal science may be distinct from each other
approached. The laws of a country are thought of as
separate, distinct, individual rules; the law of a country, C) an analysis of a countrys law, however extensive
however much we may analyse it into separate rules, is the analysis may be
something more than the mere sum of such rules. It is,
D) what meanings and interpretations laymen can
rather, a whole, a system which orders our conduct and in
make of the laws of a country
which the separate rules have their place and their
relation to each other and to the whole. Moreover, it is E) the variety of laws that can be included within the
never completely exhausted by any analysis, however far law of a country
the analysis may be pushed, and however much the
analysis maybe necessary to our understanding of the
whole.Thus, each rule which we call a law is part of the
whole we call the law. Lawyers generally speak oflaw;
laymen more often of laws.

51. It is pointed out in the passage that, in legal


science, the term law ----..

A) is very ambiguous and cannot be clearly defined


due to the variety of meanings it has

B) signifies a whole which, as a system, embodies all


the separate rules referred to as laws

C) refers to the different aspects of a countrys legal


system and, hence, is difficult to explain

D) has always been controversial and, therefore,


cannot be interpreted accurately

E) has a limited meaning that does not deserve any


analysis or understanding

52. As regards the use of the terms law and


laws, the writer claims that ----..

A) there needs to be a common approach among the


lawyers of a country

B) lawyers and the people outside the legal


profession share the same view

C) each country has a different approach because of


its distinct legal system

D) it is not lawyers but laymen who need to be more


precise

E) there is not much clarity since both terms are


often confused
54. As can be understood from the passage, the laws
of a country, ----..

A) though varied and numerous, embody a common


goal for the countrys peace and prosperity

B) which lawyers are expected to interpret precisely,


indicate the efficiency of the legal system

C) whether understood properly or misinterpreted, are


necessary for the well-being of society

D) which can also be understood accurately by


laymen, are seldom related to each other

E) though separate and distinct from each other, are


essentially interrelated
57. It is pointed out in the passage that all
55. - 58.sorular aadaki paraya gre
organisms----..
cevaplaynz.

A) know exactly how mountain slopes provide the best


conditions for survival
Moisture is removed from humid air by mountains, which
force the air to rise. As it gains altitude, the air cools, B) usually have their habitat in places where there
clouds form, and precipitation occurs, primarilyon the isnt much exposure to sunlight
windward slopes of the mountains. As the air mass moves
down on the other side of the mountain, it is warmed, C) are affected most adversely when the climate of
thereby lessening the chance of precipitation of any their environment keeps on changing
remaining moisture. This situationexists on the west coast
D) fully adapt to the conditions of the microclimate in
of North America, where precipitation falls on the western
which they live
slopes of mountains that are close to the coast. The dry
lands on the sides of the mountains away from the E) are perfectly able to cope with the physical
prevailing wind arecalled 'rain shadows.' Generally, conditions of an arid geography
differences inelevation, in the steepness and direction of
slopes, and in exposure to sunlight and prevailing winds
may produce local variations in climate known
as'microclimates, ' which can be quite different from their
overall surroundings. For an organism, the microclimate of
its habitat is of primary importance, because that is the
climate an organism actually experiences and knows how
to cope with.

55. As pointed out in the passage, microclimates -


---..

A) can best be observed along the west coast of


America where precipitation is high

B) occur as a result of differences in the geographical


features and conditions of an area

C) show a great deal of variety in the North American


dry lands known as rain shadows

D) create a secure environment for organisms that


prefer to live in areas with adequate precipitation

E) are not affected by prevailing winds because of the


steepness and direction of mountain slopes

56. It is clear from the passage that mountains ----..

A) with steep slopes are most suitable for the


formation of rain shadows

B) do not receive any amount of precipitation on their


windward slopes

C) form an obstacle for prevailing winds to cause local


variations in climate

D) always provide a large variety of habitat for all


kinds of organisms

E) play an important role in the change of moist air


into precipitation
58. It is explained in the passage that the rain
shadows of mountains are dry because ----..

A) air mass that moves over them is warm and has


almost no moisture for precipitation

B) they differ enormously from each other in terms of


elevation and steepness

C) they are fully exposed to prevailing winds and,


therefore, have no chance of precipitation

D) their microclimatic conditions vary enormously and


are therefore not suitable for rainfall

E) they are far from any coast that would provide


them with moisture and cool air
61. According to the passage, some species ----..
59. - 62.sorular aadaki paraya gre
cevaplaynz.
A) are very sensitive to the environments with narrow
temperature requirements and, therefore, would
have their habitats elsewhere
Today scientists draw attention to some of the potential
consequences of global warming on wildlife. They point B) are so adaptable to different environments that
out that each species reacts to changes in temperature they keep on migrating from one environment to
differently. Some species will undoubtedly become another
extinct, particularly those with narrow temperature
requirements, those confined to small reserves or parks, C) prefer to have their habitats not only in places such
and those living in fragile ecosystems, whereas other as mountains and wetlands, but also in tundra,
taiga, and temperate forests
species may survive ingreatly reduced numbers and
ranges. Ecosystems considered most vulnerable to D) will be positively affected by global warming and,
species loss in the short term are polar seas, coral reefs, consequently, will increase in number and range
mountains, coastal wetlands, tundra, taiga, and temperate
forests. On the other hand, some species may beable to E) such as weeds and pests would survive only in one
migrate to new environments or adapt themselves to the kind of ecosystem even though they react to
changing conditions in their present habitats. Also, some changes in temperature
species may be unaffected by global warming, whereas
others may emerge from it as winners, with greatly 62. It is stressed in the passage that, due to global
expanded numbers and ranges. Those considered most warming, ----..
likely to prosper include weeds, pests, and disease-
carrying organisms that are already common in many
different environments. A) scientists have focused their efforts on the
conservation of rare species

B) many organisms have already changed their


habitats
59. It is clear from the passage that global warming -
---..
C) species living in temperate forests will have to
migrate to a new environment
A) has caused much decline in the number of many
rare species D) there has been a sharp increase in the variety and
number of pests
B) is particularly harmful to coastal wetlands and coral
reefs E) for some species, extinction is inevitable

C) has increased dangerously and extensively


throughout the world 63. - 67.sorularda, karlkl konumann bo
braklan ksmn tamamlayabilecek ifadeyi
D) has already forced many species to migrate to new bulunuz.
environments

E) does not have the same impact on all species


63. Brian : What is to be learned from the collapse of
60. The passage gives a brief account of ----.. the viaduct when the tornado hit it?
Harry : Perhaps we should simply admit that we
cant as yet build major structures capable of
A) the precautions that need to be taken to prevent resisting tornadoes.
the extinction of certain species Brian : ----
Harry : I suppose it is. But I believe its better to
B) the possible adverse effects that global warming be safe than sorry..
will, in the future, have on different species

C) how different ecosystems react to the increase of A) After all, it was an unusually violent tornado.
the numbers of species
B) Are you sure that we cant?
D) why certain species can always survive in many
different environments C) Arent you overlooking the fact that the anchor-bolt
systems had started to corrode?
E) the vital importance that wildlife has particularly for
fragile ecosystems D) Perhaps we can develop ways of doing so.

E) Isnt that being rather defeatist?


64. Jerome :- Did you see the documentary on TV 67. Andrew :- Are the five linked circles the symbol
about the US healthcare system? of the Olympic Games?
Mark :- ---- Roger :- Yes. Thats why everyone in the
Jerome :- Thats right, many families have gone competitions wears them.
bankrupt paying for treatment for a family Andrew :- ----
member. Roger :- They certainly do. They symbolize that
Mark :- Yes, and thats not all. Some patients Earths five continents are linked in sporting
have even been refused treatment by hospitals.. activities..

A) Yes; to tell the truth, I was hoping that it would be A) Do they have a meaning?
more thorough.
B) What do they mean?
B) Yes, but I didnt have time to watch all of it.
C) Are there always five circles?
C) No, but I wanted to watch it. Did you?
D) Why are there five circles?
D) Yes. Its getting worse and worse.
E) Why are the circles linked?
E) No; I watched a film on another channel instead.
Was it very informative?
68. - 71.sorularda, verilen cmleye anlamca en
65. Sue: Ive just read Freuds The Interpretation of yakn cmleyi bulunuz.
Dreams. It was really fascinating.
Marion: Thats a book Ive always meant to read.
What does Freud say?
Sue: ---- 68. One of them must be lying, and I suspect it's
Marion: So, I should keep a dream diary and try Emma..
and see what they mean..

A) Its probably Emma who is lying, it can\'t be one of


A) Like other psychologists, he believes the dreams the others.
themselves are actually meaningless.
B) Emma is the one who is lying; it\'s clearly not one
B) He maintains that an average person sleeps six of the others.
hours a day.
C) Clearly its Emma who is lying and the others are
C) He believes that our dreams are important and can not.
reveal what we unconsciously desire.
D) If anyone is lying, it\'s got to be Emma.
D) That dreams are simply the result of random
electrical impulses in the brain. E) I have a feeling that its Emma who is lying; one of
them certainly is.
E) He suggests that we should not waste our time
trying to interpret dreams.
69. When UNICEF was established in 1946, its main
aim was to provide help for the many children in
66. Adam: Do you agree with the scientists who need as a result of World War II..
claim that life on earth will soon come to an end?
Gail: Yes, I do. We use our resources in such an
extravagant way nowadays that it is only a A) When it was founded in 1946, UNICEF took a
matter of time before they are depleted. number of steps to improve the conditions of
Adam: ---- children that survived World War II.
Gail: Well, only if we are more careful from now
on.. B) In 1946, soon after the end of World War II, UNICEF
was set up to assist children throughout the world.

A) Do you think it is possible for us to extend the time C) Because so many children had suffered during
that we have left? World War II, UNICEF was set up in 1946 to give
them a better education.
B) Does that mean we are living our last days on
earth? D) In 1946 UNICEF was set up primarily because, as a
result of World War II, large numbers of children
C) Shouldnt we pay more attention to what were in need of help.
environmentalists say regarding this matter?
E) Following World War II, UNICEF was set up in 1946
D) The movie 2012 was about the approaching end of and has since made great efforts to help children in
the world, wasnt it? the world.

E) Are we going to be able to use the resources found


on other planets in the near future?
70. The Amazon Rainforest is famous for its breath- 73. For a small country, the regions of Portugal are
taking diversity of plants along with its rather different from one another. ----. Over the
considerable number of animal species.. last few decades, the inhabitants of these rural
regions have been forced to move to more
developed areas in search of employment. Other
A) The Amazon Rainforest is widely known for the regions, such as the Algarve, Lisbon, and Oporto,
existence of wonderful plants rather than the on the other hand, are quite modern and
incredible animal species found there. crowded..

B) It is well known that the Amazon Rainforest


contains a high quantity of plants together with an A) Population movement is a serious problem in
increasing number of animal species. certain parts of the country

C) The Amazon Rainforest is notable to many with its B) Some regions are quite modern and developed
large collection of plants and animal species that
cannot be found anywhere else. C) The rural Minho and Tras-os-Montes are the most
traditional
D) Besides having plenty of animal species, the
Amazon Rainforest is home to a great number of D) The cultural characteristics of different regions are
marvellous plants. also quite varied

E) In addition to an abundant number of animal E) Working conditions of urban populations are just as
species, the Amazon Rainforest is known for its good
stunning variety of plants.

74. Although many people do not recall their dreams


71. Thats something you should ask John about as in the morning, evidence suggests that
hes the financial expert.. nonrecallers do as much dreaming as recallers. If
you take people who claim that they have never
dreamed in their life and put them in a dream
A) John is the one you should consult as he specializes research laboratory, you will get dream recall at
in financial matters. rates comparable to other people. ----..

B) John might be able to help you as he takes on


interest in financial matters. A) In contrast, people can be taught to recognize that
they are dreaming
C) If I were you, Id get Johns opinion on it first.
B) Therefore, investigators do not yet understand why
D) John is the one to ask; he is responsible for all our people dream at all
financial transactions.
C) In that respect, psychologists have demonstrated
E) Lets ask John. Hes very experienced in such that some control of dream content is possible
matters.
D) However, modern methods of study have answered
a great many questions about dreaming
72. - 75.sorularda, bo braklan yere, parada
anlam btnln salamak iin E) So, if people say, I never dream, what they really
mean is, I cant recall my dreams
getirilebilecek cmleyi bulunuz.

72. Cancer is not just about the tumour itself. It is


becoming increasingly clear that the developing
tumour needs help from neighbouring cells in
order to become cancerous. ----. If you have only
one or the other it is not enough..

A) The cells that are going to become tumour cells


and their surrounding area collaborate to make the
tumour

B) Other studies suggest that precancerous cells may


need

C) Several diffusible molecules seem to be involved in


this process

D) Cells are totally dependent on their local


environment in order to propagate

E) Inappropriate activation of the myc gene is a vital


step in many cancers
75. Four hundred years after he was born, the 17th
century Ottoman traveller Evliya elebi is making
along-overdue international comeback. Explorer,
peace broker, tax collector, war chronicler and
mystic, Evliya spent 40 years travelling in the
Middle East. His 10-volume Seyahatname is an
epic travelogue that provides a fascinating
account of everyday life in the 17th century. ----
UNESCO decreed him Man of the Year in 2011
and the recent publication of An Ottoman
Traveller allows English-language readers to
discover his masterpiece..

A) Though previously little known outside of Turkey,


Evliya is finally going global.

B) You can still feel the joy of Evliyas journey across


Turkeys vast and unspoiled countryside.

C) This work was also appreciated by foreigners,


particularly English speakers.

D) Evliya loved eating and wrote in detail of the


regional specialties he sampled.

E) One can trace the early stages of Evliyas journey


from Istanbul to Mecca.
77. (I) Various studies have indicated that people
76. - 80.sorularda, cmleler srasyla
exhibit many behaviours in about the same
okunduunda parann anlam btnln order, even though they have experienced very
bozan cmleyi bulunuz. different environments. (II) We may speak of
learning to read as a process and of reading
abilities as the products or end results. (III)
Learning as a process is of primary importance in
76. (I) Modern technology has increased not only the deciding the level of the materials of instruction.
amount of pollution but also the potential for (IV) What an individual can learn is determined
accidents in the storing or handling of dangerous not only by his level of physical and mental
nuclear or chemical pollutants. (II) An accident maturation but also by what skills, attitudes,
with toxic chemicals is often beyond the control interests, tastes, and knowledge he has already
of many of the affected people and thus may developed. (V) If the height of an individuals
create extreme feelings of helplessness. (III) enjoyment in reading is Donald Duck, for
People tend to deal with problems produced by example, it would be a mistake to begin with
pollution either by ignoring the threat or by Lady of the Lake to advance his literary taste..
concentrating on the impact the pollution will
have on them personally. (IV) Indeed, these
accidents may occur quite randomly, as in a train A) I
derailment or a tank-car accident, and thus quite
unpredictably. (V) Furthermore, the fear of B) II
accidents may pervade the entire neighbourhood
near industries where dangerous chemicals are C) III
used or manufactured, providing long-lasting
stress for residents.. D) IV

E) V
A) I

B) II

C) III

D) IV

E) V
78. (I) Obesity, diabetes, and heart disease seem to
be the curse of the world today. (II) It is easy to
take good health for granted, but if you dont
take care of yourself you wont be able to take
care of anyone else. (III) Doctors refer to these
conditions as 'multifactorial', which simply
means that lots of different things can put us at
risk of developing them. (IV) Some, like genetics,
we cant do much about, but most of the risk
factors for these and lots of other conditions are
entirely in our own hands. (V) How we live our
lives, what we eat and drink, and how we handle
stress all play a part in how healthy we are, and
even how we will live..

A) I

B) II

C) III

D) IV

E) V

79. (I) Global warming is bringing new arrivals to


British shores. (II) Since 1980, 18 new fish
species have been caught off the coast of
Cornwall. (III) As the water warms up, they feel
the need to move northward. (IV) Since they are
cold-blooded creatures, they have to find
suitable surroundings in which to regulate their
temperature. (V) Indeed, between 1960 and 1980
no new species were reported anywhere in the
area..

A) I

B) II

C) III

D) IV

E) V
80. (I) The film Last Samurai takes place in Japan
in the mid-1870s and is based on the life of
Saigo Takamori, a samurai. (II) This samurai lived
during the period of transition in which the
country's ancient feudal culture, symbolized by
samurai warriors, was being rapidly replaced by
western ways. (III) Production of the film began
at the 1000-year-old Engyoji temple in Hirneji
near Tokyo. (IV) Throughout the film, the
producers tried to be true to the elegant
simplicity of Japanese architecture. (V) The rest
of the film was made in New Zealand, where they
built a replica of a 17th century samurai village,
and in California..

A) I

B) II

C) III

D) IV

E) V
SORU CEVAP SORU CEVAP

1 A 41 D

2 E 42 B

3 D 43 B

4 C 44 E

5 D 45 C

6 B 46 A

7 E 47 E

8 E 48 D

9 E 49 A

10 A 50 E

11 A 51 B

12 B 52 E

13 A 53 C

14 B 54 E

15 D 55 B

16 A 56 E

17 B 57 D

18 C 58 A

19 A 59 E

20 E 60 B

21 C 61 D

22 A 62 E

23 B 63 E

24 D 64 D

25 C 65 C

26 E 66 A

27 B 67 A

28 D 68 E

29 C 69 D

30 E 70 E

31 D 71 A

32 E 72 A

33 B 73 C

34 C 74 E

35 D 75 A

36 E 76 C

37 D 77 A

38 C 78 B

39 C 79 E

40 A 80 B
19
6. Every community in the world recognizes certain
1. - 16.sorularda, cmlede bo braklan yerlere
activities as crimes, and has developed its own
uygun den szck ya da ifadeyi bulunuz. way of ---- them..

A) depending on
1. To the peoples of the ancient world, the
characteristic ---- of civilization such as B) breaking into
government, literature, science, and art were
necessarily products of city life.. C) looking after

D) making for
A) remains
E) dealing with
B) influences

C) declarations 7. The latest research finding that older adults are


generally happier than younger ones ----
D) commodities superficially at odds with many studies that ----
older people are at higher risk for depression
E) manifestations and other mental health problems..

2. They certainly have a very ---- lifestyle: flying in A) seems / have found
private jets and staying in 5-star hotels around
the world.. B) seemed / will find

C) has seemed / would find


A) glamorous
D) had seemed / have been finding
B) challenging
E) could have seemed / would have found
C) defiant

D) enthusiastic 8. Physics, as it ---- at the end of the nineteenth


century ---- to as classical physics..
E) sustainable
A) has been known / might be referred
3. When the euro was ---- on 1 January 1999, it was
the first time since the Roman Empire that B) is known / has been referred
Europe had a single currency..
C) had been known / had been referred

A) ensured D) was known / is referred

B) discovered E) might have been known / was referred

C) stamped
9. Our memory for emotions is highly selective, and
D) accelerated we tend to remember how good the good times
were, ---- the memories of the bad times fade
E) launched more quickly..

4. Under existing laws, drugs must be safe ---- in a A) as if


balance between risks and benefits..
B) whenever

A) inadequate C) whereas

B) enough D) even

C) even E) likewise

D) unique
10. ---- lead was widely known to be dangerous, by
E) manner the early years of the 20thcentury, it could be
found in all manners of consumer products..

5. In his fiction William Faulkner uses a


complex literary style that seems to ---- his A) Since
intricate themes of good and evil..
B) Once

A) fit in with C) Only when

B) put up with D) Even though

C) clear out E) Given that

D) make out

E) take after
11. ---- helmets had been revived for soldiers 16. ---- the theory of plate tectonics, the crust of the
and extended to many workers, safety head Earth is made up of many dynamic plates..
coverings for athletes almost inevitably
followed..
A) Instead of

A) While B) According to

B) Before C) Along with

C) Although D) Because of

D) Once E) In the event of

E) Until

12. If there ---- no formal test of children's speaking


ability, it suggests, then languages at GCSE ----
much more popular..

A) was / became

B) had been / must have been

C) is / will become

D) has been / would have been

E) were / had to be

13. Clifford Goertz, probably ---- famous


anthropologist in the world today, has
revolutionised the way anthropology is taught on
campuses..

A) more

B) most

C) the most

D) the more

E) a

14. Research into the causes of cancer is ---- vital


importance if we are to learn how to control the
disease..

A) on

B) with

C) in

D) of

E) under

15. The high rate of inflation obviously puts a great


strain ---- the majority of working families..

A) on

B) in

C) beyond

D) through

E) by
21. V.
17. - 21.sorularda, aadaki parada
numaralanm yerlere uygun den szck ya
da ifadeyi bulunuz. A) bought

B) to be bought

Nobody (I) ---- that supermarkets are very convenient. But C) having bought
the current fashion of building enormous supermarkets on
the outskirts of towns is (II) ---- small shopkeepers out of D) to buy
business. Not being able to (III)---- the low prices the big
supermarkets are offering to their customers, small shops E) to have bought
in small towns are closing down. (IV) ---- shopping in town,
people are driving to the out-of-town supermarkets (V) ----
all the food they need for a week or two.

17. I.

A) had denied

B) can deny

C) will be denied

D) would have denied

E) is denied

18. II.

A) keeping up

B) moving on

C) putting off

D) making do with

E) catching on

19. III.

A) contribute to

B) focus on

C) compete with

D) depend on

E) regard as

20. IV.

A) Instead of

B) As well

C) Despite

D) In due course

E) Also
26. V.
22. - 26.sorularda, aadaki parada
numaralanm yerlere uygun den szck ya
da ifadeyi bulunuz. A) Therefore

B) Afterwards

Are port published by the Center for the Public Interest C) However
says Americans spend about (I) ---- their food budgets for
meals eaten outside the home. Many restaurants, cafes D) Rather
and diners have increased the size of their servings. It is
difficult to know the (II) ---- of fat and nutrients in the food E) Thus
served in restaurants. Processed foods sold in food stores
(III) ---- to include this information. The food industry
advertises a lot and not always truthfully. A number of 27. - 36.sorularda, verilen cmleyi uygun
food companies have given money to an international ekilde tamamlayan ifadeyi bulunuz.
food organization to set up an Internet Web site. Their
goal is to get children (IV) ---- more.(V) ----, critics say the
food industryis not trying hard enough to protect the
public's health. 27. Whereas extra training may not accelerate
childrens motor development, ----..

A) some of them were then given extra stimulation


22. I.
B) infants deprived of physical stimulation would have
delayed motor development
A) some
C) these are the ones who do not have the
B) half opportunity to move about
C) several D) some amount of practice is necessary for
development to occur normally
D) none
E) there had been several techniques for comparing
E) any
the effects of maturation and learning

23. II.
28. Once the wind had reached the critical threshold
of 94 miles per hour, ----..
A) prevalence
A) the anchor-bolt systems have already weakened
B) distribution
B) it took only about 30 seconds for the bridge to
C) disorder
collapse
D) treatment
C) a basic problem is that of corrosion
E) amount
D) but the order of collapse was related to the
complex and changing wind directions
24. III.
E) it is possible to protect structures against the force
of an F-1 tornado
A) are required

B) had been required 29. ---- as long as the economy was good and the US
remained strong abroad..
C) were required
A) The American people seem to have ignored
D) have required
Clintons weaknesses in character during his
presidency
E) require
B) Clinton became the second president in American
25. IV. history to be impeached

C) Clinton made history by becoming the first US


A) to exercise president to testify in front of a grand jury

B) exercised D) In the second year of his presidency, Clinton faced


persistent troubles on the domestic front
C) to be exercising
E) Clintons overall popularity among Americans
D) exercise remained high

E) exercising
30. ---- before grapes can be transformed into fine 35. Maintaining the environment isnt simply
wine. . protecting animals ----..

A) Harvesting was a very costly and tiring process A) unless they had almost become extinct

B) Farmers were expecting higher prices B) that are rare and exotic

C) Drought does much damage to grape vines C) which could have been done better by a zoo

D) The farm workers in France have protested at lower D) since many of them are indeed dangerous
wages
E) whether man is the planets most dangerous
E) There is much work to be done enemy

31. ---- since it can have an overall negative impact 36. It is difficult to be precise about how big the
on a persons life.. Internet is ----..

A) There was a feeling of inadequacy among the team A) until research had shown the spread of its use
members
B) because it isnt managed by one person or
B) AIDS may cause depression indirectly organization

C) He began to feel the work was too stressful C) if there were any doubts at all about its impact

D) She put off going to a doctor about the lump D) so that millions of people throughout the world had
used it
E) Some people are more ambitious than others
E) though a survey was to be carried out last year

32. Campaigns to promote energy conservation have


been around since the 1980s ----..
37. - 42.sorularda, verilen ngilizce cmleye
anlamca en yakn Trke cmleyi, Trke
A) so that there is still a long way to go before cmleye anlamca en yakn ngilizce cmleyi
campaigners are satisfied bulunuz.

B) because it is a problem that until now has been


largely overlooked
37. Sierra Leone'daki i sava o kadar vahiydi ki ve
C) but it is only with the advent of global warming that
brakt hasarlar o kadar ar olmutur ki normal
the issue has really gained momentum
dzene hzl bir dn imkanszdr..
D) just as the next step requires a great deal of
detailed planning A) One cannot look for a speedy return to normality in
Sierra Leone, for the civil war there was horrific and
E) even though most industrialized countries have the resulting injuries excessive.
been strongly in favour of these campaigns
B) Since the civil war in Sierra Leone was so barbaric
33. ----, she returned to Ireland to work in a hospital and the injuries that resulted so grave, a speedy
near Dublin.. return to normality is not to be expected.

C) The civil war in Sierra Leone was so barbaric, and


A) If there had been another war the injuries it left so grave, that a swift return to
normality is out of the question.
B) Soon after the war began
D) The civil war in Sierra Leone was so horrific and left
C) Unless she can find suitable work in London in its wake so many injured that are turn to
normality cannot readily be achieved.
D) As soon as she hears from you
E) A quick return to normality in Sierra Leone isout of
E) Whenever she wrote to me about her problems the question, for the civil war was a savage one
and the atrocities committed particularly injurious.

34. Nile crocodiles, with their rough brown skin, look


just like floating parts of a tree ----..

A) though the river was moving fast

B) if they have found something to eat

C) since there were trees by the river

D) until they open their mouths

E) that they are looking for food


38. Thomas Wolfe, ilk romannn 1929da 40. Although the French explorer Jacques Cartier,
yaymlanmas zerine, kendi kuann en ok who reached the Gulf of St. Lawrence in 1534, is
gelecek vaat eden yazarlarndan biri olarak generally regarded as Canadas founder, the
grld.. Vikings are believed to have reached the Atlantic
coast centuries before him..

A) When his first novel was published in 1929, Thomas


Wolfe was regarded as one of the most talented A) Fransz kif Jacques Cartier 1534te St. Lawrence
writers of his generation. Krfezine ulat iin Kanadann kurucusu olarak
gsterilir, ancak, ondan yzyllar nce Vikinglerin
B) Upon the publication, in 1929, of his first novel, Atlantik kysna ulatklar sanlmaktadr.
Thomas Wolfe was considered to be one of the
most promising writers of his generation. B) Vikingler ondan yzyllar nce Atlantik kylarna
ulam olsalar da, Kanadann kurucusunun
C) As soon as his first novel was published in 1929, 1534te St. Lawrence Krfezine ulaan Fransz
Thomas Wolfe was acclaimed as one of the best kif Jacques Cartier olduu genellikle kabul
writers of his generation. edilmektedir.

D) With his first novel, which was published in 1929, C) 1534te St. Lawrence Krfezine ulaan Fransz
Thomas Wolfe won the respect of his own kif Jacques Cartier genel olarak Kanadann
generation and the next one. kurucusu saylyorsa da, Vikinglerin ondan yzyllar
nce Atlantik kysna ulatklarna inanlmaktadr.
E) One of the most promising writers of this
generation was Thomas Wolfe, whose first novel D) 1534te Fransz kif Jacques Cartier St. Lawrence
was published in 1929. Krfezine ulatnda Kanadann kurucusu olarak
kabul edildi, ancak, Vikinglerin Cartierden yzyllar
nce Atlantik sahiline ulat bilinmektedir.
39. Balkan Yarmadas, nemli tarm veya sanayi
kaynaklarna sahip olmamasna ramen, Avrupa E) 1534te St. Lawrence Krfezine ulaan Fransz
ile Asya arasndaki kara kprsnn bir paras kif Jacques Cartier, Kanadann kurucusu olarak
olmas nedeniyle pek ok atmaya sahne kabul edilse de aslnda Vikinglerin ondan yzyllar
olmutur.. nce Atlantik kylarna ulatklar bilinmektedir.

A) Though the Balkan Peninsula has neither 41. Most infectious diseases are caused by
agricultural nor industrial resources, it is part of the microorganisms that invade the body and
land bridge between Europe and Asia and so has multiply there..
been fiercely contested.

B) The Balkan Peninsula has been the scene of much A) Pek ok bulac hastaln nedeni vcuda yaylan
fighting not on account of its agricultural and ve vcutta reyen mikroorganizmalardr.
industrial resources, but because it is a part of the
land bridge between Europe and Asia. B) Mikroorganizmalar, vcuda girerek ve vcutta
oalarak, eitli bulac hastalklarn olumasna
C) Though the Balkan Peninsula has no important yol aar.
agricultural or industrial resources, it has been the
scene of many conflicts because it is a part of the C) ou bulac hastalk, vcudu igal eden ve
land bridge between Europe and Asia. vcutta oalan mikroorganizmalar nedeniyle
oluur.
D) In considering the reasons for the conflicts in the
Balkan Peninsula, the agricultural and industrial D) Bulac hastalklarn nedenlerinin ou, vcuda
resources of this region are unimportant compared girerek oalan mikroorganizmalarla ilgilidir.
to the fact that it is the land bridge between Europe
and Asia. E) reyip vcudu ele geiren mikroorganizmalar
nedeniyle birok bulac hastalk ortaya
E) It is not so much the agricultural and industrial kmaktadr.
resources of the Balkan Peninsula that gave rise to
the fighting there, as its position as the land bridge
between Europe and Asia.
42. When we feel diffident, it is easy to imagine that
other people are laughing at us, but those
negative thoughts are often distorted or
magnified..

A) Kendimize gvenmediimiz zaman, bakalarnn


bizimle alay ettii gibi arpk ve abartl
dncelere sk sk kaplrz.

B) Bakalarnn bizimle alay ettiini sandmz


zamanlar, kendimize gvenmediimiz iin bu
dncemizi arpttmz veya abarttmz
zamanlardr.

C) Bakalarnn bizimle alay ettiini dndmzde


bu dncemizin arpk ve abartl olduunu fark

D) arpk ve abartl olsa da, bakalarnn bizimle alay


ettiine inanmak ounlukla kendimize olan
gvenimizi sarsar.

E) Kendimize gvenmediimiz zaman, bakalarnn


bizimle alay ettiini sanmak kolaydr, ancak bu
olumsuz dnceler ou kez arptlm veya
abartldr.
46. According to the passage, the mice used in the
43. - 46.sorular aadaki paraya gre
experiments ----..
cevaplaynz.

A) lacked the ability to learn new behaviour


Post-traumaticstress disorder (PTSD) is an anxiety B) could recover from the fear conditioning only
disorder that can develop after exposureto a terrifying through shock therapy
event or deal. Whilemany scientific studies focus on the
molecular mechanisms for learning andmemorization, C) displayed endurance to sounds accompanied by
scientists need to address the 'unlearning' process to electric shocks
tacklePTSD. Researchers have discovered that a receptor
for glutamate, the mostprominent neurotransmitter in the D) eventually overcame their fear of the sound with
the help of mGluR5
central nervous system, plays a key role inthe unlearning
process. They made this discovery in experiments where E) soon forget the difference in various kinds of sound
theytrained mice to fear a sound by coupling it with an
electric shock to the foot.They found that if, following this
fear conditioning, the mice are repeatedlyexposed to the
sound without the electric shock, their fear
eventuallysubsides. However, mutant mice lacking the
gene-coding for metabotropicglutamate receptor 5
(mGluR5) are unable to shake off their fear of the
nowharmless sound. The researchers believe that a
similar mechanism might beperturbed in PTSD sufferers
and that metabotropic glutamate receptor 5 (mGluR5)may
provide a potential target for new the rapeutic treatments.

43. It can be understood from the passage that


mGluR5 ----..

A) might disturb PTSD sufferers learning mechanisms

B) is essential in unlearning a previously-acquired


behaviour

C) makes mutant mice unable to overcome learned


fears

D) prevents PTSD sufferers from carrying out tasks


like memorization

E) is often activated through conditioning

44. It is clear from the passage that PTSD ----..

A) hinders learning and memorization

B) often results in partial memory loss

C) results from changes in molecular mechanisms

D) may result from a traumatic experience

E) gives rise to terrible events or ordeals

45. According to the passage, scientists can deal


with PTSD ----..

A) by attending to the unlearning process

B) if they focus more on learning and memorization

C) unless they tackle their terrifying experiences that


cause it

D) in case they address issues related to unlearning

E) through the deactivation of mGluR5


49. It is suggested in the passage that because
47. - 50.sorular aadaki paraya gre
Kierkegaard expressed his major ideas in an
cevaplaynz. unusual personal way, ----..

A) their essential importance was not recognized at


Kierkegaard was one of the most original thinkers of the first
nineteenth century in Europe. He wrote widely on
religious, philosophical, and literary themes.However, his B) he was widely appreciated and fully understood by
peculiar manner of presenting some of his leading ideas his contemporaries
initially obscured their fundamental significance. He
developed his views in strong opposition to prevailing C) he immediately became famous as a philosopher in
opinions, such as certain metaphysical claims about the nineteenth-century Europe
relation of thought to existence. He reacted against the
D) they were very influential on other philosophers,
ethical and religious theories of Kant and Hegel. Moreover, including Feuerbach and Marx
he opposed the doctrines and ideas which were being
advanced by some of his contemporaries like Feuerbach E) most thinkers in Europe sensed that he was
and Marx. His discussion of the human condition, which formulating a new philosophy
emphasizes the significance of individual choice, has
arguably been his most striking philosophical legacy,
particularly for the growth of existentialism.

47. It is clear from the passage that, in view of his


focus on the question of individual choice,
Kierkegaard can be regarded as ----..

A) the only thinker in the nineteenth century who was


concerned with metaphysical matters

B) a follower of Kant and Hegel, whose ethical and


religious ideas inspired him greatly

C) the philosopher who led the way for the


development of existentialist philosophy

D) the philosopher who explained the religious and


philosophical aspects of human existence

E) a major thinker who influenced his contemporaries,


including Feuerbach and Marx

48. It is pointed out in the passage that much of


Kierkegaards most striking writings ----..

A) restate the views and theories which prevailed and


were commonly shared in his time

B) were based on the ethical and religious theories


that had been formulated during earlier centuries

C) were widely read and discussed by his


contemporaries in Europe in the nineteenth century

D) are limited to topics related to religion and


philosophy

E) deal with the idea of individual choice


50. The point is made in the passage that
Kierkegaard ----..

A) approved of certain metaphysical arguments


concerning the relation of thought to existence

B) relied heavily on Kant and Hegel for the


development of his own philosophy

C) was a thinker, some of whose ideas were unusual


for his time

D) clearly defined what existentialism meant for a full


understanding of human existence

E) benefited a great deal from the philosophical


legacy of his predecessors
54. As claimed in the passage, by 2050, the worlds
51. - 54.sorular aadaki paraya gre
population will ----..
cevaplaynz.

A) increase by more than half


The human population continues to grow by more than 75 B) definitely reach a stable level
million people annually. Since the first Earth Day in 1970,
emission rates have remained steady at about 1.2 metric C) cease to have any effect on gas emissions
tons of carbon per person per year.Unfortunately, the
1997 Kyoto Protocol has had little measurable effect on D) have a much lower growth rate
these per-capita emissions, even in the countries that
E) decrease by 75 million people per year
have agreed to national targets. More than any other
factor, population growth drives rising carbon emissions,
and the USCensus Bureau and United Nations both project
that the global population, currently 6.6 billion, will
surpass 9 billion before 2050. The implication is that one
of the best strategies for reducing future greenhouse gas
emissions is population stabilization, as quickly as can be
achieved by noncoercivemeans.

51. It is suggested in the passage that one feasible


way of reducing greenhouse gas emissions in the
future ----..

A) may be through the control of population growth

B) is through the enlargement of the measures


included in the 1997 Kyoto Protocol

C) will be through the assignment of more


responsibility to the United Nations

D) can best be found through cooperation among all


the countries in the world

E) depends to a large extent on the revision of the


1997 Kyoto Protocol

52. One understands from the passage that the 1997


Kyoto Protocol ----..

A) was originally inspired by the celebration of Earth


Day in 1970

B) can be considered a major achievement by the


United Nations and the United States

C) does contain a set of extremely harsh sanctions on


gas emissions

D) has been most effective in the reduction of


greenhouse gas emissions

E) has been largely disregarded by the countries that


have signed it

53. It is emphasized in the passage that annual


population growth in the world ----..

A) can be reduced effectively through the


implementation of the Kyoto Protocol

B) has the effect of raising the amount of carbon


emissions

C) plays no role in the increase in greenhouse gas


emissions

D) is constantly watched by the US Census Bureau so


that it can be kept stable

E) has alarmed many countries and forced them to


take measures to prevent it
57. It is pointed out in the passage that a cattle
55. - 58.sorular aadaki paraya gre
rangeland carved out of a tropical forest ----..
cevaplaynz.

A) is certainly more suitable for ranching than other


kinds of ranges
Commercial logging, mostly for export abroad, accounts
for 21% of tropical deforestation. Mosttropical countries B) is actually far more profitable and environmentally
allow commercial logging toproceed at a much faster rate harmless than commercial logging
than is sustainable. For example, in parts of Malaysia,
current logging practices remove the forest almost twice C) is more feasible in tropical countries than anywhere
as fast as the sustainable rate. If this continues, Malaysia else in the world
willsoon experience short ages of timber and will have to
D) can be viable up to ten years and, then, is over
start importing logs. When that happens, Malaysiawill
grown with shrubby plants
have lost future revenues, both from logging and from
harvesting other forest products, from its newly vanished E) brings in more income than other kinds of
forests. Moreover, in addition to commercial logging, rangeland outside tropical forests
cattle ranching also causes deforestation. Infact,
approximately 12% of tropical rainforest destruction is
done to provide open rangeland for cattle. After the
forests are cleared, cattle can beraised on the land for six
to ten years, after which time shrubby plants take over
the range. Much of the beef raised on these ranches,
which are often owned by foreign companies, is exported
to fast-food restaurants.

55. It is made clear in the passage that tropical


deforestation ----..

A) is mainly caused by commercial logging and cattle


ranching

B) should be carried out at a sustainable rate in


countries like Malaysia

C) has not yet become an environmental concern in


developed countries

D) is unavoidable since many countries including


Malaysia encourage it

E) is now so well controlled that it has ceased to be a


problem

56. In the passage, attention is drawn to the fact


that Malaysias logging practices ----..

A) have given the country a leading position in forest


products

B) enable the country to earn enormous revenues

C) are carried out at a remarkably sustainable rate

D) provide a much higher income than cattle ranching

E) will be very damaging to its economy in the long


run
58. A point made in the passage is that tropical
cattle ranches ----..

A) in Malaysia are mostly owned by international


companies

B) provide most of the beef needed by fast-food


restaurants

C) are growing much faster than the sustainable rate


for forests

D) are a major source of income for many countries in


the West

E) produce so much beef that much of it is exported


to other countries
61. According to the passage, in the early twentieth
59. - 62.sorular aadaki paraya gre
century, physicists ----..
cevaplaynz.

A) were so divided among themselves that they were


unable to undertake any research in order to reveal
In the early part of the twentieth century, theexperiments the structure of the nucleus
carried out by Ernest Rutherford and his colleagues led to
the ideathat at the centre of an atom there is a tiny but B) were greatly inspired by the research activities
massive nucleus. At the sametime that the quantum which the English physicist James Chadwick was
theory was being developed and that scientists involved in
wereattempting to understand the structure of the atom
and its electrons,investigations into the nucleus itself had C) were so encouraged by the discovery of the
nucleons that they were able finally to understand
also begun. An important question tophysicists was
the true nature of the atom
whether the nucleus had a structure, and what that
structure mightbe. In fact, it has so far turned out that the D) were engaged in the development of the quantum
nucleus is a complicated entity,and even today, it is not theory while they were also focusing on the study
fully understood. However, by the early 1930s, amodel of of the nucleus
the nucleus had been developed that is still useful.
According to thismodel, a nucleus is considered as an E) were all seriously interested in the quantum theory
aggregate of two types of particles:protons and neutrons. as a new scientific development and solely focused
A proton is the nucleus of the simplest atom which on it
ishydrogen. The neutron, whose existence was
ascertained only in 1932 by theEnglish physicist James 62. It is clear from the passage that Chadwicks work
Chadwick, is electrically neutral as its name implies.These as regards the neutron ----..
two constituents of a nucleus, neutrons and protons, are
referred tocollectively as nucleons.
A) was duly taken into account by all the physicists in
their research on electrons

B) had a great deal of influence on Rutherford and his


59. One understands from the passage that one of
colleagues
the serious questions with which physicists in
the early twentieth century was concerned was - C) was undoubtedly a significant contribution to the
---.. study of the nucleus

D) was originally inspired by the quantum theory


A) to what extent the quantum theory might
which he knew so well
transform classical physics
E) brought him much fame not only in England but
B) whether the neutron provided energy for the
elsewhere also
nucleus

C) what structure the nucleus might have


63. - 67.sorularda, karlkl konumann bo
D) whether the nucleus of hydrogen contained the braklan ksmn tamamlayabilecek ifadeyi
nucleons bulunuz.

E) why Rutherford and his colleagues were indifferent


to the quantum theory
63. Robert :- Have you read this about the whale
60. It is clear from the passage that the presence of that swam up the river Thames?
a nucleus in an atom ----.. Dan :- ----
Robert :- No, there isnt. Theres even a very
clear photo of it as it swims past the Houses of
A) had always been a controversial issue among Parliament!
physicists until the early 1930s Dan :- Let me have a look at it..

B) was understood as a result of the experiments


undertaken by Rutherford and his colleagues A) There must be a mistake.

C) had been known for a long time before Rutherford B) That cant be true.
and his colleagues clearly defined its structure
C) Youre joking.
D) was revealed only after Chadwick discovered and
defined the neutron D) There arent any whales in the zoo!

E) is still a major question for physicists since the E) Surely, you dont expect me to believe that!
nature of the nucleons needs to be fully described
64. Paul: - Wasn't the Rio company founded to 67. Andrew: While surfing on the Internet, I found an
mine copper in Spain? Peter: - That's right In article claiming that national anthems were
1873. And the company was set up in the same being used by ancient people in primitive times.
year. 200, 000 shares were put on offer at UK Madeline: What may have caused them to do so?
10. Paul: - --- Peter: - So it seemed. But by the Andrew: It says they primarily played them to
end of the century Rio was the largest mining threaten and deter their enemies.
company in the world.. Madeline: ----.

A) How many of them were sold? A) Actually, most of them mention the geographical
properties of a country.
B) Some people were being far too optimistic, weren\'t
they? B) So, anthems written in hard times of a nation sound
like one another.
C) There\'s been a merger since then, hasn\'t there?
C) Yet, theyre more commonly used in national
D) Who was responsible for all this? festivals and sports events in our time.

E) Does the company only mine copper? D) Undoubtedly, poets who wrote them should be very
proud of themselves.

65. Mike: Ive read an interesting article saying that E) Well, I know there are anthems that dont have any
recounting an experience may enhance our lyrics and are purely instrumental.
understanding of it, which then dulls our opinion
of the incident.
Elena: That sounds interesting indeed. I cant
68. - 71.sorularda, verilen cmleye anlamca en
seem to think of any examples, though. Can you?
Mike: ---- yakn cmleyi bulunuz.
Elena: Excellent! This pretty much sums it up for
me..

68. Science does not produce a unified picture of the


A) Of course. Suppose you visualize in your mind that environment on which all can agree, instead it
youre getting a promotion. Youre improving your provides multiple views, each of which may be
chances of getting it by doing so. valid from a particular ideological angle..

B) For instance, if you avoid thinking about your last


failure, youre more likely to leave your bad A) There is not one single view of the environment
memories behind. that can be provided through science that
everyone will agree on, rather it gives different
C) Lets say you use different routes every day while perspectives, all of which are valid depending on
driving home. This will make you better at giving the ide
directions to it.
B) The environment has been described by scientists
D) If you keep dwelling on past experiences, for in many different ways rather than in just one way,
example, you cant enjoy the moment. and each of these have their own validity according
to the observers own ideology.
E) Well, describing how good a cupcake tastes could
make you enjoy it less. C) Science represents many diverse and particular
ideological angles, and from these a valid and
unified description of the environment can be
66. Martha : It has been stated that schizophrenia is produced that respects multiple views.
a major public health problem throughout the
world. D) Scientists from different ideological backgrounds
Colleague : Yes, Ive read somewhere that have come together to agree upon a unified picture
schizophrenia is more prevalent than Alzheimers of the environment on which scientists can all
disease, diabetes, or multiple sclerosis. agree upon its validity.
Martha : ----
Colleague : Well, what makes a person E) Multiple views on the environment are the result of
vulnerable to schizophrenia isnt known, but may science being unable to produce a unified
include genetic predisposition.. description upon which those from different
ideological backgrounds can agree.
A) In fact, as a serious mental disorder, it is
characterized by loss of contact with reality.

B) However, over longer periods, the prognosis of


schizophrenia varies.

C) I strongly believe that it is to do with a persons


genes.

D) Clearly, the severity and types of symptoms can


vary significantly.

E) It is true that schizophrenia is associated with


about 10 per cent risk of suicide.
69. As there was a great deal of rain in the spring we
are expecting a good fruit harvest this year..

A) There should be an abundance of fruit this year as


it rained so much in the spring.

B) Plenty of rain in the spring always produces a good


fruit crop later in the year.

C) It rained so heavily all through the spring time that


the fruit harvest will certainly be affected.

D) This year we can expect a better fruit harvest even


though we didnt have a wet spring.

E) Though it rained often this spring, the fruit trees


are yielding plenty of fruit.
70. The harnessing of the wind to generate
72. - 75.sorularda, bo braklan yere, parada
electricity dates back to 1890, but few notable
advances were made until 1970 when energy anlam btnln salamak iin
prices began to rise fast.. getirilebilecek cmleyi bulunuz.

A) In 1890 a few efforts were made to generate


electricity from wind power, but it was only in 1970 72. All contemporary societies are to varying
when energy prices rose steeply, that such a degrees male-dominated. In Britain, as in other
scheme was taken seriously Western societies, prestige is attached to
mens work while the womens place of work is
B) In I890, when there was a sudden increase in seen as the home. Mens work earns a salary or a
energy prices, efforts were made to harness the wage while womens domestic labour is unpaid. -
power of the wind for the generation of electricity ---. Part-time and low-paid jobs are filled
for the first time since 1890 overwhelmingly by women..
C) It was in 1890 that the wind was first used to
generate electricity, but it was long before 1970 A) It is not surprising that some women have
when energy prices started to rise sharply, that real campaigned to improve the status of women in
advances were made society

D) Between 1890 and 1970 various attempts were B) It is argued that the biological differences between
made to harness power of the wind as the rising men and women are not the cause of female
prices of energy made this desirable inequality

E) Impressive advances in the generation of electricity C) Role expectations between men and women
from wind power coincided with rising energy determine differences in status and power
prices in 1970, but the practice actually dates back
to 1890. D) When women enter paid employment, their
average incomes are lower than mens
71. I happened to run into him on my way to the
E) Women account for approximately 40 per cent of
library..
Britains total labour force

A) Fortunately, I met him just before I entered the


library.

B) We met and went to the library together.

C) I met him by chance as I was going to the library.

D) I was quite surprised when I met him in the library.

E) I happened to see him going into the library.


73. There are more than 20, 000 documented
shipwrecks off the coast of Britain. ----. And they
offer the scuba diver a fantastic world that is just
asking to be explored..

A) They range from majestic passenger ships to


historical war vessels

B) There are several underwater skills to be learned


before one can become a scuba diver

C) Others among them sank after being torpedoed

D) Another battleship lies on its side on the seabed

E) Moreover, wrecks can contain dangerous materials


74. ---- Their findings raise possible health concerns
for those working in the manufacture of the
materials. Carbon nanotubes are rolled-up sheets
of graphite thousands of times thinner than a
human hair. Because they are immensely strong
and are good electricity conductors, they are
poised for use in a wide range of fields, from
engineering to medicine. However, their
similarity in shape to asbestos fibres, which are
known to damage the lungs, is giving rise to
fears of their adverse effects on human health..

A) Carbon nanotubes are unlikely to pose risks to the


general public when incorporated into products.

B) In studies done on mice, inhaling nanotubes


affected the function of T-cells, a type of white
blood cell that organizes the immune system.

C) Scientists are trying to determine if the production


of carbon nanotubes has any biological after-effect.

D) Suppression by nanotubes of the immune system in


mice has been halted by administering one of the
standard anti-inflammatory drugs.

E) Inhaling carbon nanotubes can suppress the


immune system, according to scientists who have
just completed a study of this new substance.

75. In early Roman times, Belgium occupied part of


the Roman province of Belgica, named after
its native people, the Belgae. The area was
conquered by Julius Caesar in 57-50 B.C. -
---.Later, in the eighth century, it became part of
Charlemagnes empire. In the following
centuries, it was invaded and ruled by various
powers, and it was only in 1830 that the country
gained its independence..

A) Then in the fifth century A.D., it was overrun by the


Franks

B) In the sixteenth century, the country was inherited


by Emperor Charles V

C) It has about 40 miles of seacoast on the North Sea

D) Its rivers are most convenient for commercial


navigation

E) During the wars that followed the French


Revolution (1789), the country came under French
rule
76. - 80.sorularda, cmleler srasyla
okunduunda parann anlam btnln
bozan cmleyi bulunuz.

76. (I) Diabetes is the leading cause of blindness in


people aged 30 to 65 years old. (II) Blindness
occurs 20 times more frequently in diabetic
patients than others and is most often seen after
the disease has been manifest for at least 15
years. (III) Similarly, type II diabetes may occur
in children or adolescents but usually begins
after age 30. (IV) Approximately 10 to 15% of
type I diabetic patients become legally blind
(visual acuity of 20/200 or worse in the better
eye). (V) The primary cause of visual loss is
retinopathy..

A) I

B) II

C) III

D) IV

E) V

77. (I) NASA is considering using nuclear boosters to


lift spacecraft into orbit at a fraction of the cost
of today's all-chemical launchers. (II) The nuclear
assisted rocket produced far more thrust than
any of the conventional ones which meant the
time required for the launching was still further
reduced. (III) But anti-nuclear protesters claim
nuclear launchers would make accidents much
more dangerous and accuse NASA of 'playing
Russian Roulette'. (IV) NASA, however,
disregards all this and remains keen to move
away from conventional chemical rockets to
lighter, more powerful propulsion systems. (V)
'Nuclear systems give you a chance to reduce
your mass and so your overall costs to orbit' says
R. Adams, of NASA's Marshall of Space Flight
Center in Alabama..

A) I

B) II

C) III

D) IV

E) V
78. (I) Geologists are presently seeking ways 79. (I) Kazakhstan, the ninth-largest state in the
of detecting earthquakes before they strike. (II) world, sits on a vast stretch of land between
If they are successful, then people may be Communist China, Orthodox Russia, Islamic
evacuated from a danger area before the Central Asia and the religiously mixed Caucasus.
earthquake happens. (III) Geologists are also (II) This fact and certain turning points in its
carrying out experiments in earthquake control. history have transformed Kazakhstan into a
(IV) It is calculated that there are as many as multicultural environment. (III) This environment
500, 000 earthquakes in a single year. (V) If is rich in natural beauties too. (IV) There,
these experiments are successful, it may be representatives of more than 100 ethnic groups
possible, in the future, to minimize and many religions co-exist peacefully. (V) This is
the destructive force of earthquakes.. a legacy of the 1930s, when, during Stalinist
repression, Kazakhstan became a second
homeland for thousands of ethnic Germans,
A) I Poles, Koreans, Greeks and Kurds..

B) II
A) I
C) III
B) II
D) IV
C) III
E) V
D) IV

E) V

80. (I) The Central Bank has cut interest rates to a


much lower range. (II) It has also established a
fund to buy 60 billion dollars of government
bonds. (III) The currency of the country fell
briefly against the dollar on the news, but soon
bounced back. (IV) In the recent months the
volume of exports has increased dramatically.
(V) Banking authorities are watching closely to
see if this bounce-back will hold its place for a
long time..

A) I

B) II

C) III

D) IV

E) V
SORU CEVAP SORU CEVAP

1 E 41 C

2 A 42 E

3 E 43 B

4 B 44 D

5 A 45 A

6 E 46 D

7 A 47 C

8 D 48 E

9 C 49 A

10 D 50 C

11 D 51 A

12 C 52 E

13 C 53 B

14 D 54 A

15 A 55 A

16 B 56 E

17 B 57 D

18 C 58 B

19 C 59 C

20 A 60 B

21 D 61 D

22 B 62 C

23 E 63 A

24 A 64 B

25 D 65 E

26 C 66 C

27 D 67 C

28 B 68 A

29 E 69 A

30 E 70 E

31 B 71 C

32 C 72 D

33 B 73 A

34 D 74 E

35 B 75 A

36 B 76 C

37 C 77 B

38 B 78 D

39 C 79 C

40 C 80 D
20
6. Its a delightful novel, and the reader soon gets -
1. - 16.sorularda, cmlede bo braklan yerlere
--- in the lives of the characters as the story
uygun den szck ya da ifadeyi bulunuz. progresses..

A) caught up
1. The ---- of sunlight into electricity is possible
through the use of solar panels.. B) found out

C) put out
A) inversion
D) turned over
B) conversion
E) fed up
C) refinement

D) rotation 7. The judges of the international tribunal at The


Hague ----, on 1st February 2002, that charges
E) compression relating to all three wars ---- together..

2. Both Malatya and Gaziantep have grown A) would agree / had been heard
economically in the past three decades and both
are now considerably ---- than they used to be.. B) have agreed / have been heard

C) were agreed / will be heard


A) weaker
D) agreed / would be heard
B) stricter
E) had agreed / were heard
C) simpler

D) riskier 8. As acquisition opportunities within the EU ----


less, western European energy companies ---- to
E) wealthier turn eastwards in the hunt for
expansion opportunities..

3. When two or more atoms ----, they form a


molecule.. A) grow / are being forced

B) will grow / would have been forced


A) depart
C) are growing / would be forced
B) eradicate
D) have grown / had been forced
C) combine
E) grew / have been forced
D) correlate

E) withdraw 9. The writing of Turkish became easier ---- the


Latin alphabet was introduced in place of the
Arabic abjad in 1928..
4. NASA was created ---- in response to the Soviet
launching of Sputnik in1957..
A) however

A) sophisticatedly B) until

B) largely C) by the time

C) uniquely D) during

D) distantly E) when

E) safely
10. Play allows children to use their creativity ----
developing their imagination and physical,
5. The visitors were taken to the airport, and cognitive, and emotional strength..
there they were ---- by the assistant manager..

A) so that
A) made off
B) while
B) seen off
C) in case
C) put through
D) in order to
D) turned out
E) provided that
E) looked over
11. Once chemists had developed techniques to peer 16. Mammals range in size from bats, ---- weigh less
---- the heart of chemical reactions, they opened than 30 g, to the blue whale, which weighs more
---- a whole new world for study.. than 100,000 kg..

A) for / on A) many of them

B) up / in B) both of whom

C) at / with C) any of whose

D) into / up D) that

E) in / for E) some of which

12. There is nothing remarkable ---- the subject


matter of his novels, but he will always be
remembered ---- his delightfully vivid settings..

A) about / for

B) from / in

C) in / from

D) out of / through

E) of / over

13. In Ethiopia, primitive farming practices and


uncontrolled clearing of natural vegetation have
transformed much of the countrys highlands ----
bare landscapes, but all destroyed ---- erosion..

A) above / of

B) for / with

C) into / by

D) in / under

E) to / at

14. Experts believe that sunlight has the potential to


supply 5,000 times ---- energy ---- the world
currently consumes..

A) as much / as

B) both / and

C) more / that

D) so / that

E) such / so

15. In addition to its potential for immense strength


and its initial ability ---- to virtually any form,
concrete is fire resistant and ---- one of the most
common building materials in the world..

A) to be adapted / becomes

B) adapted / became

C) to adapt / has become

D) having adapted / had become

E) adapting / will become


21. V.
17. - 21.sorularda, aadaki parada
numaralanm yerlere uygun den szck ya
da ifadeyi bulunuz. A) irrelevant

B) keen

Canadianresearchers have discovered a set of genes that C) insurmountable


determine the lifespan of thecommon nematode, a type of
worm. This (I) ---- shedsnew light (II) ---- the aging D) sharp
processwhich may eventually allow them to delay the
inexorable process of aging anddeath. The team at McGill E) invaluable
University in Montreal was able to increase thelifespan of
the nematode fivefold by (III) ---- thenewly discovered
genes. Altering the genes apparently slowed the
metabolism ofthe worms to a more leisurely pace. This
(IV) ----may slow the accumulation of the DNA defects
thought to lead toaging. Although there are undoubtedly
other factors contributing to aging inhumans, researchers
are confident that the discovery will provide (V) ---- clues
about this mysterious process.

17. I.

A) invention

B) finding

C) conclusion

D) notion

E) struggle

18. II.

A) for

B) in

C) towards

D) on

E) about

19. III.

A) disrupting

B) suggesting

C) blending

D) manipulating

E) training

20. IV.

A) in turn

B) of late

C) in connection

D) instead of

E) after all
26. V.
22. - 26.sorularda, aadaki parada
numaralanm yerlere uygun den szck ya
da ifadeyi bulunuz. A) as much

B) far more

Solarradiation is the (I) ---- source ofenergy for the natural C) so


processes that create diversity and change on the
earth.However, if the earth continually received energy D) few or no
from the sun (II) ---- returning an equal amount to
space,the oceans (III) ----. Since theaverage temperature E) such
of the atmosphere remains (IV)---- the same from one year
to the next, the earth must be returningabout (V) ----
energy into space as itreceives from the sun. Of course, 27. - 36.sorularda, verilen cmleyi uygun
not all locations on earth have equal energygains and ekilde tamamlayan ifadeyi bulunuz.
losses. Each year, tropical regions receive a greater
amount ofenergy than they radiate back into space.

27. ----, which is why it is vital to ensure that it is


healthy..
22. I.
A) The liver is located under the ribs on the right hand
A) gloomy side of the body

B) principle B) Formerly, the liver received little attention

C) deceitful C) Liver surgery today has advanced enormously

D) alternate D) The truth is that the liver, as a hard-working organ,


has always received a great deal of medical
E) absent attention

E) The liver is involved in more than 500 body


23. II. functions, from detoxification to hormone balance

A) by 28. Although conventional IQ tests are good


predictors of college grades, ----..
B) as

C) without A) there are actually very few really exceptional


individuals
D) through
B) they are still the best single predictor of overall
E) to success

C) they are less valid for predicting later job success


24. III.
D) some weaker students have become charismatic
leaders
A) would boil
E) brain damage does not necessarily impair certain
B) were boiling types of intelligence

C) have boiled
29. If labour could be measured adequately in simple
D) boil homogeneous units of time, such as labour
hours, ----..
E) may boil

A) it covers many other kinds of areas as well, such as


25. IV. social security and worker satisfaction

B) a change in the organization of the communitys


A) adversely labour would be likely to increase the annual
production of wealth
B) exclusively
C) earlier economists failed to find a simple relation
C) rarely between the value of a product and the quantity of
labour that it embodied
D) preciously
D) different uses of the available supply of labour
E) nearly need to be considered

E) the problems of economics would be considerably


simplified
30. When a firm wants to fill a vacancy, ----.. 34. Many elderly people have no hint of Alzheimers
disease, ----..

A) there are always staff who want to leave for various


reasons A) for example, anxiety and frustration over their
failing memory results in aggression
B) the candidates have all been required to write an
essay on the current economic situation in Europe B) but some daily activities such as bathing and
dressing become challenging in the late stages
C) the applicants were invited for an interview before
a committee of specialists C) because disruption of brain cell function begins in
regions involved in forming new memories
D) it is essential that future managers must be trained
as thoroughly as possible D) then they may have difficulty in understanding
visual images and spatial relationships
E) it usually puts an advertisement in a newspaper
E) so it is not an inevitable consequence of aging as
most people believe it to be
31. As the miners have called off the strike ----..

35. ----, children almost never receive any such


A) he daily coal production had increased considerably explicit information about their native language
during the early years when acquisition takes
B) the Coal Board is prepared to reconsider the place..
proposals of the union leaders

C) if the chairman of the Coal Board expressed his A) Even though linguistic input does not always take
satisfaction place in the classroom

D) there would have been a sense of relief nationwide B) Until there is some hard evidence that language
and thought are strictly separated
E) the extent of the dispute was being ignored
C) As it is hard for adults to learn abstract ideas and
theories
32. Bulimia nervosa occurs with equal prevalence in
various social classes and ethnic groups, ----.. D) Unlike many second language learners who are
taught grammar rules and vocabulary directly
A) while people who have this disease are distressed
E) Because universal patterns of language
by it development can be explained by language-
specific input
B) but both eating disorders occur far more often in
women than men
36. Illegal aliens in America have been a problem -
C) even though anorexia nervosa has a mortality rate ---..
of 5% to 10%

D) since an antidepressant drug can often help A) when the early regulations encouraged immigration
control the disease
B) ever since the first immigration restriction was
E) whereas anorexia nervosa appears primarily imposed
among the upper classes
C) if necessary precautions are not taken
33. Women tend to choose fields of study like D) because it was virtually impossible to maintain
education, English, psychology, biology and art control over them all
history ----..
E) although the use of false IDs increases at an
alarming rate
A) if women had lower salaries, less laboratory space
and fewer resources

B) although women comprise 43 per cent of the


workforce, but only 23 per cent of scientists and
engineers

C) since many experts who study male/female


differences provide strong support for this idea

D) while men are much more interested in physics,


mathematics, computer science and engineering

E) because there are so few women in the fields of


mathematics and physical sciences
39. Mikroskop altnda incelenmek zere ine ile
37. - 42.sorularda, verilen ngilizce cmleye
alnm karacier dokusu rnei ieren karacier
anlamca en yakn Trke cmleyi, Trke biyopsisi, vakalarn yalnzca % 75i kadarnda
cmleye anlamca en yakn ngilizce cmleyi tany dorular. .
bulunuz.

A) A liver biopsy, which is carried out by the removal


with a needle of a sample of liver tissue for
microscopic examination, is indispensable for a
37. Dnyann en nde gelen dilbilimcilerinden biri
reliable diagnosis in 75% of cases.
olan Eugene A. Nida, ilkel topluluklarn dilleri ve
leheleri alannda uzmanlamtr..
B) About 75% of cases can be definitively diagnosed
through a liver biopsy, which involves the removal
A) Eugene A. Nida, one of the foremost linguists in the by needle of a sample of liver tissue for
world, has specialised in the field of the languages examination under a microscope.
and dialects of primitive communities.
C) The diagnosis of about 75% of cases can be
B) The famous linguist Eugene A. Nida is especially confirmed through a liver biopsy, which is
interested in the field of the languages and dialects performed with the removal of a sample of liver
of primitive communities. tissue for microscopic examination.

C) One of the best linguists in the world is Eugene A. D) A liver biopsy, in which a sample of liver tissue is
Nida, who has specialised in the field of the removed by needle for examination under a
languages and dialects of primitive communities. microscope, confirms the diagnosis in only about
75% of cases.
D) His specialisation in the field of the languages and
dialects of primitive communities is what has put E) In about 75% of cases, the diagnosis is confirmed
Eugene A. Nida among the foremost of the worlds by means of a liver biopsy, which is performed by
linguists. removing with a needle a sample of liver tissue for
examination.
E) One of the best-known linguists in the world is
Eugene A. Nida since he has specialised in the field
of the languages and dialects of primitive
communities.

38. Dnyada var olan tm elementlerden


muhtemelen hibiri, altndan daha fazla
insanlarn dlerini harekete geirmemitir..

A) Probably, the only element in the world ever to fire


mens dreams must be gold.

B) Gold must surely be the only element in the world


that has really stirred mens dreams.

C) Surely, no element in the world other than gold has


ever stirred mens dreams.

D) Probably, no element in the world other than gold


has the power to make men pursue their dreams.

E) Of all the elements present in the world, probably


none has stirred mens dreams more than gold.
40. Since comets appear in the sky without any
signal in advance, people in antiquity and
especially during the Middle Ages believed that
they had a special meaning..

A) Kuyruklu yldzlar gkyznde nceden herhangi bir


iaret vermeden grndkleri iin, eski ada ve
zellikle Ortaa boyunca, insanlar onlarn zel bir
anlam olduuna inanyorlard.

B) Eski ada ve zellikle Ortaada insanlar,


gkyznde nceden hibir iaret vermeden
beliren kuyruklu yldzlarn olaand bir anlam
olduuna inanmlardr.

C) Kuyruklu yldzlar eski ada ve zellikle Ortaada


gkyznde herhangi bir iaret vermeden ani
olarak grndkleri iin, insanlar onlarn zel bir
anlam olduuna inanmaktayd.

D) Eski ada ve zellikle Ortaada insanlarn, zel


bir anlam olduuna inandklar kuyruklu yldzlar,
gkyznde nceden herhangi bir iaret vermeden
belirirlerdi.

E) Kuyruklu yldzlarn nceden bir iaret olmakszn


gkyznde grnmeleri, eski ada ve zellikle
Ortaada insanlarn onlarda zel bir anlam
olduuna inanmalarna neden olmutur.
41. Because much is still unknown about breast
cancer and no single treatment works all the
time, doctors may have different opinions about
the most appropriate treatment..

A) Gs kanseri iin her zaman sonu veren tek bir


tedavi ynteminin bulunmamasnn yan sra
hastalkla ilgili yeterli bilginin olmamas nedeniyle
hekimler arasnda en etkili tedaviye ilikin farkl
grler mevcuttur.

B) Gs kanserine ilikin ok az ey ortaya konduu


ve her zaman sonu veren belirli bir tedavi yntemi
bulunamad iin, hekimler en geerli tedavi
hakknda farkl dnceler ileri srmektedir.

C) Hekimler gs kanseri iin en uygun tedavi


hakknda eitli grler ileri srmektedir; nk,
bu hastalk hakknda yeterli bilginin olmamas, tek
bir tedavinin her zaman geerli klnmasn
engellemektedir.

D) Gs kanseri hakknda yeterli bilgi olmad iin


her zaman sonu veren tek bir tedavi yntemi
yoktur ve bu nedenle hekimler en uygun tedaviye
ilikin elikili dnceler ortaya atmaktadr.

E) Gs kanseri hakknda ok ey hl
bilinmediinden ve hibir tedavi tek bana her
zaman sonu vermediinden, hekimler, en uygun
tedaviye ilikin farkl grlere sahip olabilirler.
42. In the 16th century, Turkish potters moved away
from their Chinese inspirations towards a Turkish
ceramic art of their own..

A) in etkisinden uzaklamak isteyen Trk mlekiler


16. yzylda kendilerine zg bir mlekilie
yneldiler.

B) 16. yzyln Trk mlekileri, in etkilerinden


uzaklap daha Trklere zel bir seramik sanatna
yaklatlar.

C) 16. yzylda in etkisinden kopan Trk mlekiler,


ok daha Trklere zel olan bir seramik sanatna
yneldiler.

D) 16. yzylda Trk mlekiler, in etkilerinden


uzaklaarak kendilerine zg bir seramik sanatna
yneldiler.

E) 16. yzylda Trk mlekilerini kendilerine zg bir


seramik sanat trne ynelten, onlarn in
etkisinden uzaklamas oldu.
46. According to the passage, the absence of an
43. - 46.sorular aadaki paraya gre
instructor protein ----..
cevaplaynz.

A) causes receptors to form on the surface of cells


A team of scientists have recently identified one of the B) results in the forming of a sheet by cells
molecules responsible for the tubular shape of blood
vessels. Originally known for regulating blood vessel C) helps cells to form a tube-like shape with its
development for life, the vascular endothelial growth neighbours
factor (VEGF) proteins analyzed by the research team
have proven to be more interesting than initially thought. D) attracts VEGF proteins to stem cells
The team found that a certain variation of VEGF attracts
E) helps cells dock with their closest neighbours
an 'instructor' protein. When this variant, with the
instructor protein attached, docks with a receptor on the
surface of acell, a signal telling the cell to form a tube-like
shape with its neighbours is sent to it. When the
instructorcell is absent, the cells line up next to one
another to form a sheet. Scientists believe that not only
could this contribution allow blood vessels to be created
from stem cells, but the discovery might be employed in
other tube-like structures in the body, such as the lungs
and the intestines. Far from being confined to blood
vessels, the discovery thus opens the door to resolving
the problem of three-dimensional reconstruction of organs
from stem cells.

43. It is clear that the study mentioned in the


passage revealed that ----..

A) VEGF proteins have functions which were


previously unknown

B) VEGF proteins help cells to form sheets

C) blood vessel development does not last a lifetime

D) VEGF proteins help to identify molecules that give


blood vessels their tubular shape

E) it is barely possible for VEGF proteins to regulate


blood vessel development

44. One can understand from the passage that the


instructor proteins ----..

A) regulate lifetime blood vessel development

B) make the analysis of VEGF proteins easier

C) are present in all cells found in organisms

D) contribute to the vascular endothelial growth

E) make cells form tube-shaped structures

45. It can be understood from the passage that the


discovery ----..

A) proves the impossibility of forming blood vessels


from stem cells

B) may only be used in blood vessel reconstruction

C) excludes medical applications related to lungs and


intestines

D) could help scientists to rebuild organs from stem


cells

E) confines stem-cell research to the reconstruction of


tube-shaped organs
49. According to the passage, general levels
47. - 50.sorular aadaki paraya gre
of intelligence ----..
cevaplaynz.

A) are usually particularly high when inconjunction


with the ability to reason withmathematical
What differences are there in intellectual precision
function between men and women? Major sex
differences in function seem to lie in patterns of ability B) are closely related to gender
rather than in overall level of intelligence (measured
as IQ), although some researchers have argued that there C) are harder to measure than are the variouskinds of
exists a small IQ difference favouring human males. ability
Differences in intellectual pattern refer to the fact that
D) vary very little, if at all, between men andwomen
people have different intellectual strengths. For example,
some people are especially good at using words, whereas E) are frequently low in people who have theability to
others are better at dealing with external stimuli, such deal with external stimuli
as identifying an object in a different orientation.
Two individuals may have differing cognitive
abilities within the same level of general intelligence. 50. In the passage, the phrase 'favouring
human males' (line 8) means ----..
Sex differences in problem solving have
been systematically studied in adults in
laboratory situations. On average, men perform better A) with the priority given to men
than women at certain spatial tasks. In particular,
men seem to have an advantage in tests that require the B) putting men in first place
subject to imagine rotating an object or manipulating it in
some other way. They also outperform women in C) with the improvement of men
mathematical reasoning tests and in navigating their way
D) recognizing male superiority
through a route. Women, on average, excel on tests that
measure recall of words and on tests that challenge E) giving encouragement to men
the person to find words that begin with a specific letter or
fulfill certain specific conditions. They also tend to be
better than men at rapidly identifying matching items and
performing certain precision manual tasks.

47. This passage is mainly concerned with ----..

A) establishing a connection between ability andIQ


levels

B) differences in cognitive abilities amongpeople,


especially between men and women

C) explaining why men perform most spatialtasks


better than women do

D) showing how the ability to recall and usewords is


reflected in IQ levels

E) the conduct of laboratory situations to gaugelevels


of ability

48. According to the passage manual tasks


that require a certain degree of exactness ----..

A) need to be monitored in laboratory situations

B) require higher levels of intelligence than do spatial


ones

C) are carried out well by women as are mathematical


reasoning tests

D) can be carried out by people with a low IQ

E) are generally performed well by women, but they


don\'t perform spatial ones so well
53. One understands from the passage that the
51. - 54.sorular aadaki paraya gre
Athenians ----..
cevaplaynz.

A) were more experienced in naval and tactical


matters than any other city-state
In ancient Greece, Athens and Sparta both wanted to be
the leading power. Athens was rich and more civilized, but B) had a deep-seated fear of the Spartans and,
Sparta was some what backward, though brave and well- therefore, kept a well-trained army
organized. The Athenians had asplendid navy, while the
Spartans had the best trained soldiers. Both city-states C) and the Spartans tried their best to live in peace
had allies that would support them. They also had and strengthen friendship
enemies who were looking for the chance to attack.
D) always believed that, against their enemies, they
Moreover, Athens and Sparta were rivals in another way.
could depend on the Spartans
Each believed that their city had the best laws and life
style. The Spartans thought that the Athenians were soft E) were biased against the Spartans and despised
and pleasure-seeking. The Athenians looked on the them
Spartans as being harsh and dull. In 461 B.C., Athens
attacked Sparta and its allies. The fighting continued for
14 years until the Athenians won, and Sparta was forced
to admit that Athens was the strongest power in Greece.

51. As described in the passage, in ancient Greece, -


---..

A) Sparta had the best legal system, which was much


envied by the Athenians

B) Athens had more allies than Sparta and


collaborated with them to win the war

C) Athens had the monopoly of overseas trade and


caused the collapse of the Spartan economy

D) there was a fierce rivalry between Athens and


Sparta, which ultimately led to a war

E) the economic prosperity of Athens was completely


ruined by the war with Sparta

52. As emphasized in the passage, contrary to


Sparta, Athens ----..

A) was a decadent city-state and, in military matters,


depended only on its allies

B) was much more advanced politically, culturally, and


economically

C) was unable to resolve the political crisis that


undermined its military power

D) had a lifestyle that was characterized by political


and moral corruption

E) always tried to impose its hegemony on all the city-


states in Greece
54. As pointed out in the passage, upon its defeat,
Sparta ----..

A) accepted the conditions of peace jointly specified


with the Athenians

B) lost its naval power completely to resist Athens

C) had no other choice but to recognize Athenss


superiority

D) was invaded by Athens, and its people were taken


prisoners

E) was deserted by its allies who, then, joined the


Athenians
58. The passage is mainly concerned with ----..
55. - 58.sorular aadaki paraya gre
cevaplaynz.
A) the Magellan space probes findings about Venus

B) the major question of why the surface of Venus is


Earth and Venus, being roughly the same size and covered by rocks
distance from the sun, are often regarded as twin planets.
So it is natural to wonder how the crust of Venus C) Venus cloud formations, which is an issue still
compares with that of our own world. Although centuries debated by scientists
of telescopic observations from Earth couldgive no insight,
beginning in 1990 the Magellan space probes orbiting D) planetary scientists on going research into Earths
radar penetrated the thick clouds that enshroud Venus continental crust
and revealed its surface with stunning clarity. From the
E) a scientific analysis of the pictures sent by the
detailed images of land forms, planetary scientists can Magellan space probe
surmise the typeof rock that covers Venus. It seems that
our sister planet is completely covered by rocks of basaltic
composition, which are very much like the dark, fine
grained rocks that line the ocean basins of
Earth.Magellans mapping, however, failed to find
extensiveareas comparable to Earths continental crust.

55. It is stressed in the passage that, in view of


itslargeness as well as remoteness from the sun,
Venus ----..

A) is far different from Earth

B) is very much like our own planet

C) is incomparable with any other planet

D) still seems to be incomprehensible

E) has always been a favourite for telescopic


observations

56. According to the passage, Magellans


explorationof Venus ----..

A) was largely prevented by the planets thick clouds


and turned into a total failure

B) benefited a great deal from the data obtained


through centuries of telescopic observations

C) has revealed a kind of rock composition which


resembles the rocks in Earths ocean basins

D) have enabled planetary scientists to explain why


the planet is enshrouded in thick clouds

E) not only showed clearly the size of the planet but


also how its thick clouds were formed

57. The passage draws attention to the fact that,


despite centuries of telescopic observations, -
---..

A) planetary scientists have failed to understand the


geological composition of Venus rocks

B) there has been no information available about the


surface of Venus

C) extensive areas of Venus have yet to be observed

D) the clouds that surround Venus have only once


been penetrated

E) planetary scientists were not aware of the fact that


Venus and Earth were twin planets
62. According to the passage, the reason that 5-year
59. - 62.sorular aadaki paraya gre
olds say that there are more checkers in a
cevaplaynz. straight row than in a cluster with the same
number is that they ----. .

The young childs reliance on visual impressions is made A) are trying to confuse the experimenter
clear by an experiment on the conservation of number. If
two rows of checkers are matched one for one against B) become confused when the experimenter moves
each other, young children will say, correctly, that the the checkers
rows have the same number of checkers. If the checkers
in one row are brought closer together to form a cluster, C) have a counting ability on the same level with that
5-year-olds say there are now more checkers in the of older children
straight row even though no checkers have been
D) depend on the outward appearance of things to
removed. The visual impression of a long row of checkers understand them
over rides the numerical equality that was obvious when
the checkers appeared in matching rows. In contrast, 7- E) are frustrated with the experiment and unwilling to
year-olds assume that if the number of objects was equal carry on
before, it must remain equal. At this age, numerical
equality has become more significant than visual
impression. 63. - 67.sorularda, karlkl konumann bo
braklan ksmn tamamlayabilecek ifadeyi
bulunuz.

59. The passage is mainly concerned with ----..

A) differences in counting ability between 5- and 7- 63. Hank :- Im travelling to South America next
week.
year-old children
Linda :- Oh, then you should have a typhoid
B) the game of checkers immunization.
Hank :- Why? Everyone knows it isnt 100%
C) the fact that older children are better at playing effective.
checkers than younger children Linda :- ----.

D) an incorrect assumption made by most 7-year olds


A) You should drink only carbonated bottled drinks or
water that has been boiled.
E) differences in perception between older and
younger children
B) Even so, it can offer you some protection against
the disease.
60. It can be inferred from the passage that, while
very small children depend more on what they C) You know that even if you are immunized, you may
see to form a concept of their environment, older still contract other gastro-intestinal disorders while
children ----. . youre there.

D) Are you travelling to any small villages? You will be


A) depend more on their intellect at greater risk if you are.

B) never make a mistake when counting objects E) Foods that are cooked and served hot are less
likely to cause an infection.
C) know that the experimenters are trying to trick
them
64. Daniel: Have you read about that interesting
D) are more interested in their friends NASA landing on Mars in 2004? Maxine : ----
Daniel: Exactly, and the results from both sites
E) have a better understanding of games were very interesting. Actually, contrary to
popular belief, water appears to have played
only a minor role in the past few billion years.
61. By the words conservation of number in the Maxine : Yes, I must admit I found that quite
passage is meant ----. . surprising..

A) numbers should be used sparingly so as not to run A) What was the purpose of the landing? Were they
out of them trying to find traces of life?

B) even when the members of a group move their B) Are you talking about the research that was able to
places, the number of the members remains determine the age of the planet?
constant
C) Was that really in 2004? I thought it was much
C) every child should learn how to play checkers more recent than that.

D) when the members of a group move their places, D) Do you mean the one where they landed two really
the number of members may change complex machines at two different sites?

E) the visual impression of a long row of checkers E) Wasn\'t that the experiment where they were able
taking precedence over numerical equality, at least to prove there was no water on the planet?
for very young children
65. Zeynep: Id like to pursue an academic career
68. - 71.sorularda, verilen cmleye anlamca en
after Ive graduated from my department.
Professor Raimes: Why not? You ought to check yakn cmleyi bulunuz.
with the Institute of Social Sciences.
Zeynep: Really? Why should I do so?
Professor Raimes: ----.
68. You dont have to attend the next meeting which
concerns the details of implementation, but
A) Theyll tell you all the requirements for admission. naturally well be delighted if you do..

B) Dont you know they havent announced any


vacancies in the department yet? A) We were unable to discuss the details of
implementation so that has been left for the next
C) Before that you should have submitted all your meeting which we sincerely hope you will be able
documents including the proof of language to attend.
proficiency.
B) The meeting to discuss the details of
D) Have you ever thought of becoming a teacher? In implementation doesnt concern you, but
fact, you can teach at the same time. nevertheless we shall be pleased if you decide to
attend.
E) It all depends whether the examining committee
will find your knowledge of field sufficient to pursue C) You neednt have attended the meeting concerning
an academic career. the details of implementation but a lot of people
were gratified by your presence.
66. Steve : I was reading an article about fish farms D) The details of implementation will have to be
and it sounds as if that is what we need to be discussed at the next meeting which you will
investing in for the future. presumably agree to attend.
Harold: But I thought they were one of the E) You arent obligated to be present when we meet
biggest causes of water pollution. to discuss the details of implementation, but it goes
without saying that well be very pleased if you do
Steve : ---- join us then.

Harold: Do you mean the part of the coast on


which they are located?. 69. It was not until the time of Shakespeare that
companies of players emerged Who made the
stage their profession..
A) True, but there\'s no other option if we want to
continue eating fish.
A) Before the age of Shakespeare companies of
B) That\'s true, but it really depends on where they professional actors were already beginning to
are situated. appear.

C) Yes, you\'re right, but things are improving in many B) By Shakespeare's time there were already groups
countries. of actors whose theatrical activities were
conducted on professional lines
D) I agree, but there are ways of reducing the
pollution along the coast. C) The actors of Shakespeare's time were
professionals and organised themselves into
E) I thought so too, but apparently they\'re not. It companies.
depends on the country.
D) By the age of Shakespeare, acting had become a
profession and the players were grouped into
67. Ken : - What are you going to do in Tunisia? various companies.
George : - Well, Im going to spend a lot of my E) Companies of players who made a profession of
time on the beach, but my wife plans to do a lot acting came into being for the first time in the age
of shopping. of Shakespeare.

Ken :- ---- .

George : - I suppose so. We usually do..

A) Havent you seen the Sahara desert and the Roman


ruins yet?

B) Then youll be coming back with a lot of heavy


luggage.

C) They say it is an attractive country especially for


those coming from Europe.

D) If you want I will give you a list of the hotels and


the restaurants in Tunisia.

E) As far as I know, the weather is very hot at this


time of the year.
70. The word robot was popularized in 1921 by
Czech playwright Karel Capek, but people have
envisioned creating robot-like devices for
thousands of years..

A) For thousands of years, the notion of creating


robot-like devices was a very popular on.

B) Robots have, in theory, been around for thousands


of years, but the first real robot was created by
Karel Capek in 1921.

C) People have had the idea of creating robots for a


long time, but it wasnt until 1921 that the term
robot became widespread through the work of
Karel Capek.

D) Robot-like devices were actually created thousands


of years ago, but were not popular until one
appeared in a play in 1921.

E) Karel Capek was the first person who took the idea
of creating robot-like devices seriously.
71. There is a growing body of research that shows 73. Locomotion can be considered to be a flow of
that optimism could extend your life.. mass from one location to another. ----. They
seek and find paths and rhythms that allow them
to move their mass the greatest distance per
A) Showing that you are an optimist could be a good expenditure of useful energy while minimizing
thing to do. thermodynamic imperfections such as friction..

B) More research could show a connection between


optimism and longevity. A) All of these designs allow for the maximum transfer
of material with the least amount of resistance
C) Being optimistic could be a life-saver if you need
one. B) A flow is an equilibrium of areas with high and low
resistivities
D) Extending the human life-span is somewhat
optimistic. C) Animals move on the surface of Earth in the same
way as rivers, winds and oceanic currents
E) Studies reveal that optimists may actually live
longer. D) A river basin configures and reconfigures itself so
that the water is discharged with the least
resistance through the mouth of the river
72. - 75.sorularda, bo braklan yere, parada
anlam btnln salamak iin E) One of the basic goals of any design whether its
an animal or a machine is to get maximum output
getirilebilecek cmleyi bulunuz.
for minimum energy

72. Lamartine was a French poet who lived between


1790 and 1869. At school he learned Greek,
Italian and English, and was strongly influenced
by the classics. ----. From 1811 to 1819 he
attempted to write an epic and several tragedies
in the classical manner. Yet it was as a lyrical
poet that he achieved fame..

A) In general, the Romantic writers placed the


individual, rather than society, at the centre of their
vision

B) The term Romantic Age is used to describe life


and literature in Europe in the late eighteenth and
early nineteenth centuries

C) He was a great admirer of Petrarch, Shakespeare


and Byron

D) The French Revolution, which he strongly


supported, gave great importance to liberty and
equality

E) Like any Romantic, he believed in the possibility of


progress and improvement for humanity
74. Corporate globalization has been increasing
since the middle of the 20th century, as
successful exporters gradually put down roots in
their foreign markets, to be closer to them and
to save transport costs. Over time, a number of
these integrated their operations into truly
global companies. ---- The main beneficiaries
have been India, notably in software, and China
in manufacturing..

A) Many individuals wont start businesses in foreign


countries.

B) Both countries are growing fast, and the two


countries may become economic superpowers
within the next 20 years.

C) Some believe that even lower-cost outsourcing


destinations like Ghana and Vietnam will become
more competitive.

D) The next phase saw Western firms moving


manufacturing activities to cheaper labour
markets.

E) Like many of the ideas management has to deal


with, globalization is not a new one.

75. The word panic, meaning fear, comes from the


name of the Greek god Pan, a noisy musician
who was thought to play his pipes day and night
in the woods. Long ago people thought Pan made
the sounds that frightened travellers in the
wilderness at night. ----..

A) The word panic soon came to describe their fear

B) Sometimes an author does not state directly


everything that is happening

C) One way to find the meaning of unfamiliar words is


to use the context

D) A synonym is a word that has almost the same


meaning as another word

E) When we look up an unfamiliar word in a


dictionary, we may find more than one definition
76. - 80.sorularda, cmleler srasyla
okunduunda parann anlam btnln
bozan cmleyi bulunuz.

76. (I) Shipping has long been a crucial part of the


Egyptian economy. (II) With coastlines on the
Mediterranean, the Red Sea and the world
famous Suez Canal, it could hardly be otherwise.
(III) But the government has put a lot of effort
into transforming the sector from a mere means
of transporting goods into and out of the country
into a much more central part of the economy.
(IV) The country has enjoyed good tourism
revenues in recent years despite earlier security
problems and terrorism scares. (V) It is now
attempting to use the countrys geographical
position to turn it into one of the leading
transhipment centres in the world..

A) I

B) II

C) III

D) IV

E) V
77. (I) The Asian financial crisis is generally 78. (I) Smoking has become unfashionable in most
considered to have started in July 1997, with the Western countries over the past decade or more.
devaluation of the Thai baht. (II) Thailand had (II) Yet many people in these countries still
been showing signs of financial strain for more smoke. (III) The campaign clearly highlighted the
than a year. (III) During 1996, it became benefits of a smoke-free work place. (IV) Even
apparent that too many office towers had been laws against smoking in shared work spaces and
built; first the nations real estate market, then on public transport have had limited success in
its stock market, went into decline. (IV) In the reducing the number of smokers. (V) In these
first half of 1997, speculation about a possible countries, the highest rates of smoking tend to
devaluation led to a remarkable loss of foreign be found especially among people in the 20-29
exchange reserves and on July 2nd, the country age group, teenage women and the working
attempted a controlled 15%devaluation. (V) class..
Together with Thailand, Malaysia and South
Korea were feeling the effects of economic
slowdown in their largest industrial neighbour, A) I
Japan..
B) II

A) I C) III

B) II D) IV

C) III E) V

D) IV
79. (I) Humans communicate their inner thoughts via
E) V language. (II) But some linguists think that this
is not a strict, one-way case of cause and effect;
the language we speak may determine the
thoughts that we try to express. (III) If language
influences our thoughts, could it also affect the
way we look at the world? (IV) In the early 20th
century, Edward Safirand Benjamin Whorf first
supported the theory that language affects the
way humans interact with the world. (V) In the
late 1950s, Chomsky came up with the idea that
language is a natural ability with a deep, unified
structure..

A) I

B) II

C) III

D) IV

E) V
80. (I) The telephone has come along distance
indeed since Alexander Graham Bell invented the
first crude transmitter in 1876. (II) Today we can
place calls from automobiles, ships at sea and
places in the air. (III) Using the phone system,
we can fax documents around the world. (IV)
Most local telephone markets are still
monopolized by one company, but competition is
growing. (V) Soon we will be able to dial up
images of computerized data..

A) I

B) II

C) III

D) IV

E) V
SORU CEVAP SORU CEVAP

1 B 41 E

2 E 42 D

3 C 43 A

4 B 44 E

5 B 45 D

6 A 46 B

7 D 47 B

8 A 48 E

9 E 49 D

10 B 50 B

11 D 51 D

12 A 52 B

13 C 53 E

14 A 54 C

15 C 55 B

16 E 56 C

17 B 57 B

18 D 58 A

19 D 59 E

20 A 60 A

21 E 61 B

22 B 62 D

23 C 63 B

24 A 64 D

25 E 65 A

26 A 66 B

27 E 67 B

28 C 68 E

29 E 69 E

30 D 70 C

31 B 71 E

32 E 72 C

33 D 73 C

34 E 74 D

35 D 75 A

36 B 76 D

37 A 77 E

38 E 78 C

39 D 79 E

40 A 80 A
21
6. The foreman has a way of making most people do
1. - 16.sorularda, cmlede bo braklan yerlere
what he wants, but somehow I don't think he'll
uygun den szck ya da ifadeyi bulunuz. manage to ---- the new director!.

A) get around
1. For many years, Britains National Health Service
has provided a basic standard of ---- at low cost.. B) make up for

C) turn over
A) reversal
D) keep up with
B) symptom
E) rule out
C) expenditure

D) expectancy 7. French lorry-drivers ---- on strike and now they -


--- the roads, which is their usual way of showing
E) care their discontent..

2. Vacationers in Bodrum and Marmaris are having A) went / had blocked


a hard time finding ---- rental units, as prices
have risen above $1,000 for houses close to the B) will go / would block
sea..
C) have gone / are blocking

A) affordable D) were going / block

B) productive E) would go / are going to block

C) attentive
8. Since Swedish industrial relationships ----
D) reducible so many fundamental changes, it is
hardly surprising that the existence of a
E) prestigious Swedish model ----..

3. Although China has, in recent years, made A) were undergoing / will be questioned
notable advances, particularly in manufacturing,
agriculture will long ---- fundamental to the B) underwent / has been questioned
economy..
C) had undergone / had been questioned

A) produce D) are undergoing / was being questioned

B) differ E) have undergone / is being questioned

C) extend
9. ---- proving useful in many scientific fields,
D) remain computers-generated knowledge, ---- Professor
Hayes acknowledges, is not without pitfalls..
E) resist
A) Although / because
4. Countries vary ---- in their approach to human
rights and their record of human rights B) Since / while
protection..
C) While / as

A) harmfully D) As long as / even if

B) destructively E) Once / whether or not

C) strongly
10. ---- the span of a suspension bridge increases,
D) widely the structure develops a 'gravity' stiffness
due to its own weight..
E) daily
A) As
5. He must have had some sound reasons for ----
such a rigid timetable.. B) However

C) Whereas
A) setting out
D) While
B) drawing up
E) In case
C) running up

D) turning off

E) looking into
11. ---- the interest rates are concerned, it looks as if 16. Dancing was a formal element in Christian
the inflation has at least been curbed.. worship until the A.D. 1100's, ---- religious
leaders began to prohibit it because they
believed it was too worldly an activity..
A) As far as

B) Owing to A) when

C) By way of B) why

D) Just as C) whose

E) As regards D) which

E) where
12. As long as the two applications ---- similar with
comparable requirements for paint protection
and performance, the experiences of others ----
valuable lessons..

A) were / had taught

B) will be / teach

C) may be / taught

D) has been / would have taught

E) are / can teach

13. Tibetan monks, who are experts ---- meditation,


rid themselves of negativity ---- augmenting the
brains gamma waves, which can be measured in
a lab..

A) of / from

B) on / about

C) for / through

D) in / by

E) at / towards

14. The main attention at the World Economic


Forum, when it was not focused ---- China,
centred on Africas remarkable economic growth
---- the past couple of years..

A) in / with

B) on / over

C) of / towards

D) by / at

E) from / through

15. Alternative energy is the name given to any type


of energy used to replace a different source of
energy, often ---- the negative consequences of
its use..

A) because of

B) besides

C) in order to

D) with the exception of

E) in spite of
21. V.
17. - 21.sorularda, aadaki parada
numaralanm yerlere uygun den szck ya
da ifadeyi bulunuz. A) might be accepted

B) must have been accepted

Thirty-eightof the fifty American states have laws C) has been accepted
permitting them to execute people (I) ---- guilty of capital
crimes. Capitalpunishment is one of the widest debated D) will have been accepted
issues in the United States today.Opponents say the death
sentence is too (II) ----.Others say that people who kill E) had been accepted
should die for their crimes. In 1972, theSupreme Court of
the United States banned executions. It (III) ---- its
decision on two amendments to theConstitution. The court
ruled that the death penalty was cruel and unusual (IV) ----
the way the states enforced it. Yetthe decision left open
the possibility that capital punishment (V) ---- in the
future. The decision meant thiscould happen if people
were executed only for some crimes, under
limitedconditions.

17. I.

A) to find

B) finding

C) to be found

D) find

E) found

18. II.

A) moderate

B) preventable

C) severe

D) loose

E) relevant

19. III.

A) depended

B) proved

C) influenced

D) based

E) covered

20. IV.

A) instead of

B) because of

C) consistent with

D) in spite of

E) in return for
26. V.
22. - 26.sorularda, aadaki parada
numaralanm yerlere uygun den szck ya
da ifadeyi bulunuz. A) the fact that

B) whatsoever

Aspeaker (I) ---- a persuasive speechcan make use of the C) how


testimonial technique. The speaker chooses a person
admired by the audience. Sometimes the person is local, D) if
(II) ---- more often he/she is a nationalfigure. Names (III) -
--- Abraham Lincoln, Susan B. Anthony, DwightEisenhower, E) whom
and Martin Luther King inspire good feelings in the
audience. The speaker then implies that if one of
theseheroic people (IV) ---- there, thatperson would do 27. - 36.sorularda, verilen cmleyi uygun
something this way or that way. There is seldom any ekilde tamamlayan ifadeyi bulunuz.
evidence that these people ever spoke about theparticular
issue, but the speaker claims he or she knows (V) ----
these famous people would feel.
27. ---- where you can read a book or meet with
friends..

22. I. A) Security at the new art museum is provided by


young artists
A) calculating
B) In Paris new contemporary art space, the Palais de
Tokyo, there is a salon
B) relating
C) The new contemporary museum is being built for
C) damaging
the French collector Franois Pinault
D) delivering
D) None of the French contemporary artists enjoys
international prominence
E) educating
E) France never paid much attention to its own
23. II. contemporary artists, beginning with the 19th
century impressionists

A) because
28. ---- before the town planners took their job
B) indeed seriously.

C) also
A) City planning has continued to be their prime
D) but concern

E) contrary B) A great many ugly apartment blocks were built

C) A good architect views his work within the context


24. III. of the area at large

D) In this respect new cities are at a great advantage


A) like
E) New cities never have the same atmosphere as the
B) just old

C) similar to
29. ---- as if surgery may be unnecessary..
D) as

E) unlike A) They had already explained

B) In the light of the latest report, it looks


25. IV.
C) The doctor might have decided

A) has been D) According to the final tests one can conclude

B) may be E) The patient was encouraged

C) will be

D) had been

E) were
30. However far-fetched the story may seem, ----..

A) people say that truth is stranger than fiction

B) newspaper headlines are, after all, far from reliable

C) we can assume that there is a basis of truth in it

D) we have always known she loves to exaggerate

E) it really doesnt matter if some details are wrong


31. As farmers dont want to lose their animals to 34. ----, the person is said to suffer circulatory
disease, ----.. shock..

A) they grow several different kinds of crops A) Just as the heart is severely damaged from any
cause
B) they vaccinate their animals
B) Even though any factor that interferes with venous
C) they use pesticides to protect their crops from return can also lead to decreased cardiac output
insects
C) If ever the cardiac output falls below that level
D) they suffer from communicable diseases which is required for adequate nutrition of the
tissues
E) their animals may suffer from such illnesses
D) So long as the cardiac output fell so low that the
tissues throughout the body began to suffer
32. Debates arise over which voting technology is nutritional deficiency
the most accurate and least susceptible to
tampering ----.. E) While the cardiac outputs in healthy human beings
are surprisingly constant
A) that could easily have been prevented

B) whenever national elections are held

C) whether it is plausible for the majority or not

D) until balloting security is jeopardized

E) as if all votes have been counted

33. The human ear is equipped to register sound


frequencies between about 20 and 20, 000 cycles
per second ----..

A) even if the inner ear consists of a complicated


group of interconnected canals and sacs

B) unless dogs and some other animals can hear


sounds of various frequencies

C) while the ear is ten times more sensitive than the


eye

D) since one of the functions of the human ear is to


help maintain equilibrium

E) although individuals vary greatly as regards their


age and health
35. Movements of the hands and head are often used
in order to emphasize certain points that are
being made, ----..

A) because they have a tremendous impact on the


way we live now

B) although people use their body effectively and


efficiently

C) since the speaker wants to convey particular


nuances of meaning

D) as the best method of communication is obviously


the written work

E) just as social relationships take a long time to


develop

36. Japans efforts to reduce piracy incidents in the


area were highly successful ----..

A) so several other countries adopted similar tactics

B) unless various local security forces had also


responded

C) though the usual policy of the government will be


one of non-interference

D) once they choose to rise to the challenge

E) when it lacked the desire to assert itself


37. - 42.sorularda, verilen ngilizce cmleye
anlamca en yakn Trke cmleyi, Trke
cmleye anlamca en yakn ngilizce cmleyi
bulunuz.

37. Emilyyi tekrar grdmzde, onun olduka


imanladn ve salarnn beyaza dnmekte
olduunu fark ettik..

A) When we met Emily, we saw that she looked


extremely fat, and that her hair had turned white.

B) When we next saw Emily, we noticed that she had


grown rather fat and that her hair was turning
white.

C) By the time we met Emily again she had grown fat


and her hair was nearly white

D) When we saw Emily last, we were surprised to see


how fat and white haired she had become.

E) When we met Emily again, she was looking quite


fat and her hair was rather white.
38. 2000 seimlerinden sonra Bakan George W.Bush 39. Polisiye roman, 19. yzyln sonuna doru,
her ne kadar lke ii sorunlara odaklanm olarak zellikle Sherlock Holmes hikyeleriyle ok
Beyaz Saraya gelmise de ksa srede, dikkatini yaygnlk kazanan bir yazn biimidir..
d konulara evirmek zorunda kald..

A) The Sherlock Holmes stories helped to make the


A) Although, after the 2000 election, President George detective novel so popular at the end of the 19th
W. Bush came into the White House focused on century.
domestic issues, he was soon forced to turn his
attention to foreign affairs. B) The detective novel is a form of writing, which
became very popular towards the end of the 19th
B) Following the 2000 election, President George W. century, particularly through the Sherlock Holmes
Bush came into the White House with a full stories.
awareness of domestic problems but, within a short
time, he had to focus his attention on international C) The most popular examples of the detective novel,
issues. written in the late 19th century, were the Sherlock
Holmes stories.
C) After the 2000 election, when President George W.
Bush entered the White House primarily concerned D) By the end of the 19th century, the Sherlock
with domestic matters, his attention was Holmes stories had made the detective novel one
immediately focused on foreign affairs. of the most popular types of writing.

D) Even if, after the 2000 election, President George E) With the Sherlock Holmes stories towards the end
W. Bush began his tenure in the White House of the 19th century, the detective novel became
thoroughly aware of domestic affairs, his attention quite the most popular type of writing.
soon turned to international matters.

E) A short time after the 2000 election, President 40. Modern astronomy began with Copernicus in the
George W. Bush began to focus his attention on sixteenth century, who asserted that the Sun
foreign issues, although he had come into the was at the centre of the solar system..
White House mainly concerned with domestic
affairs.
A) Kopernik, on altnc yzylda, Gnein, gne
sisteminin merkezini oluturduunu belirtmi ve
bylece modern astronomiyi balatmtr.

B) Gne sisteminin merkezinin, Gne olduunu ileri


sren Kopernik, modern astronominin on altnc
yzylda baladn ifade etmitir.

C) Modern astronominin balangc, Kopernikin, on


altnc yzylda, Gnein, gne sisteminin merkezi
olduunu ileri srmesine dayanr.

D) On altnc yzylda Kopernikin, gne sisteminin


merkezini, Gnein oluturduunu ifade etmesiyle,
modern astronomi balamtr.

E) Modern astronomi, on altnc yzylda, Gnein,


gne sisteminin merkezinde olduunu ne sren
Kopernikle balamtr.
41. In his controversial book The Break down of
Climate: Towards A Global Disaster?, Peter
Bunyard maintains that severe man-made
climate changes could occur much sooner than
previously predicted..

A) ok tutulan klimin Bozulmas: Kresel Bir Felakete


mi Doru? adl kitabnda Peter Bunyard, insann
neden olduu ciddi iklim deiikliklerinin, daha
nce tahmin edilenin tersine, hemen olabileceini
savunmaktadr.

B) Peter Bunyardn klimin Bozulmas: Kresel Bir


Felakete mi Doru? adl ok konuulan kitabnda
savunduu ve insandan kaynaklanan ciddi iklim
deiikliklerinin tahmin edildiinden daha erken
meydana gelecei yolundaki gr, tartmalara yol
amtr.

C) Peter Bunyard, insandan kaynaklanan ciddi iklim


deiiklerinin daha nce tahmin edilenden de hzl
oluabilecei grn, klimin Bozulmas: Kresel
Bir Felakete mi Doru? adl ok tartlan kitabnda
dile getirmitir.

D) ok tepki eken, klimin Bozulmas: Kresel Bir


Felakete mi Doru? adl kitabnda Peter Bunyard,
insann yol at ciddi iklim deiikliklerinin, daha
nce de tahmin edildii gibi hzla olumaya
baladn belirtmektedir.

E) klimin Bozulmas: Kresel Bir Felakete mi Doru?


adl tartmal kitabnda Peter Bunyard, insann
neden olduu ciddi iklim deiikliklerinin, daha
nce tahmin edilenden ok daha erken meydana
gelebileceini ileri srmektedir.

42. Historically, geographically, and economically,


Finland is a bridge between East and West,
linking Western Europe and the former states of
the Soviet Union..

A) Hem Bat Avrupa hem de eski Sovyetler Birlii


devletleri ile balar olan Finlandiya, Dou ve Bat
arasnda tarih, corafi ve ekonomik bir kpr
grevi grmektedir.

B) Bat ve Dou arasnda tarih, corafi ve ekonomik


bir kpr grevi gren Finlandiya, Bat Avrupa ve
eski Sovyetler Birlii devletleri arasnda balar
kurulmasna yardmc olur.

C) Finlandiya Dou ve Bat arasnda tarih, corafi ve


ekonomik bir kpr grevi grdnden Bat
Avrupa ve eski Sovyetler Birlii devletleri ile gl
balar vardr.

D) Bat Avrupann yan sra eski Sovyetler Birlii


devletleri ile balar olan Finlandiyann Dou ve
Bat arasnda tarih, corafi ve ekonomik bir kpr
grevi grmesi kanlmazdr.

E) Tarih, corafi ve ekonomik olarak Finlandiya, Dou


ve Bat arasnda bir kpr olup, Bat Avrupa ve eski
Sovyetler Birlii devletlerini birbirine balar.
46. It is clear from the passage that the CluR1
43. - 46.sorular aadaki paraya gre
protein ----..
cevaplaynz.

A) excites new addictive behavioural patterns that


previously did not exist
A study by a group of scientists has resulted in amajor
step forward in overcoming drug addiction. Asaddiction B) is the by-product of molecular changes in the brain
was known to cause molecular changes in the brains of
addicts, causing their neurons to transmit much stronger C) requires the re-administration of cocaine
signals of dopamine, amessenger molecule involved in
reward-seeking behaviours, the scientists were hoping to D) immediately re-ignites the addiction when switched
prove experimentally that certain key proteins in the on
dopamine-producing neurons influence drugaddiction. It
E) can increase resistance to addiction when active
was an inspired guess, as the studyfound that mice in
which these key proteins had beenselectively switched off
displayed clear addictive behaviour. For instance, mice in
which the CluR1 protein was switched off showed a much
longer period of dependence. Conversely, re-
administration of cocaine after a long break immediately
re-ignited the addiction, but mice whose NR1 protein had
been deactivated resisted relapsing into addictive
behaviours. The ability of these proteins to determine
addictive behavioural patterns makes them fascinating.

43. According to the passage, the NR1 protein in


themice involved in the experiment ----..

A) helps to determine CluR1 protein levels

B) could not be deactivated completely

C) is found to have no relevance to addictive


behaviours

D) causes a relapse into addiction when active

E) is currently the only substance used in the


treatment of addiction

44. One can understand from the passage that drug


addiction ----..

A) diminishes the strength of neural signals of


dopamine

B) switches off reward-seeking behaviours

C) affects the molecular structure of the brain

D) exterminates dopamine-producing neurons

E) causes a selective deactivation of proteins

45. As can be understood from the passage, the


discovery ----..

A) could offer new ways of treating addictive


behaviours in humans

B) showed that addictive behavioural patterns are too


complex to be classified

C) proved the importance of refraining from long


breaks between cocaine doses

D) revealed that mice and human addictive


behavioural patterns were completely different

E) may, in the future, be used to re-ignite substance


addiction
49. According to the passage, the ideas of two
47. - 50.sorular aadaki paraya gre
linguists received approval ----..
cevaplaynz.

A) which, however, did not last long as there was not


sufficient evidence
The notion that different languages may result from
different cognitive skills goes back centuries. Since the B) but later were rejected by other linguists on the
1930s, the idea has become associated with American grounds that they were racist
linguists Edward Sapir and Benjamin Lee Whorf, who
studied how languages vary and proposed ways that C) when they were first presented to the scientific
speakers of different tongues may think differently. circles in the 1970s
Although their ideas met with much excitement early on,
D) following their successful attempts to prove them
there was one small problem: a near complete lack of
scientifically
evidence to support their claims. By the 1970s, many
scientists had become disappointed with the Sapir-Whorf E) based on past evidence that language and thought
hypothesis, and it was all but abandoned as a newset of are universally combined
theories claiming that language and thought are universal
came onto the scene. But now, decades later, a concrete
body of practical evidences howing how languages shape
thinking has finally emerged. The evidence overturns the
long-standing dogma about universality and yields
fascinating insights into the origins of knowledge and the
construction of reality. The results have importan
timplications for law, politics and education.

47. The writer of the passage is convinced that ----..

A) the idea that speakers of different languages think


differently is undisputed among scientists

B) the Sapir-Whorf hypothesis can now be supported


with the emergence of a concrete body of empirical
evidence

C) the two American linguists failed to persuade the


critics despite abundant evidence

D) the fresh evidence found recently will be unable to


revive the ideas of the two American linguists

E) if Edward Sapir and Benjamin Lee Whorf had


presented their ideas, they would have been
proven wrong

48. As stated in the passage, in the 1930s, Edward


Sapir and Benjamin Lee Whorf ----..

A) varied to a great extent in the way they studied


languages

B) opposed the idea that different languages may


reveal different cognitive skills

C) suggested that speakers of languages may differ in


the way they think

D) disagreed with each other about how languages


are linked to cognitive skills

E) worked together to explain he differences between


the languages
50. We can understand from the passage that
presently the Sapir-Whorf hypothesis ----..

A) lays down all the principles of how reality is


constructed through language

B) claims that languages emerge long after thoughts


are shaped by reality

C) is receiving its deserved acclaim through fresh


evidence recently uncovered

D) is no longer supported by a concrete body of


empirical evidence suggesting universality of
perception

E) has proved to be very important to law-makers,


politicians and educationalists
54. The writer claims that ----..
51. - 54.sorular aadaki paraya gre
cevaplaynz.
A) sharing information with other parents with the
same problems may be fruitful
If your child has a learning difficulty, you may be eager for B) visiting certain sites on the Internet may help only
information and support. Learning disabilities (LD) may be some parents
the source of your childs struggle. Beware of the popular
myths and misconceptions about LD. You can visit the C) parents should not trust the information in
website www.schwablearning.org, a parents guide to LDrelated articles, publications, and other
helping kids with learning difficulties. They offer free resources
information you can trust: research-based articles,
D) parents cannot overcome LD in their children, no
publications, other resources and an online community of
matter how hard they try
parents who share the same concerns.Send your child to a
special institution, such as that described by E) children with LD may develop self-confidence if
www.sparktop.org, a safe, fun place where kids with parents do not intervene at all
learning difficulties can gain confidence about themselves
and how they learn.

51. One of the major ideas in the passage is that -


---..

A) children with LD do not worry about how they learn

B) if your child is struggling to learn, you should


distract him with fun activities

C) LD makes children lose confidence in themselves

D) parents should try to get free help, since costs of


raising a family put pressure on the budget

E) parents aim should not be fun for their children,


so special institutions must be avoided

52. The purpose of the writer is to ----..

A) give guidance and advice

B) warn against LD

C) give scientific information about LD

D) compare children with LD to children without LD

E) give information about relevant publications

53. From the passage, we understand that parents of


children with LD ----..

A) do not believe in myths and misconceptions

B) write research-based articles

C) all send their children to fun institutions

D) may be very interested in finding out about LD and


getting help

E) do not trust the information they get


57. It is clear from the passage that the new towns
55. - 58.sorular aadaki paraya gre
and residential areas built after World War II in
cevaplaynz. Europe ----. .

A) were the work of a very talented group of young


During the hunger winter of 1944 in Amsterdam, over architects
20, 000 people died of starvation. Many of the citys trees
were cut down, and the interiors of abandoned buildings B) immediately became popular and have remained
broken up for fuel. When peace came this once most so to this day
beautiful and urbane of cities was in urgent need of large-
scale reconstruction. In the years following the end of C) became the most beautiful areas in and around the
World War II in Europe, modern architecture had an cities
unprecedented opportunity to demonstrate a socially
D) failed to provide a sense of community for the
minded, urban style. The consensus today is that in most residents
places it failed. The young Dutch architect Aldo van Eyck
was one of the earliest critics of the mechanistic approach E) had playgrounds designed by van Eyck
taken by his modernist colleagues to urban
reconstruction. The failure of architecture and planning to
58. According to the passage, Aldo van Eyck ----..
recreate forms of urban community and solidarity has
become a problem in post-war Europe, as so many
acclaimed housing estates, new towns, or newly designed A) approved of the mechanistic design approach of his
urban quarters, around Europe, have been troubled by modernist colleagues
vandalism, disrepair and abandonment. Van Eyck saw this
coming. In 1947 at the age of 28, he went to work for the B) thought that many post-war residential areas
Office for Public Works in Amsterdam and, as his first deserved acclaim
project, built a small playground. This was in line with his
C) viewed the post-war period as an opportunity to
belief that by promoting and shaping the dailyencounter
display his design ability
or in between-ness of social space, architecture could
humanize cities and create public trust. D) thought the post-war period was a good
opportunity to destroy older buildings and design
modern cities

55. We understand from the passage that, in the late E) believed that urban architecture had the power to
1940s, Europe was in need of massive create a sense of solidarity and trust in a citys
reconstruction due to ----.. residents

A) the destruction caused by World War II

B) the mechanistic approach taken by post-war


architects to urban reconstruction

C) vandalism, disrepair and abandonment of property

D) the unprecedented opportunity to demonstrate a


socially minded, urban style

E) the failure of modern architecture

56. It is stated in the passage that although ----, it


was mostly unable to do so..

A) a small playground was the first project designed


by van Eyck while at the Office for Public Works in
Amsterdam

B) modern European architecture had a chance after


World War II to create an attractive new style in
cities

C) post-war architecture in Europe was greatly


influenced by the ideas of Aldo van Eyck

D) Amsterdam suffered major destruction during


World War II

E) post-World War II architecture in Europe took a


mechanistic approach
62. It is clear from the passage that the doctors
59. - 62.sorular aadaki paraya gre
blindfolded the volunteers ----..
cevaplaynz.

A) as if the subjects visual cortexes were shown to


have been activated
The loss of one sense encourages the development of the
four others. This has now been demonstrated convincingly B) in case the subjects carried out exercises designed
in a study by a team of doctors. During five days, to stimulate their sense of touch
volunteers taking part in the study were blindfolded and
asked to carry out exercises designed to stimulate the C) so as to see how their sense of touch responded to
sense of touch. When their brains were then tested using the temporary loss of sight
an MRI, an area of the visual cortex was shown to have
D) in order to see how long it took to lose a natural
been activated as if, being underused, this area was
skill
brought in to help the subjects when they had to rely on
touch. About 24 hours later, after the blindfolds came off, E) so that they could force the subjects to rely on their
the participants lost this aptitude. Previously, scientists visual skills
had always believed that the brain was organized
intodistinct and highly-specialized systems. This new
study, however, shows that the human brain has the 63. - 67.sorularda, karlkl konumann bo
ability to reorganize itself. In addition, the rapid braklan ksmn tamamlayabilecek ifadeyi
reversibility of the process suggests that it is not based on bulunuz.
the creation of new nervous connections but on the
activation of previously inhibited zones.

63. Lecturer: As the division of labour has become


more specialized, it is no longer possible for
59. According to the passage, zones in the brain that skills to be
were inactive before ----.. passed down within the family.

Student: What could be the consequences of


A) continually undergo reverse processes such a thing in modern life?

B) activate the previously inhibited nervous Lecturer: For one thing, it has increased the
connections importance of education in all societies to a
great extent.
C) become inhibited when used excessively
Student: ----
D) are created by new nervous connections
Lecturer: Absolutely!.
E) may become activated when one sense is lost

A) Then, youre implying that this should signal the


60. According to the passage, the study made it
end of centuries of adopting parents professions.
clear that ----..
B) That explains why some social norms had to be
A) the volunteers lost their aptitude when blindfolded internalized in society along with a number of other
rules.
B) the brain is organized into highly-specialized
systems C) In line with that, individualism gained popularity
over collectivism, and self-centrism was promoted
C) MRI tests yielded no reliable results on the by the media.
activation of the visual cortex
D) But, we should be dealing with the equality of
D) self-reorganization is an important trait of the opportunity that the current educational system
human brain provides.

E) one of the five senses was usually more developed E) As a matter of fact, the distinction between the
than the other four in humans educated and uneducated is far greater than was
thought.

61. One can understand from the passage that ----..

A) the loss of one sense forces the brain to reorganize


itself

B) the visual cortex is stimulated when under-used

C) it took about one day for the subjects to get used


to visual stimuli

D) the subjects were people with poor or no eyesight

E) new nervous connections are rapidly reversed by


active zones
64. Vicki : Ive just eaten a large meal, and Im
already hungry again!
Bryan : ----
Vicki : I had soup and bread, and a baked potato
with vegetables.
Bryan : Youre hungry again because you ate
mostly carbohydrates. Your body has already
used the energy from your meal and is looking
for more energy..

A) Are you feeling angry about something? That


always makes me feel hungry!

B) How long ago did you have lunch?

C) Youre eating far too quickly.

D) What did you eat for lunch?

E) Whats your favourite meal?


65. Professor: Globalization is to a certain extent 67. Aya : - Do you think environmental factors like
reversing nowadays as countries weigh the risks diet and stress affect the ageing process as
of not having enough goods in their own much as the decline of hormonal systems?
markets. Berkan : - ----
Student: Can you give us an example? Aya : - So, you mean physiological and
Professor: ----. environmental factors contribute to ones
longevity to the same degree.
Berkan : - Definitely! I also think living in an
A) Well, the world is still a highly uncertain place. extended family and playing an important role in
society bring in some beneficial effects.
B) Yes, finance is badly needed global economy .

C) Yes, small banks are closing down


A) I dont believe dietary habits and lifestyle have
D) Well, China produces most of the worlds products much to do with ageing. Its all about the gradual
failing of the body to be able to repair itself and
E) Yes, India has tried to ban cotton exports recently replace cells.

B) I heard some people live longer and have fewer


66. Teacher :- As you know, many of the animals that health problems than others thanks to their easy-
existed in the past have become extinct; that is, going lifestyle and the amount of vegetables they
they have disappeared from the face of the consume.
earth, mostly as a result of human activities.
Student :- Is it only human activities that have C) Perhaps, calorie restriction and anti-ageing
caused the extinction of animals? treatments can be successful interventions that
Teacher :- ---- may cause increases in life expectancy.
Student :- I think dinosaurs could be an example
of such extinct species. I read in a science D) Even if ageing seems to be a serious problem for
magazine that they became extinct when a huge many people, some rely on plastic surgeries to
meteor hit the earth, millions of years before cope with it.
humans existed..
E) Extending ones lifespan isnt simply a case of
stopping the ageing process, because ageing isnt
A) No. There are other causes that have resulted in
a scientifically recognized cause of death.
extinction. Can you think of any examples?

B) What we do to our planet is more than enough to


cause extinction.

C) Mostly, yes. Just think of the dinosaurs, which


became extinct some 65 million years ago.

D) Of course. Natural events have never caused the


extinction of any species other than the dinosaurs.

E) Indeed, the human contribution to natural disasters


cannot be ignored, as in the case of dinosaurs.
68. - 71.sorularda, verilen cmleye anlamca en
yakn cmleyi bulunuz.

68. The British are not actually more modest than


other nations, but they do have a set of social
rules that include maintaining a front of modesty
at all times..

A) Modesty is maintained by the British through the


observation of strict rules that have always been of
great importance throughout history.

B) Although not necessarily more modest than others,


the social rules they have show that it is important
for British people to appear modest.

C) Besides the fact that most British people are


modest, they also have more social rules than
other nations.

D) Other nations may actually be more modest than


the British despite not having a set of social rules

E) For the British, it is important that they are viewed


as the most modest nation in the world since they
have a set of strict social rules.
69. The view of the city from the top floor of the
hotel is absolutely wonderful, especially at night
when all the lights are on..

A) The view of the city, especially at night when its all


lit up, is well worth a visit to the top floor of the
hotel.

B) You must go up to the top floor of the hotel,


preferably at night, for a magnificent view of the
city and its lights.

C) From the top floor of the hotel you get a truly


splendid view of the city, particularly at night when
its all lit up.

D) Be sure to go to the top floor of the hotel to get a


magnificent view of the city and its lights.

E) After dark when the lights have come on, you get
an unforgettable view of the city from the top floor
of the hotel.

70. Even if one forgets about the dangers involved,


coal-mining cannot be a very attractive way of
earning a living..

A) Its the risks that are involved that really make


coal-mining such an unattractive form of
employment.

B) Working in a coal mine must be rather an


unpleasant form of employment even when one
ignores the risks.

C) Work in a coal mine may still be unpleasant but


there are no more risks than in other forms of
employment.

D) Work in a coal mine is not the only form of


employment that has unpleasant aspects and even
risks.

E) Its hard to imagine anyone enjoying working for a


living in a coal mine even if it is perfectly safe.
71. Anti-nausea drugs are recommended to prevent
72. - 75.sorularda, bo braklan yere, parada
the vomiting that sometimes accompanies
migraines.. anlam btnln salamak iin
getirilebilecek cmleyi bulunuz.

A) A way to stop the vomiting that may occur when


you have a migraine is to use anti-nausea drugs.
72. Although pain is an almost universal experience,
B) Migraines can sometimes be prevented by taking it is remarkably difficult to define. Some define
anti-nausea drugs. pain as perhaps the most universal form of
stress. ----. Still others emphasize the subjective
C) Vomiting is a symptom caused by certain migraine nature of pain..
prevention drugs.

D) If you suffer from vomiting during a migraine, stop A) Chronic pain never has a biological benefit, but life
taking drugs. without pain produces even more problems

E) Anti-nausea drugs can sometimes cause vomiting if B) Pain is sometimes classified by stages, according to
you have a migraine. the duration of pain

C) Others concentrate on the physiology that


underlies the perception of pain

D) A common example of chronic recurrent pain is


headache pain, especially the pain of migraine
headaches

E) Until about 100 years ago, pain was most


frequently considered a direct consequence of
physical injury

73. Imagine you are planning to buy a small car. Two


models stand out: a small car that does not use
much gas, and an expensive sports car. After a
good deal of checking, you decide on the sports
car. ----. You ask yourself whether or not the
more economical type would have been more
suitable..

A) But as soon as you have driven it home, you


wonder if you have done the right thing

B) And the sports car has side air bags and a CD


player

C) After you have bought it you know youve made


the right choice

D) The good features of the chosen car outweigh


those of the other one

E) In order not to regret your decision, you


exaggerate the faults of the other car
74. After earning his degree, John M. Keynes became
a civil servant, taking a job with the India Office
in Whitehall, London. ----. With the onset of
World War I, Keynes returned to government
employment, this time in the Treasury..

A) Between the wars, Keynes wrote his most famous


work: The General Theory of Employment, Interest
and Money

B) After World War I, he attended the Versailles Peace


Conference

C) His book, The Economic Consequences of the


Peace, condemned the Versailles Peace Treaty and
its negotiators

D) His wife, a Russian ballerina, was very active in


promoting the arts

E) After a while, he returned to Cambridge, where he


taught economics at the University
75. Near-death experiences are relatively
stereotyped in format. ----. They often describe
sensations likefloating or flying. These
experiences can also cause fear or a feeling of
detachment, but they are usually described as
intensely real not at all like a dream or a
hallucination..

A) People who have had such experiences say that


they seemed to be no longer in their own body but
outside it, looking down on themselves from two or
three metres above

B) Survivors of these experiences may, weeks later,


experience a surge of emotionality, as if emotions
of every sort were being stimulated or released

C) Sometimes, near-death experiences are followed


by a sudden onset of musical or artistic interest

D) Tony Cicoria reported seeing a bluish-white light


that surrounded him and having a sense of ecstacy

E) Similar near-death experiences have often been


described by people who have been, or believed
themselves to be, in great danger

76. - 80.sorularda, cmleler srasyla


okunduunda parann anlam btnln
bozan cmleyi bulunuz.

76. (I) The easiest way to appreciate the importance


of money is to imagine what economic life would
be without it. (II) We are so accustomed to using
money that we seldom notice the roles it plays in
almost all of our everyday transactions. (III) The
most important function of money is to serve as
a medium of exchange, a generally accepted
means of payment. (IV) Currency and bank
deposits on which checks may be written
certainly qualify as money. (V) In addition to its
primary function, it serves as a unit of account,
that is, as a widely recognized measure of value..

A) I

B) II

C) III

D) IV

E) V
77. (I) Historically, farming societies have had higher
birth-rates than urban societies; their
populations have also tended to be younger. (II)
Marked by a high regard for traditional values,
farming communities were often regulated by
kinship customs and ritual. (III) These were
related: it was to a farmer's advantage to have
many children who could work in the fields and
then would support their parents as they grew
older. (IV) Generally, however, as the children
became older, there was not enough land for all
of them to support their own families, and some
would migrate to the cities. (V) In this way, cities
have absorbed the excess population of the
countryside, thus becoming filled with
comparatively older people living in smaller
families..

A) I

B) II

C) III

D) IV

E) V

78. (I) Stephen Longstreet was a writer of novels and


detective stories. (II) There are many different
forms of art, such as dance, music, and
literature. (III) He was also an accomplished
artist. (IV) While living in Europe in the 1920s, he
became acquainted with some famous artists. (V)
Henri Matisse and Pablo Picasso were among
them..

A) I

B) II

C) III

D) IV

E) V

79. (I) A tragedy is the imitation in dramatic form of


an action that is serious and complete, with
incidents arousing pity and fear with which it
causes a climax of such emotions. (II) The chief
characters in a tragedy are noble, and the
actions they perform are noble actions. (III) The
plot involves a change in the heros fortune, in
which he falls from happiness to misery. (IV)
Unlike tragedy, comedy aims at ridicule and so
satirizes. (V) Such misfortune is brought upon
him not by his bad character but by some error
of judgement..

A) I

B) II

C) III

D) IV

E) V
80. (I) Britains largest robotics laboratory will open
in the autumn. (II) On the contrary, this one is to
be well funded. (III) Two universities are jointly
in charge of the project. (IV) There will be 50
researchers there aiming to develop intelligent,
independent robots. (V) These, it is hoped, will
work in a variety of areas ranging from social
care to industrial cleaning..

A) I

B) II

C) III

D) IV

E) V
SORU CEVAP SORU CEVAP

1 E 41 E

2 A 42 E

3 D 43 D

4 D 44 C

5 B 45 A

6 A 46 E

7 C 47 B

8 E 48 C

9 C 49 A

10 A 50 C

11 A 51 C

12 E 52 A

13 D 53 D

14 B 54 A

15 A 55 A

16 A 56 B

17 E 57 D

18 C 58 E

19 D 59 E

20 B 60 D

21 A 61 A

22 D 62 C

23 D 63 A

24 A 64 D

25 E 65 E

26 C 66 A

27 B 67 B

28 B 68 B

29 B 69 C

30 C 70 B

31 B 71 A

32 B 72 C

33 E 73 A

34 C 74 E

35 C 75 A

36 A 76 D

37 B 77 B

38 A 78 B

39 B 79 D

40 E 80 B
22
6. In the early twentieth century, a number of
1. - 16.sorularda, cmlede bo braklan yerlere
scientists, who had been trained as physicists,
uygun den szck ya da ifadeyi bulunuz. were interested in the study of biological
organisms, and their efforts ---- the field we now
call molecular biology..

1. Schools are one of the most important ---- of


socialization on which peer groups and teachers A) took leave of
have a major impact..
B) made up for

A) degrees C) gave rise to

B) rules D) showed up

C) customs E) fell apart

D) agents
7. The discontent that ---- in many countries at last
E) advances ---- an outlet in the wave of revolutions which
spread across Europe in 1646..

2. The date of Homer was ---- in antiquity, and is no


less so today.. A) was brewing / had found

B) had been brewing / found


A) necessary
C) brewed / will find
B) competent
D) has been brewing / has found
C) controversial
E) would be brewing / was finding
D) valuable

E) prevalent 8. Researchers in the 1890s ---- the atom as a


homogeneous sphere of positive charge inside of
which there ---- tiny negatively charged
3. Expressionist artists in painting, sculpture, and electrons..
literature ---- to distort or exaggerate natural
appearance in order to create a reflection of the
inner world.. A) visualized / were

B) have visualized / had been


A) pretended
C) had visualized / have been
B) expected
D) will visualize / are
C) persuaded
E) were visualizing / would have been
D) tended

E) offered 9. Some climate change is thought to result from


human behaviour, ---- natural changes in the
atmosphere..
4. History is the study of the past, ---- the written
record and oral traditions passed down from
generation to generation verbally.. A) rather than

B) as
A) particularly
C) even if
B) excessively
D) whenever
C) apparently
E) the same as
D) poorly

E) neglectfully 10. Women in Tasmania were given the right to vote


in 1902, ---- , in Switzerland, this right was
obtained only in 1971..
5. In the introduction to this book, the writer ---- a
foreign policy that makes world peace the top
priority.. A) whereas

B) when
A) calls for
C) before
B) holds out
D) hence
C) keeps up
E) unless
D) puts on

E) brings about
11. In Kuwait, foreign companies are presently taxed 16. If you ---- the idea that a particular group of
at 55 per cent of profits, whereas the Kuwaitis people have no right ---- their opinions then you
themselves pay ---- or ---- tax depending on their don't believe in democracy..
status..

A) had accepted / to be expressed


A) less / least
B) accepted / expressing
B) little / no
C) will accept / being expressed
C) more / less
D) accept / to express
D) few / fewer
E) would accept / expressed
E) less / none

12. Two factors that contribute ---- the Eastern


Sierras wildflower diversity are its local habitat
diversity and the fact that it is the meeting point
---- three floristic regions..

A) over / from

B) for / in

C) into / at

D) on / about

E) to / for

13. The changes of energy in nuclear reactions are


enormous ---- comparison ---- those in chemical
reactions..

A) above / over

B) in / for

C) beyond / to

D) by / with

E) of / after

14. The first European mission to another planet is -


--- its way, ---- the delight and relief of
all involved..

A) about / by

B) off / for

C) in / over

D) through / with

E) on / to

15. Lacto-vegetarians do not eat eggs, but they


drink milk and eat milk ---- products ---- butter
and cheese..

A) so many / that

B) such / as

C) the same / as

D) much / than

E) as much / as
21. V.
17. - 21.sorularda, aadaki parada
numaralanm yerlere uygun den szck ya
da ifadeyi bulunuz. A) Since

B) Just as

Itis widely believed that medical science has been C) Unless


gradually and systematicallyreducing civilisation's
diseases. In the past, it was the (I) ---- of ignorance and D) Such
superstition thatresulted in epidemics like the medieval
plagues. However, during the 19th and20th centuries, E) While
scientists finally managed to discover (II) ---- caused
infectious diseases and beganto cure them with drugs and
immunisation. Above all, improved technology and
advances in anaesthesia (III) ---- surgeons to carry out
sophisticatedoperations; penicillin and antibiotics helped
complete the advance. And yet, some people disagree.
They point outthat more credit ought to be given to social
reformers who campaigned for purerwater and better
sewage disposal (IV) ---- livingstandards could be
improved. It is their opinion that the drugs and
antibioticsmerely speeded up the process. (V) ----
thiscontribution was valuable, it did not lower the level of
disease ingeneral.

17. I.

A) statement

B) combination

C) opportunity

D) rivalry

E) reason

18. II.

A) which

B) whether

C) what

D) whom

E) that

19. III.

A) grew

B) advised

C) enabled

D) shaped

E) appeared

20. IV.

A) so that

B) in addition

C) because

D) for

E) in case
26. V.
22. - 26.sorularda, aadaki parada
numaralanm yerlere uygun den szck ya
da ifadeyi bulunuz. A) even so

B) if

Theorigins of baseball probably (I) ---- backto 1839, when C) as


Abner Doubleday, a civil engineering student, laid out a
diamond-shapedfield at Cooperstown, New York and D) although
attempted to standardize the rules governingthe playing
of (II) ---- games as 'townball' and 'four old cats', the E) still
ancestors of baseball. By the end of the Civil War, interest
in thegame (III) ---- rapidly. There wereover 200 teams or
clubs, some of which toured the country playing rivals. 27. - 36.sorularda, verilen cmleyi uygun
Theybelonged (IV) ---- a nationalassociation of ' Baseball ekilde tamamlayan ifadeyi bulunuz.
Players' that had proclaimed a set of standardrules.
These teams were amateurs orsemi-professionals, but (V)
---- thegame waxed in popularity, it offered opportunities
for profit, and the firstprofessional team, the Cincinnati 27. Whatever Luhrmanns New York critics may think
of his new production of La Bohme, ----..
Red Stockings, appeared in 1869. Other cities soon
fielded professional teams,and in 1876 the present
National League was organized. A) people had waited in long queues for last-minute
cancellations

B) they claimed to have pursued creative freedom


22. I.
C) the beautiful side of life will have been expressed
with beautiful music
A) happen
D) he has certainly breathed new life into this opera
B) serve
E) opera used to be characterized by unnatural,
C) affect alienating conventions

D) stretch
28. After Dali was expelled from art school in Madrid
E) allow in 1926, ----..

23. II. A) he had been much influenced by the earlier


Spanish painters

A) same B) a number of paintings depict unusual landscapes

B) some C) especially mentally disturbed people had interested


him enormously
C) as
D) he joined a group of painters who called
D) the themselves \'surrealists\'

E) such E) today his paintings still amaze but also entertain


many people
24. III.
29. While the British painter Turner had a profound
influence on the French Impressionists and the
A) had grown German Expressionists, ----..

B) grows
A) his work remains for the most part little-known by
C) would have grown the world at large

D) were to grow B) in output he would have been the most productive


of all painters
E) has been growing
C) this is because his works are exhibited in a few
galleries in Britain
25. IV.
D) he loved and interpreted certain themes from
classical mythology
A) at
E) another great landscape-painter is Gainsborough
B) to

C) for

D) of

E) in
30. Due to the numerous repairs that The Galata 34. Portugal, an independent state since the 12th
Tower has undergone over time, ----.. century, was a kingdom ----..

A) its appearance has changed A) although the country is crossed by three large
rivers that flow into the Atlantic
B) it was completed in 1349
B) until a revolution in 1910 drove out King Manoel II,
C) originally it was built by the Genoese and a republic was proclaimed

D) it was partly destroyed by fires in the 16th century C) which occupies the western part of the Iberian
Peninsula
E) it has from time to time been the scene of
interesting events D) so that the Roman Empire conquered the region in
about 140 B.C.

31. In spite of the fact that some industries are far E) since the Portuguese Empire in the 16th century
ahead of others in their use of information extended to Africa and South America
technology, ----..

35. ----, London has decided to transform its poorest


A) managers can gain a keener insight into whether neighbourhood into a display of what the
their firms are ready for strategic information Olympic Games can mean beyond medals..
systems

B) there are those that need to develop much more in A) Since the athletes are competing to win gold, silver
this area and bronze medals

C) such changes often involve both internal and B) Because many people question the value of hosting
external blurring of some organizational boundaries the Olympic Games

D) information systems are too important to be left to C) Whenever the idea of hosting the Olympic Games
a small technical group emerges

E) there is a connection between the strategy of an D) Though the political support would be dependent
organization and its internal structure on being chosen to host the Olympic Games

E) If hosting international events is considered to be a


32. Disorders that affect joints and their valuable experience
components, such as muscles, bones, and
tendons, are called 'connective tissue diseases' -
---.. 36. In general, copyright law takes the view that
computer programs are not patentable ----..

A) when a disease affects a specific tissue or organ


A) when an antivirus program is recommended for
B) since, in rheumatoid arthritis, chronic inflammation safe computer use
damages the joints cartilage
B) if checking e-mails on a daily basis has become a
C) in case the symptoms of one disease overlap with habit
another
C) now that lengthy court battles benefit neither party
D) because these structures contain large amounts of
connective tissue D) even though some programs have simply
undergone a digital transformation
E) although unusual antibodies can be detected and
measured in the blood E) unless they are genuine innovations with industrial
applications

33. ----, there remain tremendous development


needs in the region, resulting from poverty and
political instability..

A) While the pace of progress in Latin America over


the past two decades has been impressive

B) Since most Latin American countries maintain


closer relations with the United States

C) Just as Europe and Latin America share historic and


cultural ties stretching back over 500 years

D) Before the European Union became the largest


foreign investor in Latin America

E) So long as the European Investment Bank provides


loans to a number of Latin American countries
39. The probability of natural disasters and adverse
37. - 42.sorularda, verilen ngilizce cmleye
climatic conditions such as hurricanes, floods or
anlamca en yakn Trke cmleyi, Trke freezing weather conditions make it riskier for
cmleye anlamca en yakn ngilizce cmleyi global companies to invest in some areas rather
bulunuz. than in others..

A) Tayfun, sel veya dondurucu hava gibi kt hava


koullar ve doal afetler yznden risk almak
37. Charles Kingsley, Su Bebeklerini en kk
istemeyen global irketler, baz blgelere yatrm
ocuu iin, ona doruyu, merhameti, adaleti ve
yapmaktan dierlerine nazaran daha ok kanrlar.
aslnda btn soylu nitelikleri sevmeyi retmek
umuduyla yazd..
B) Baz blgeler tayfun, sel veya dondurucu hava gibi
kt hava koullar ve doal afetler nedeniyle
A) The Water Babies was written by Charles Kingsley global irketlerin yatrm yapmas ynnden
in an effort to help young children admire such dierlerinden daha riskli olabilir.
noble qualities as truth, mercy and justice.
C) Global irketler tayfun, sel veya dondurucu souk
B) The Water Babies by Charles Kingsley aims to gibi kt hava koullar ve doal afetler bulunan
teach young children to love truth, mercy and blgeleri yatrm yapmak iin dier blgelerden
justice as well as other noble qualities. daha riskli bulabilir.

C) The Water Babies is by Charles Kingsley, and he D) Global irketlerin yatrm yapma olasl olan baz
wrote it to teach his youngest child about such blgeler, tayfun, sel veya dondurucu hava gibi kt
noble qualities as truth, mercy and justice. hava koullar ve doal afetler nedeniyle
dierlerinden daha riskli olabilir.
D) Charles Kingsley wrote The Water Babies for his
youngest child, hoping to teach him to love truth, E) Tayfun, sel veya dondurucu hava gibi kt hava
mercy and justice and indeed all noble qualities. koullar ve doal afetler olasl, global irketler
iin baz blgelere yatrm yapmay dierlerinden
E) Truth, mercy and justice are just some of the noble daha riskli hle getirir.
qualities Charles Kingsley hoped to teach his
youngest child through The Water Babies.
40. Some countries have stopped the import of milk,
vegetable and fruit from areas near the
38. lk Dnya Kupas 1930da Monte videoda earthquake-damaged nuclear power plant in
yapld, ancak byle bir yarma kavram, Japan..
1920de Antwerpte yaplan bir FIFA
kongresinde domutu..
A) Baz lkeler Japonyada depremden zarar gren
nkleer santral yaknndaki blgelerden st, sebze
A) It wasnt until 1930 that the first World Cup was ve meyve ithalatn durdurdu.
held in Montevideo, but the concept of such a world
competition had been born at a FIFA meeting in B) Baz lkelerin, Japonya'da depremden zarar gren
Antwerp in 1920. nkleer santral yaknndaki blgelerden st, sebze
ve meyve ithalatn durduraca syleniyor.
B) It was in Montevideo in 1930 that the first World
Cup was held, but the possibility of such a C) Baz lkelerde st, sebze ve meyve ithalat, bunlar
competition had been discussed at a FIFA congress Japonyada depremden zarar gren nkleer santral
in Antwerp in 1920. yaknndan getirildii iin durduruldu.

C) In 1920, at a FIFA congress in Antwerp, the concept D) Baz lkeler Japonya'da depremden zarar gren
of such a competition came into being and resulted nkleer santral yaknndaki blgelerde retilen st,
in the first World Cup in 1930 in Montevideo. sebze ve meyvelerin ithalatna snrlama getirdi.

D) The first World Cup was held in Montevideo in E) Baz lkeler Japonyadan gelen st, sebze ve
1930, but the need for a competition of this kind meyvelerin, yurda sokulmasn nkleer santral
had been recognized at the FIFA congress in yaknnda retildii gerekesiyle durdurdu.
Antwerp in 1920.

E) The first World Cup was held in Montevideo in


1930, but the concept of such a competition had
been born at a FIFA congress held in Antwerp in
1920.
41. Even if domestically produced goods are more
expensive than imports, purchases carried out by
leading firms should be directed towards them..

A) nde gelen firmalarn almlarnda pahal ithal


mallar semek yerine yerli mallara younlamalar
gerekmektedir.

B) nc irketlerce satlan mallar, ithal mallardan


daha pahal olsalar bile byk lde yerli
mallardan olumaldr.

C) thal mallar daha pahal olduunda, tannm


irketler tarafndan gerekletirilen almlar yerli
mallara doru ynlendirilmelidir.

D) Yerli mallar ithal mallardan daha pahal olsalar bile,


nde gelen irketler tarafndan gerekletirilen
almlar bunlara ynlendirilmelidir.

E) Tannan irketler tarafndan gerekletirilen


almlarda yerli mallar daha pahal olsa bile ithal
mallara gre daha ok tercih edilirler.
42. In addition to traffic fatalities, alcohol use has
been implicated in many other deaths among
young people, including drownings, falls,
suicides, and homicides. .

A) Genler arasnda alkol kullanm, trafik


lmlerinden baka, boulmalar, dmeler,
intiharlar ve cinayetler gibi dier lm trlerinin de
nedeni olmutur

B) Genler arasndaki alkol kullanm, trafik lmlerinin


yan sra, boulmalar, dmeler, intiharlar ve
cinayetler gibi dier pek ok eit lme yol
aabilmektedir

C) Alkol kullanm, genler arasnda, trafik lmlerine


ek olarak, boulmalar, dmeler, intiharlar ve
cinayetler dahil dier birok lmn nedeni
olmutur

D) Trafik lmlerinden ayr olarak, boulmalar,


dmeler, intiharlar ve cinayetler, alkol kullanm
nedeniyle genler arasnda yaygn olan dier lm
biimleridir

E) Alkol kullanm nedeniyle, trafik lmlerinden ayr


olarak, genler arasnda yaygn olan dier lm
biimleri, boulmalar, dmeler, intiharlar ve
cinayetlerdir
45. It is clear from the passage that ----..
43. - 46.sorular aadaki paraya gre
cevaplaynz.
A) Vitamin D is absorbed only by 7-
dehydrocholesterol
A deficiency in Vitamin D is known to cause various B) not all forms of Vitamin D require sunlight to
diseases due to insufficient calcium or phosphate in the synthesize
bones. Vitamin D is actually an umbrella term that covers
a group of steroid molecules. Of these, only Vitamin D3 C) some mammals cannot synthesize Vitamin D
requires sunlight to synthesize. It is formed in the skin of
all mammals when light energyis absorbed by a precursor D) steroid molecules need Vitamin D3 to function
molecule called 7-dehydrocholesterol. A recent study properly
found that at theheight of summer, two minutes exposure
E) Vitamin D3 production is increased by a factor of
of the faceand arms to the sun, three to four times a six in the summer
week, could supply enough Vitamin D3. This rose to 15
minutes in the winter. Ironically, Vitamin D deficiency,
which may also lead to skin cancer, is actually very
common in some of the sunniest but most under
developed countries in the world. This is notbecause of
malnutrition or a lack of dietary supplements, as most
people mistakenly think. Infact, it is because very dark
skin colour slows the rateof Vitamin D3 production by a
factor of six, and people in these countries are usually
heavily veiled when outside, as well. In addition, wearing
sunscreen with a sun protection factor greater than eight
willalso block Vitamin D3 production.

43. According to the passage, the high rate of


Vitamin D3 deficiency in under developed
countries with plenty of sunshine ----..

A) may be prevented through the use of high-factor


sun protection

B) is mainly due to malnutrition

C) is due to the fact that people are dark coloured and


dress heavily

D) can easily be blocked with dietary supplements

E) results in the formation of steroid molecules

44. It can be understood from the passage that


Vitamin D ----..

A) is found in sunscreens with protection factor


greater than eight

B) is synthesized at a higher rate in people with


darker skin colour

C) has to be supplemented in the diet for the health of


the bones

D) might cause cancer when consumed in excessive


amounts

E) provides protection from both skeletal diseases and


skin cancer
46. One can understand from the passage that ----..

A) exposure to the sun in summer for even two


minutes can cause skin cancer

B) one should wear sunscreen in very sunny areas to


facilitate Vitamin D synthesis

C) the human body can synthesize Vitamin D3 all year


round, including winter

D) steroid molecules function better when combined


with dietary supplements

E) lack of dietary supplements is a major cause of


skeletal diseases
50. It is clear from the passage that the city of
47. - 50.sorular aadaki paraya gre
Hierapolis became wealthier under Byzantine
cevaplaynz. rule ----..

A) through the contributions of every facet of the


The ruins of Hierapolis, a once-grand city near modern society there
Denizli in Turkey, brilliantly bring into being a picture of
life in the early centuries of the modern era.It was here B) because of the miracles of the Plutonium spring
that the mix of pagan, Roman, Jewis hand early Christian
elements evolved into a distinctly Anatolian environment. C) although this wealth was mostly centered around
Founded around 190 B.C. by Eumenes II, King of the early Christian population
Pergamum, Hierapolis was a cure centre that grew
D) than it had been under Roman rule
wealthy under the Romans and even more so under the
Byzantines. It had a large Jewish community and an early E) especially due to its large Jewish community
Christian church.Frequent earthquakes regularly brought
disaster;after the one in 1334, the locals finally gave up
and moved away. Near the Hierapolis Archaeology
Museum stand the ruined foundations of a Temple of
Apollo. This temple had a spring called the Plutonium
dedicated to Pluto, Roman god of the underworld.The
spring gives off toxic vapours, and the temple priests
would demonstrate its powers to visitors by throwing in
small animals and birds and watching them die.

47. It can be inferred from the passage that,


historically, visitors used to come to Hierapolis in
order to ----..

A) seek treatment for their illnesses

B) request aid from Eumenes II of Pergamum

C) throw animals into the Plutonium spring

D) become rich by doing business there

E) get a taste of the distinctly Anatolian environment

48. It is pointed out in the passage that, by


visitingthe ruins of Hierapolis, people ----..

A) can see a statue of King Eumenes II of Pergamum


near the Archaeology Museum

B) can be cured of all their diseases

C) can understand what life there must have been like


in the past

D) can understand the reasons why its people


deserted the place

E) can see the well-preserved Temple of Apollo

49. We see from the passage that Hierapolis was


abandoned in the 14th century due to ----..

A) the poisonous gases given off by the Plutonium


spring

B) the earthquakes that kept recurring in the area

C) the unending conflicts that took place among the


peoples of different religions living there

D) the rise of the modern city of Denizli nearby

E) invasions by different foreign forces


54. We can understand from the passage that
51. - 54.sorular aadaki paraya gre
consumers who are willing to wait before buying
cevaplaynz. a product ----..

A) are willing to pay a lot of money for the product


Most books are published in two forms hardback and
paperback the paperback edition being published and B) must generally settle for a lesser-quality product
available for sale several weeks or months later than the
hardback edition. The cost of producing a hardback book C) may not be able to find the exact product they
is only about 20% higher than the cost of producing the want to buy
same book as a paperback, but the price of a hardback
book is about three times the price of a paperback book. D) will be able to choose from a greater selection of
products
Why? The key to solving this puzzle is the fact that
hardback books are published first, followed by the E) will usually have the chance to pay a lower price
paperback edition. Booksellers use hardbacks and
paperbacks to distinguish between two types of
consumers: those who are willing to pay a lot and those
who are willing to pay a little. The people who are willing
to pay the most are eager to read the book as soon as it
comes out, so they pay $24 for a hardback book. The
people who are willing to pay less are more patient and
are willing to wait a few months for the $8 paperback
version. The pricing of hardback and paperback books is
an example of price discrimination, in which certain
people are prepared to pay a higher price for what they
want.

51. By the words this puzzle in the text, the


authoris referring to the fact that ----..

A) paperback books are published later than hardback


books

B) booksellers charge the same price for paperback


books as hardback books

C) the prices of hardback books are excessively high


in relation to the cost of their production

D) some people are willing to pay a higher price for


their books

E) there are basically two types of consumers

52. We understand from the passage that some


consumers buy hardback books because ----..

A) they do not want to wait for the paperback edition


to come out

B) they think hardback books have better-quality


materials

C) booksellers use extensive advertising to sell them

D) they have more money to spend

E) their cost of production is not much higher than


that of paperback books

53. We understand from the passage that the cost of


producing a hardback book ----..

A) is reflected in the sale price

B) is constantly rising

C) depends largely on its length

D) exceeds that of a paper book by 20%

E) means that very few people can afford them


57. It is suggested in the passage that the first
55. - 58.sorular aadaki paraya gre
majorform of genetic therapy raises problems
cevaplaynz. because ----..

A) using technology in this process impairs the


Gene replacement therapy is being developed forseveral organic nature of the gene
genetic diseases. Because many difficulties are inherent in
treating most serious genetic diseases, scientists have B) it is difficult to apply the normal gene into a
dreamed of developing actual cures. Today, genetic fertilized egg
engineering is bringing these dreams closer to reality.
Such therapy could take two main forms. One approach C) it is ethically controversial
would be to introduce copies of a normal gene into a
D) the clinical tests have produced some unexpected
fertilized egg, using modifications of the technology
results
already used to produce transgenic animals. In some
transgenic animals the introduced gene can remain stable E) its application to transgenic animals has been
from generation to generation, constituting atrue 'genetic opposed by some researchers
cure.' However, this approach raises such complex ethical
problems that it is not being actively pursued at this time.
A second strategy to introduce the normal gene into only
some body cells (somatic cell gene therapy) is receiving
increased attention today. The rationale is that, although
a particular gene may be present in all cells, it is
expressed only in some. Expression of the normalallele in
only the cells that require it may be sufficient to give a
normal pheno type. Although this approach presents a
number of technical obstacles, which must be overcome,
gene therapies for a number of genetic diseases are
undergoing development or are being tested on patients
in clinical trials.

55. According to the passage, somatic cell


genetherapy ----..

A) is regarded by physicians ethically applicable

B) is cheaper and more practical

C) leads to a number of complications

D) has been practised over decades

E) is related to body cells

56. One understands from the passage that


clinicaltrials for gene therapies ----..

A) have given little hope

B) are still underway

C) have been much costly

D) cannot be carried on due to ethical problems

E) have been going on for several decades


58. According to the passage, gene
replacementtherapy ----..

A) is a cure under development, especially for


hereditary genetic diseases

B) is a dream in the medical world, which can hardly


be achieved

C) is fully beneficial for a variety of diseases

D) has already been tried and very good results


obtained

E) has been used by the physicians successfully for a


long time
62. It is clear from the passage that the Mosquito -
59. - 62.sorular aadaki paraya gre
---..
cevaplaynz.

A) did not achieve the effect that the owners of the


theatre had hoped for
For humans to be able to hear a sound, it must be both
loud enough and within the right frequency range as B) creates a sound that many people find deeply
measured by the number of vibrations persecond, or hertz disturbing
(Hz). The average person is mostsensitive to sounds in the
1, 000-5, 000 Hz range, and most lose the ability to hear C) produces a high-frequency sound that only
very high frequencies(above around 20, 000 Hz) with age. teenagers can hear
Even so asizeable proportion of the population do seem
D) only had a disturbing effect upon the elderly
toremain sensitive to the very low frequency 'infrasound'.
High-frequency sounds have more thanjust audible effects E) was regarded as offensive by young and old alike
as teenagers in Swindon discovered in 2006. Tired of
having crowds of youngsters collecting around the town
theatre, the owners installed the Mosquito, a device that 63. - 67.sorularda, karlkl konumann bo
emits sonic energy at very high frequencies. Only the braklan ksmn tamamlayabilecek ifadeyi
teenagers could hear it and it forced them to meet bulunuz.
elsewhere.

63. Robert :- I read an article about alternative


59. According to the passage, the average
energy sources. According to this article, wind
humanbeing ----..
power is the fastest-growing source.
Sarah :- ----
Robert :- Why is that? Its such a clean power
A) is only sensitive to sounds within a very narrow
source.
range of frequency
Sarah :- Some say that wind turbines are noisy
B) is exposed to high frequencies and this may cause and a threat to birds and bats..
his hearing loss
A) Really? As far as I know, there is a lot of opposition
C) only hears sounds within a given frequency range
to it.
D) can develop his own range of sound frequency
B) Some critics say the more different our energy
sources are, the better.
E) can hear sounds that are loud enough no matter
what their frequency is
C) Most developed countries prefer wind power to
other energy sources.
60. We understand from the passage that elderly
people ----.. D) I dont think that wind power is an economical
alternative.

A) often lose the ability to hear very high-frequency E) Scientists suggest that the use of alternative
sounds sources will increase a lot.

B) are unable to hear low-frequency sounds


64. James: ----
C) often prefer not to hear what is going on around Robin: What happened? Didnt they say they
them needed you for the new graduate programme for
the next semester?
D) usually wont admit that their hearing has become James: True. I talked about it with my wife, but
impaired she just cant dare to move to another city and
start from zero.
E) can hear the sonic sounds at very high frequencies Robin: I think she is absolutely right, as you
with the help of the Mosquito currently hold the same position here in
Glasgow..

61. It is clear from the passage that age ----..


A) The more applicants theyve got for the
programme, the more teaching staff theyll need in
A) enables us to hear sounds of a lower frequency the coming years.

B) has a considerable effect on what we hear B) The department head held a meeting with faculty
members and said they wouldnt be accepting new
C) has little effect on our hearing graduate students.

D) makes us irritable on exposure to infrasound C) Sooner or later, I will resign from my post in the
department and apply to the university for a
E) does not necessarily cause deterioration of our teaching position.
sense of hearing
D) Well, I suppose new faculty members will be hired
because weve got more applicants this year than
expected.

E) You know, I was intending to apply for an associate


professorship post at another university. I decided
not to do so
65. Peter:- Are you going to join one of the further
68. - 71.sorularda, verilen cmleye anlamca en
training schemes the bank is offering?
James:- ---- yakn cmleyi bulunuz.
Peter:- Yes; you really ought to. But at least
explain the situation to the manager.
James:- Good idea. Ill do that..
68. While the country is prepared for rapid
development, the President knows that no
A) No; Im content with things as they are. amount of economic growth can ever
compensate for the degradation of the countrys
B) No; I like to keep my evenings free. natural resources and rich biodiversity..

C) Im not sure. Ill think it over.


A) Because of the exhaustion of the countrys natural
D) No, Im not. Things are difficult at home as you resources and rich biodiversity, the President was
know. But I know I ought to. uncertain whether to continue or halt the rapid
development of the country.
E) Perhaps. Im still thinking about it. What about you?
B) Since the country is about to develop quickly, the
President is ready to make use of the countrys
66. Timur : - Do you know the difference between the natural resources and abundant biodiversity and he
use of barbecues and conventional gas cooking?
will compensate the people for their loss.
Levent : - ----
Timur : - What can be done to reduce this? C) The President thinks that no economic growth can
Levent : - I think we should use gas over substitute for the loss of the countrys natural
barbecue as it contributes to an increase in smog resources because rapid development would mean
levels.. that the destruction of the country is finishing them
off.
A) Burning charcoal releases carbon monoxide into
D) Although the country seems ready to develop, the
the air much more than cooking with gas does.
President is aware that no economic growth could
make up for the deterioration of the countrys
B) Well, actually, both are good ways of cooking, but
natural resources and abundant biodiversity.
barbecues take longer to prepare.
E) The country is ready for rapid development;
C) Gas cooking is far more efficient than using a
however, the government does not realize that
barbecue, as you can adjust the heat according to
abundant resources and biodiversity could be lost
your needs.
as a result of this economic growth.
D) Unfortunately, both are harmful and cause damage
to the environment. 69. Get Jim to give the speech of welcome; he's
quite the best person for the job..
E) Gas appears to be more dangerous as there is a
high risk of explosion.
A) Try to persuade Jim to give the speech of welcome:
he\'s good at such things.
67. Teacher : - Your son has adapted quite well
socially. Hes had no problems making friends. B) Jim will give a far better speech of welcome than
Also, hes quite a leader among them.
anyone else would get him to do it.
Parent : - Im glad to hear that. What about his
class work? C) As he\'s good at things like that, why don\'t you get
Teacher : - ---- Jim to give the speech of welcome?
Parent : - He's never been very good at sitting
still and focusing. D) One person who is good at speechmaking is Jim;
. ask him to do it.

E) Jim will make as good a speech of welcome as


A) His math skills are very good, but he needs to work
anyone l know; see if he\'ll agree to do it.
harder on his language skills.

B) He enjoys group work, probably because he likes to


socialize so much.

C) I think he needs a private tutor to help him with the


more difficult subjects.

D) I really enjoy having him in class because hes such


a good example to others.

E) He has the ability but he seems to lack the


concentration to do the work.
70. Not only the general populace, but also those
who work in the field of economics find it difficult
to understand the differences and the
relationships between theory and practice..

A) The general population, including those working in


the field of economics, can understand the theory,
but cannot grasp the practice of economics.

B) Understanding the differences and the relationships


between theory and practice in economics is
challenging for those who do not specialize in
economics.

C) Both the general public and the people working in


the area of economics fail to fully comprehend how
theory and practice differ from and relate to each
other.

D) Ordinary people have difficulty in understanding


the theory and practice of economics but the
people working in the field can see them clearly.

E) Specialists understand the differences between


theory and practice in the field of economics
because they can see how they affect the general
populace.
71. Social behavior depends very much on
72. - 75.sorularda, bo braklan yere, parada
the information we collect about other people..
anlam btnln salamak iin
getirilebilecek cmleyi bulunuz.
A) Our social behavior reflects to some extent what
we feel about the people around us.

B) The conduct of the people around us dictates our 72. Historically the Turkish electricity sector has
own social behavior. been dominated by state-owned enterprises that
provide distribution, generation, trading and
C) Our knowledge of others has a considerable effect transmission services. However, privatization
upon our social behavior. has been widespread for some time. Privately-
owned firms have entered the sector through
D) Our attitude towards the people we come into build operate-transfer schemes. ----.
contact with is naturally reflected in our social
behavior.
A) They distribute the goods they produce to various
E) As we collect information about other people the regions.
way we behave towards them may alter radically.
B) They account for about 21% of electricity
generation.

C) The home consumers of private electricity pay


higher bills than industrial consumers.

D) The government is unwilling to permit private firms


to enter the industry.

E) The government is doing its best to make the gas


sector more efficient.

73. The dengue fever virus can mutate much more


rapidly than anyone thought, which could hold
up the efforts of the teams rushing to create a
much needed vaccine. ----. Indeed, two-fifths of
the world's population are now at risk, and
global warming will allow the virus to increase its
range..

A) In fact, the virus killed only 12,000 people last year

B) Apparently, the new vaccines that are being


developed are genetically very narrow and the
virus is capable of rapid change

C) This mosquito-borne virus has spread dramatically


and now infects about 50 million people each year
in tropical areas

D) So far there is no effective vaccine that protects


against all sorts of virus

E) So one solution might be to make annual changes


to the make-up of any dengue vaccine
74. The two masks of drama one with the corners
76. - 80.sorularda, cmleler srasyla
of its mouth turned down, the other with the
corners of its mouth turned up are familiar okunduunda parann anlam btnln
everywhere. ---- Indeed, just as life gravitates bozan cmleyi bulunuz.
between tears and laughter, they seem to imply
that all drama is divided between tragedy and
comedy..
76. (I) Like all medicines, sleep medicines have side
effects. (II) On the other hand, daytime
A) But drama is an ancient literary form. drowsiness can be best avoided by taking the
lowest dose possible. (III) The most common of
B) Polonius in Hamlet says of a visiting troupe of these effects are dizziness, light headedness,
players that they act either tragedy, comedy, and difficulty with coordination. (IV) Sleep
history, pastoral, pastoral-comical, or poem medicines can also make one sleepy during the
unlimited. day. (V) How drowsy one feels depends upon
how ones body reacts to the medicine..
C) In its development from the beginnings to the
present, it has produced a rich variety of plays.
A) I
D) Can we be sure that it would accurately classify all
plays and would not demand a totally new B) II
category?
C) III
E) Derived from masks actually worn by the actors in
ancient Greek plays, they symbolize the two D) IV
principal modes of drama.
E) V
75. China is an easy target to blame for the climate
crisis. In the midst of its industrial revolution,
China has overtaken the United States as the
worlds biggest carbon dioxide producer. And
everyone has read about the one-a-week pace of
power plant construction there. But those
numbers are misleading, and not just because a
lot of that carbon dioxide was emitted to build
products for the West to consume. ----.

A) This is because the United States is only concerned


about its own gross domestic product.

B) Each Chinese person is individually responsible for


the carbon dioxide emission.

C) Because of Chinas high development rate, more


carbon dioxide is emitted than ever.

D) But also because China has four times the


population of the United States.

E) The Chinese are as responsible for global warming


as the Americans.
77. (I) The Industrial Revolution means the sudden
acceleration of technical and economic
development that began in Britain. (II)
Birmingham is an industrial city and the
administrative headquarters of central England.
(III) It is a major manufacturing, engineering,
commercial and service centre. (IV) The citys
concert halls, theatres and three universities
also make it an important cultural and
educational centre. (V) Its main products are
cars, machine tools and electrical equipment..

A) I

B) II

C) III

D) IV

E) V

78. (I) A dramatic cut in the cost of a super-efficient


new breed of solar cell could put domestic solar
power on a more economic footing. (II) The cells,
which helped take NASAs electric-powered
aircraft Helios to record altitudes, have until now
been too expensive. (III) But their manufacturer
has found a way to make them as much as 20
times cheaper. (IV) The cells convert light energy
into electricity with an efficiency of 20 per cent
which means they generate one-third more
electrical power than conventional silicon solar
cells. (V) NASAs electrically powered plane
Helios so ared to altitudes above 96, 000 feet (29
kilometres) a world record for a winged plane
not powered by a rocket engine..

A) I

B) II

C) III

D) IV

E) V
79. (I) Jean Piaget is the psychologist whose work 80. (I) The global ocean is a huge body of salt water
has had the greatest influence on the study of that surrounds the continents and covers almost
child development. (II) Among the first to accept three fourths of Earths surface. (II) It is a single,
Piagets theories were educators, who began to continuous body of water. (III) Not surprisingly,
develop school curricula based on his ideas. (III) aquatic life zones are different in almost all
During the 1940s and 1950s American child respects from terrestrial ecosystems. (IV) Yet,
psychology turned toward environmental and geographers have divided it into four sections
conditioning methods. (IV) Hes different from (the Pasific, Atlantic, Indian, and Arctic oceans),
any other psychologist in that his theoretical separated by the continents. (V) The Pasific
views are still widely accepted in some form by Ocean is the largest by far: it covers one-third of
many of todays developmental psychologists. Earths surface and contains more than half of
(V) Further Piagets many books on child Earths water..
psychology remain the greatest contribution to
the field by a single scholar..
A) I

A) I B) II

B) II C) III

C) III D) IV

D) IV E) V

E) V
SORU CEVAP SORU CEVAP

1 D 41 D

2 C 42 C

3 D 43 C

4 A 44 E

5 A 45 B

6 C 46 C

7 B 47 A

8 A 48 C

9 A 49 B

10 A 50 D

11 B 51 C

12 E 52 A

13 D 53 D

14 E 54 E

15 B 55 E

16 D 56 B

17 B 57 C

18 C 58 A

19 C 59 C

20 A 60 A

21 E 61 B

22 D 62 C

23 E 63 A

24 A 64 E

25 B 65 D

26 C 66 A

27 D 67 E

28 D 68 D

29 A 69 B

30 A 70 C

31 B 71 C

32 D 72 B

33 A 73 C

34 B 74 E

35 B 75 D

36 E 76 B

37 D 77 A

38 E 78 E

39 E 79 C

40 A 80 C
23
5. I dont know how we are going to ---- the
1. - 16.sorularda, cmlede bo braklan yerlere
expected surge of immigrants into the country..
uygun den szck ya da ifadeyi bulunuz.

A) look over

1. Pain is a highly subjective ---- that no two people B) run through


experience in exactly the same way..
C) put out

A) regulation D) take down

B) sensation E) cope with

C) reinforcement

D) coincidence

E) inquiry

2. Ecosystems are being transformed, and, in some


cases, irreversibly degraded, a large number of
species have gone ---- in recent history..

A) domestic

B) sufficient

C) extinct

D) widespread

E) unique

3. Anaesthetics are drugs given to patients before


undergoing surgery to ---- the sense of feeling
either in a localized area or across the whole
body..

A) attract

B) reflect

C) reduce

D) approve

E) evaluate

4. Although the arch is usually remembered as a


Roman invention, examples have been found
earlier, ---- in the Egyptian civilization..

A) effectively

B) fairly

C) likely

D) nearly

E) particularly
6. According to the World Banks latest figures from
2005, of 196 countries around the world, 131 ----
food imports to feed their citizens..

A) keep up

B) rely on

C) drop out

D) bring about

E) run out

7. When I ---- in to his Office at 11 oclock, he ----


his coffee..

A) will go / will have

B) go / has had

C) was going / has

D) went / was having

E) have gone / would be having

8. Everyone was surprised to see us because we ----


to return before the middle of the moth..

A) are not expected

B) have not expected

C) were not expected

D) are not expecting

E) have not been expecting

9. A running tap wastes up to nine litres of water


per minute, ---- turning it off while brushing your
teeth helps save plenty of water..

A) when

B) since

C) so

D) instead

E) otherwise
10. When walking in a group, the walking pace
should be that of the slowest member ---- he or
she does not feel left behind..

A) in case

B) so that

C) when

D) even if

E) provided that
11. With its excellent beaches ---- fascinating 15. ---- affecting a large number of children and
historical buildings, Malta is one of the most women in developing countries, iron deficiency is
attractive destinations in the Mediterranean.. the only nutrient deficiency which is also
significantly prevalent in industrialized
countries..
A) as well as

B) rather more A) As well as

C) so much B) As if

D) each other C) Much as

E) provided that D) With regard to

E) Due to
12. Scientific research ---- extremely high, especially
if it ---- the use of expensive equipment..

A) may cost / could be involved

B) costs / is involved

C) has cost / will involve

D) can cost / involves

E) may have cost / has involved

13. Advances ---- the understanding of


brain structure have placed novel treatments
for brain disorders ---- reach..

A) with / on

B) of / at

C) in / within

D) through / in

E) on / off

14. ---- the same time as modern humans pushed


into Europe, some of the same group that had
paused ---- the Middle East spread east into
Central Asia..

A) Of / for

B) About / in

C) With / at

D) During / to

E) On / through
16. Psychologists and educators recognize ---- new
learning can profit from old learning because
learning one thing helps in learning something
else..

A) what

B) that

C) how

D) who

E) which
21. V.
17. - 21.sorularda, aadaki parada
numaralanm yerlere uygun den szck ya
da ifadeyi bulunuz. A) many

B) more

Balloonshave recently been used in a new medical C) little


procedure, known (I) ---- balloon valvuloplasty, to open
upstiffened heart valves in aging adults. The four valves in D) such
the heart keep theblood going in the right direction, and
(II) ---- thevalves become stiff, the result can be deadly. In E) much
this procedure, a smallballoon is inserted (III) ---- theheart
and then inflated with a saline solution (IV)---- up to
fortyseconds. The inflated balloon will split the valves
apart, allowing them toopen and close (V) ---- freely.

17. I.

A) with

B) for

C) by

D) as

E) to

18. II.

A) if

B) besides

C) though

D) thus

E) even if

19. III.

A) towards

B) through

C) with

D) from

E) into

20. IV.

A) at

B) about

C) for

D) to

E) on
26. V.
22. - 26.sorularda, aadaki parada
numaralanm yerlere uygun den szck ya
da ifadeyi bulunuz. A) in

B) by

Writersof advertisements are amateur C) during


psychologists. They know just what will (I) ---- toour
instincts and emotions. In general,there are three major D) to
areas in our nature (II)---- advertising aims preservation,
pride and pleasure. Preservation, (III)----, relates to our E) onto
innate desire to live longer, know more, andlook better
than our forefathers did. Pride encompasses all sorts of
things - our desire to (IV) ----, to brag about our prosperity 27. - 36.sorularda, verilen cmleyi uygun
or ourgood taste, to be one of the elite. Weall want to ekilde tamamlayan ifadeyi bulunuz.
enjoy the fruits of our labours, and this is where the
pleasureprinciple comes in. We want to beentertained, to
eat and drink well, and to relax (V)---- comfortable
surroundings. 27. Thanks to the development of the Internet and e-
commerce, ----..

A) the enormousness of such a task should be


22. I. examined in light of rapid changes in technology

B) certain service activities are increasingly


A) persuade outsourced from independent service suppliers
B) appeal C) it is crucial to distinguish between sourcing on a
contractual basis and sourcing on an intrafirm
C) insist
basis
D) attract
D) the advantages to be gained by licensing depend
on the technology, firm size, product maturity and
E) insert
extent of a firms experience

23. II. E) trade barriers keep international markets


separated and permit the affiliate to operate
profitably
A) at which

B) that 28. Just because global warming has come to


dominate conversations about the environment,
C) who ----..

D) when
A) it does not mean that industrial pollution has
disappeared
E) which
B) Japan has passed a law that will reduce industrial
24. III. pollution

C) the US has decided not to introduce technological


A) nonetheless changes

B) afterwards D) global pollution problems are becoming more


serious
C) moreover
E) developing nations such as Turkey have been
D) in the mean time negatively affected

E) for example
29. When the four-thousand-year-old tomb was
finally opened, ----. .
25. IV.

A) the archaeologists had looked down into it in


A) keep off disbelief

B) fall out B) there seems to be nothing in it of any interest at all

C) take up C) they have all looked at each other in amazement

D) show off D) the most exciting find was a set of surgical


instruments
E) hand down
E) it would be a moment of unbearable suspense
30. ---- that experiment and reason became the basis 34. A 10% increase in the cost of hospital services
of scientific knowledge.. would cause poorer households to cut back their
hospital care by 4.7%, ----..

A) It was referred to Galileo


A) in spite of the fact that many developing nations
B) Galileo attacked the much admired teachings of subsidize medical care by about 5%
Aristotle
B) thus the difference between poor and wealthy
C) In Galileos time a great many people were households is even larger
involved
C) whereas the wealthy would have to do the same by
D) It is largely due to Galileo and his discoveries only 2.9%

E) A number of theories concerning the universe were D) because the higher price of medical care would not
proved wrong by Galileo affect the wealthy at all

E) but the same pattern occurs in the demand for


31. ---- as to why human mental capacities are so medical services in poorer households
much greater than those of chimpanzees..

A) Two scientists recently reported

B) Scientists have always suspected

C) Researchers have found a clue

D) The newly adopted scheme also includes a third


category

E) Such a decision was reached after days of debate

32. A patient presenting with such symptoms should


be suspected of poisoning ----..

A) only if there were a history of depression

B) as it has to be proved

C) as if there were no easy diagnosis

D) whether it seemed likely or not

E) until there is evidence to the contrary

33. Some firms provide special services for frequent


buyers such as encouraging repeat business with
discounts ----..

A) because their previous experience may not be


relevant

B) so that making purchases from them becomes a


part of the customers routine

C) although this practice is similar to the process of


problem solving

D) when consumers face a really new concept

E) as it does not confirm how long this offer will be


valid
35. ----, the station has been forced to make an
effort to win them back..

A) At a time when launching a radio station is cheaper

B) As young audiences have stopped watching its


main television channel

C) Even if a channel for young children costs more for


each user

D) After an alternative rock-station received plenty of


promotion

E) While most efforts are directed towards finding a


new audience

36. ----, many European countries took a close


interest in the New World..

A) After Columbus discovered America in 1492

B) Since explorations across the Atlantic had lasted


for decades

C) Although the population of Europe in the 15th and


16th centuries rose at a stable rate

D) Just as Columbus was of Italian origin

E) Since African slaves were chiefly imported for


agricultural purposes
37. - 42.sorularda, verilen ngilizce cmleye
anlamca en yakn Trke cmleyi, Trke
cmleye anlamca en yakn ngilizce cmleyi
bulunuz.

37. Shakespearein Coriolanus tragedyas, halka


hakaret ettii iin Romadan srlen marur
komutan Caius Marcus Coriolanusun yaamn
ve lmn ele alr..

A) Shakespeares tragedy Coriolanus is based on the


life and death of the proud commander Caius
Marcus Coriolanus who was driven out of Rome by
the angry people there.

B) The life and death of Caius Marcus Coriolanus, a


confident commander who was driven out of Rome
for ill-treating the people, is the subject of
Shakespeares tragedy Coriolanus.

C) Shakespeares tragedy Coriolanus concerns the life


and death of Caius Marcus Coriolanus, a proud
commander driven from Rome for insulting the
people.

D) The tragedy, Coriolanus, by Shakespeare, is based


on the life and death of the proud commander
Caius Marcus Coriolanus forced by the people he
had insulted there, to leave Rome.

E) Shakespeares tragedy Coriolanus is about the


arrogant commander Caius Marcus Coriolanus who
so insulted the people of Rome that they drove him
out of their city.
38. Leibniz, bilgisayar programnn icadndan 250 yl 39. Bilim adamlar insan vcudunun nasl altna
nce yaam olmasna ramen, modern ilikin daha fazla bilgi edinmek iin eitli deniz
algoritmik bilgi dncesine ok yaklamt. . yaratklarndan yararlanmaktadr..

A) Although Leibniz lived 250 years before the A) According to some scientists, the study of sea
invention of the computer programme, he came creatures can lead to a better understanding of the
very close to the modern idea of algorithmic human body.
information.
B) Several scientists are now making use of sea
B) Leibniz, who lived some 250 years before the creatures to help them understand how the human
introduction of the computer programme, was in body works.
fact fully familiar with the idea of modern
algorithmic information. C) Scientists are making use of various sea creatures
to learn more about how the human body works.
C) It was just 250 years before the development of the
computer programme that Leibniz lived and put D) Scientists have now begun to study various sea
forward the modern idea of algorithmic creatures and are learning more about how the
information. human body works.

D) Living 250 years before the launching of the E) Scientists have now begun to study various sea
computer programme, Leibniz had a notion of creatures and are learning more about how the
modern algorithmic information. human body works.

E) Even though the computer programme was


invented 250 years after Leibniz, he was actually 40. A research report prepared by a financial firm
aware of the idea underlying modern algorithmic suggests that the private sector in China
information. accounts for no more than 30 per cent of the
economy..

A) Bir finans irketinin son aratrma raporuna gre,


inde zel sektrn ekonomideki pay % 30dan
fazla deildir.

B) Bir finans kuruluunun raporunda belirtildii gibi,


inde zel sektr, ekonominin % 30undan
fazlasn oluturmamaktadr.

C) Bir finans irketinin hazrlad aratrma raporuna


gre, in ekonomisinin % 30unu zel sektr
oluturuyor.

D) Bir finans irketinin yapt aratrma, inde zel


sektrn ekonomide yaklak % 30 pay olduunu
iddia ediyor.

E) Bir finans irketi tarafndan hazrlanan aratrma


raporu, inde zel sektrn ekonominin % 30-
undan daha fazlasn oluturmadn ileri sryor.
41. Animals have internal defence mechanisms to
protect them against disease-causing organisms
that enter the body through air, food and water
and through wounds in the skin..

A) Vcuda hava, yiyecek ve su ile ve derideki yaralar


yoluyla giren organizmalar, hayvanlarn dahili
savunma mekanizmalarn etkisiz hale getirerek
hastala neden olabilirler.

B) Hayvanlarn sahip olduu dahili savunma


mekanizmalar, onlar hava, yiyecek ve su ile ve
derideki yaralar yoluyla vcuda giren hastalk
yapc organizmalara kar korur.

C) Hayvanlar dahili savunma mekanizmalarna sahip


olmalarna ramen hava, yiyecek ve su ile ve
derideki yaralar yoluyla vcuda giren hastalk
yapc organizmalara kar daima savunmaszdrlar.

D) Hayvanlarn, hava, yiyecek ve su ile veya derideki


yaralar yoluyla vcuda giren hastalk yapc
organizmalara kar gelitirdikleri dahili savunma
mekanizmalar vardr.

E) Hayvanlar hava, yiyecek ve su ile ve derideki


yaralar yoluyla vcuda giren hastalk yapc
organizmalara kar kendilerini koruyan dahili
savunma mekanizmalarna sahiptir.

42. Each year in the United States, there appear an


estimated number of 180, 000 new cases of
breast cancer, and more than 40, 000 women die
from it..

A) Her yl Birleik Devletlerde ortaya kan tahmini


180.000 yeni meme kanseri vakasnn 40.000 den
fazlas lmle sonulanmaktadr.

B) Birleik Devletlerde ortaya kan, yllk tahmini


180.000 yeni meme kanseri vakasnda her yl
40.000den fazla kadnn ld grlmektedir.

C) Her yl 40.000den fazla kadnn meme kanserinden


ld Birleik Devletlerde her yl tahminen
180.000 yeni vaka grlmektedir.

D) Her yl Birleik Devletlerde tahminen 180.000 yeni


meme kanseri vakas ortaya kmaktadr ve
40.000den fazla kadn bundan lmektedir.

E) Birleik Devletlerde her yl gzlemlenen tahmini


180.000 yeni meme kanseri vakasnn 40.000 den
fazlasnda hastalarn kaybedildii belirlenmitir.
45. It is pointed out in the passage that the loss
43. - 46.sorular aadaki paraya gre
ofneurons in the mid-brain ----..
cevaplaynz.

A) is an incidence that can be observed easily


Parkinsons and Alzheimers diseases are the B) initiates Alzheimers disease
mostcommon diseases which affect many people in
theworld. Approximately, 1 million people in the C) results from general protein deficiency
worldsuffer from Parkinsons disease, a motor
disordercharacterized by difficulty in initiating movements D) happens when the bodys movements begin to slow
andslowness of movement. Patients often have amasked down
facial expression, poor balance, and a flexedposture. Like
E) leads to Parkinsons disease
Alzheimers disease, which ischaracterized by confusion,
memory loss, and avariety of other symptoms, Parkinsons
disease isprogressive, and the risk increases with age.
Theincidence of the Parkinsons disease is about 1% atthe
age of 65, and about 5% at the age 85.Parkinsons disease
appears to result from acombination of environmental and
genetic factors.Evidence for a genetic role includes the
fact thatsome families with an increased incidence
ofParkinsons disease carry a mutated form of the genefor
a protein which is important in normal brainfunction. The
symptoms of Parkinsons disease resultfrom the death of
neurons in the mid-brain. As aresult, at present, there is
no cure for Parkinsonsdisease, although various
treatments can help controlthe symptoms.

43. According to the passage, Parkinsons disease -


---..

A) has its adverse effects on the bodys movements

B) can easily be treated at an advanced age

C) can best be understood through a close study of


the memory

D) can be cured provided that its symptoms are taken


into account at an early stage

E) is a consequence of purely environmental factors

44. It is clear from the passage that


Alzheimersdisease ----..

A) affects more people in the world than Parkinsons


disease

B) is a major genetic disorder that can today be


controlled

C) can be recognized through the patients loss of


memory as well as a number of other symptoms

D) is common mostly among elderly people over the


age of 80

E) has attracted more medical attention in the world


than Parkinsons disease
46. As clearly pointed out in the passage, there is -
---..

A) a great deal of protein deficiency in many families


prone to Alzheimers disease

B) much evidence to claim that Parkinsons disease


can be cured in the near future

C) a variety of treatments whereby the symptoms of


Parkinsons disease can be controlled

D) a need to develop a new technique whereby the


movements of the body can be improved

E) much controversy among physicians in the world


on the treatment of Alzheimers disease
49. The point has been made in the passage that the
47. - 50.sorular aadaki paraya gre
American people ----..
cevaplaynz.

A) have always been remarkably hospitable to Asians


visiting America
The assumption that a persons attitudes determine his or
her behaviour is deeply ingrained in Western thinking, and B) in the 1930s really did dislike Asians
in many instances the assumption holds. However,
research has shown that the relationship between C) have waged war against racial discrimination since
attitudes and behaviour is complex. A classic study the 1930s
conducted during the 1930s was the first to question the
link. A white professor travelled across the US with a D) and the Asians have always discriminated against
each other
young Chinese couple. At that time, there was quite
strong prejudice against Asians, and there were no laws E) always do their best to provide comfort for their
against racial discrimination. The three travellers stopped guests, no matter what their racial background
at over 200 hotels, motels and restaurants, and were
served at all the restaurants and all but one of the hotels
and motels without problem. Later, a letter was sent to all
of the establishments visited, asking them whether or not
they would accept a Chinese couple as guests. Of the 128
replies received, 92 per cent said they would not. In other
words, these proprietors expressed attitudes that were
much more prejudiced than their behaviour.

47. One understands from the passage that the link


between attitudes and behaviour ----..

A) had already been established before the 1930s

B) has always been a major topic of psychological


research

C) has aroused a great deal of controversy in the US

D) is an area which has been neglected and, indeed,


deserves fresh study

E) is not so definite as has been generally assumed

48. As one concludes from the passage, racial


discrimination ----..

A) is the way whereby the relationship between


attitudes and behaviour can best be assessed

B) has always been a major issue in the US and


cannot be eradicated

C) was a problem of the 1930s in the US, but has now


disappeared altogether

D) in the US and the rest of the world can be fought


most effectively through legislation

E) was not practised by a great majority of the


proprietors visited by the professor and his Chinese
companions
50. It is clear from the passage that the replies the
professor received from the proprietors to whom
he sent letters ----..

A) revealed a strong attitude of racial discrimination


against the Chinese

B) could not be regarded as an indication of prevalent


racism in the US

C) were not concerned with the interrelationship


between ones attitudes and behaviour

D) were confined only to a very small part of the US

E) clearly showed why ones attitudes determine


ones behaviour
54. We can infer from the passage that Kenneth Lay
51. - 54.sorular aadaki paraya gre
expected Mr Bush to offer him a high position in
cevaplaynz. his administration because Mr Lay ----..

A) had contributed a very large amount of money to


Until the giant American energy company Enron collapsed, Mr Bushs presidential campaign
and its director Kenneth Lay was imprisoned, his life had
been a model of the American dream of rising from rags B) was an important player in the Texan oil industry
to riches on the strength of merit and hard work. His
beginnings were socially and financially very modest. He C) was frustrated with his political life
was born in Tyrone, Missouri, in 1942, as the son of a
preacher who was also a part-time salesman. He helped D) had obtained a high level of education, and was
therefore quite knowledgeable
his father make ends meet by cutting grass and delivering
papers. His start in the energy industry seemed similarly E) had become very rich through his hard work
modest. After obtaining a doctoral degree in economics
from the University of Houston, he got his start in the
booming Texan oil industry. In 1985 he merged Houston
Natural Gas with Inter North of Nebraska in order to form
Enron. As Enron became stronger, Mr Lay turned
increasingly to politics and was one of the biggest donors
to the Bush-Cheney campaign. After Mr Bush entered the
White House, Mr Lay had hopes of a seat in the cabinet,
perhaps as energy secretary or even at the Treasury.
However, for reasons that remain unclear, Mr Bush
overlooked him, so his professional life ended in
frustration.

51. According to the passage, after Mr Bush was


elected president of the US, Kenneth Lay ----..

A) turned increasingly to politics

B) became involved in the Texan oil industry

C) was not offered a cabinet seat

D) obtained a doctoral degree from the University of


Houston

E) dissolved Enron, the company he had created

52. It is clear from the passage that the giant


American energy company Enron was founded
through ----..

A) Kenneth Lays increasing interest in politics

B) Kenneth Lays dream of rising from rags to riches

C) the fact that Kenneth Lay had been imprisoned

D) Kenneth Lays modest beginnings as the son of a


preacher and part-time salesman

E) the merging of two companies: Houston Natural


Gas and InterNorth

53. We understand from the passage that, when


Kenneth Lay was a child, he ----..

A) worked as a part-time salesman together with his


father

B) wanted to become a preacher like his father

C) moved with his family from Tyrone, Missouri, to


Houston, Texas

D) contributed to his familys income by working at


part-time jobs

E) dreamed of becoming an oil tycoon


58. According to the passage, the limited supply of
55. - 58.sorular aadaki paraya gre
oxygen underground ----..
cevaplaynz.

A) travels along the thin layers of rock or mineral


Contrary to popular belief, underground fires are a B) is a very frequent phenomenon in Australia and
surprisingly frequent phenomenon, the fuel being coal and China
the fire travelling along the seams, or the thin layers of
rock or mineral. Such fires travel slowly due to the limited C) prevents underground fires from burning for any
supply of oxygen, but can burn fora very long time: the length of time
underground fire at Burning Mountain Nature Reserve in
Australia is thought to have been continuing for the past D) slows down the speed at which underground fires
travel
5, 500 years. The number of such subterranean fires
worldwide is countless. According to one study, E) has been contributing to global warming for the
subterranean fires in China alone are consuming some past 5,500 years
200 million tonnes of coal a year and pumping into the air
as many pollutants as all the cars in the United
States.Along with numerous human-related factors, such
fires are also contributing substantially to global warming.

55. It can be understood from the passage that


underground fires ----..

A) are a direct result of human activities

B) are a rare phenomenon that is observed solely in


Australia and China

C) play an important role in the process of global


warming

D) could contribute to efforts to prevent climate


change resulting from global warming

E) have not been witnessed for the past 5,500 years

56. According to the passage, one misconception


about underground fires is that they ----..

A) are not common

B) dont last for a very long time

C) dont have a limited supply of oxygen

D) travel fast

E) contradict the laws of nature

57. Underground fires in China ----..

A) release about 200 tonnes of pollutants into the air


every year

B) cause as much air pollution as do all the vehicles in


the United States

C) have been burning for more than five millennia

D) consume as much energy as do all the cars in the


United States

E) are most frequently the result of industrial activity


61. According to the passage, stratigraphy ----..
59. - 62.sorular aadaki paraya gre
cevaplaynz.
A) helped scientists to understand the relationship
among only fossil records
Palaeontology was once limited to digging up fossil sand B) deals mainly with vegetative life in a given region
trying to deduce their age with inaccurate methods.
However, fossil analysis improved dramatically in the C) started to be used in the study of fossils long after
1960s, with the advent and refinement of two techniques: its emergence as a science
radiometric dating and stratigraphy. The first radiometric
method was also known as carbon-14 dating, and it was D) later established the basis for DNA dating
usable for specimens younger than 50, 000 years. Later,
E) allowed scientists to refine the methods of
potassium-argon dating revolutionized the field by
biostratigraphy
enabling scientists to detect the radioactive decay of
elements found naturally in rocks and soil surrounding
much older fossils. Stratigraphy, which is the study of rock 62. The passage is mainly concerned with ----..
layering, actually was developed well before the 1960s,
but that was the decade scientists began to better
A) the advanced methods palaeontologists use to date
understand how geological conditions, earthen layers, and fossils
fossil records all relate. The resulting refinement of
biostratigraphy, i.e., the study of the complete life of a B) how palaeontology came to be recognized as a
stratum of earth, allowed scientists to determine the field of science
environment and lifestyle of human ancestors based on
fossilized flora and fauna found within the same layer as C) recent advances made in the study of genetic
the hominine fossils. Since the 1960s, DNA testing has mutations
come to be used widely. As all living organisms have the
D) the development of DNA testing and its use in
same genetic code, scientists can use DNA variations as a
deciphering the genetic code
molecular clock. After splitting with a common ancestor,
each generation develops a constant rate of genetic E) the relationship between humans and the
mutations. The molecular clock allows scientists to environment they live in
calculate how long ago the split occurred based on the
number of differences between species. The method is
now helping scientists map the routes that humans took 63. - 67.sorularda, karlkl konumann bo
out of Africa. braklan ksmn tamamlayabilecek ifadeyi
bulunuz.

59. It can be understood from the passage that the


carbon-14 method ----.. 63. Pelin: Although the Great Mosque and Hospital of
Divrii is on the World Heritage List, it doesnt
get the attention it deserves by the media.
A) involves the use of radioactivity to make fossils
Cansu: What do you think they can do to increase
decay
its recognition both across the country and in the
world?
B) can be used to date fossils of any age
Pelin: ----
Cansu: Im sure those ideas would create a great
C) revolutionized potassium-argon dating
influence..
D) was no longer practiced after the 1960s
A) TV programmers could choose to feature the place
E) was the earliest form of radiometric dating
in their most-watched programs and the
newspapers could mention it.
60. It is clearly stressed in the passage that, before
the development of different dating methods, - B) I dont expect any help from them, because theyre
---.. generally interested in doing contests for
amusement.

A) the molecular clock was used by scientists to C) I think documentaries about historical places are
determine the age of fossils very boring, thats why nobody wants to watch
them.
B) the age of fossils could not be determined reliably
D) In our modern world, people who work for hours
C) palaeontology was regarded as a field of study that dont have enough time to watch TV or read
was only concerned with the excavation of fossils newspapers regularly to be aware of such facts.

D) it was impossible to know whether Africa was the E) If I were a famous singer or an actress, I would visit
home of the first human beings there and thus everybody would hear about it
thanks to me.
E) only the age of fossils which were older than
50,000 years could be determined accurately
64. Nicole: According to a study, almost half of the 67. Andy: Here is an article about the latest epidemic
worlds billionaires live in the US. of swine flu. It says that it started in Mexico and
Claudia: ---- it has now infected at least one hundred people
Nicole: Thats true, but it doesnt mean the in ten other countries. Beth: ---- Andy : Well,
whole populace of the country is rich and some airlines stopped their flights to the country
prosperous. and some governments urged their citizens not
Claudia: Yes, we cannot disregard the homeless to travel there. Beth: Those precautions certainly
and needy people who reside in the streets.. seem reasonable..

A) I know. On the other hand, Nigeria with a low A) What\'s being done to stop the spread of the
economic activity has the poorest citizens. disease?

B) That didnt surprise me at all, as the financial B) Do they have any research to support their claims?
centre of the world is located there.
C) Should we go to the doctor to get a flu shot?
C) I bet Bill Gates is one of them with an enormous
income that he gained with the help of his genius. D) Do you think we should fly to Mexico for holiday?

D) Thus, the national debt of the US surpassed 10- E) Are the drug companies taking advantage of the
trillion dollars, bringing the country into the largest disease?
national debt in 2008.

E) Thats why people argue about whether 68. - 71.sorularda, verilen cmleye anlamca en
globalization has eradicated borders and cultural yakn cmleyi bulunuz.
differences

65. Jale: - Here is an article about how people react


in emergencies. Researchers say that when more 68. As inhabitants of one of earths most densely
people are around, it reduces the chances of populated continents, Europeans know well that
actually being helped. environmental protection is not a luxury, but
Adnan: - ---- rather a necessity..
Jale: - Apparently, onlookers provide a model for
action. If they are docile and disinterested, the
situation may seem less serious. A) Living in one of the worlds most densely-populated
Adnan: - I think if there is only one bystander, continents, Europeans are well aware that they
your chance of being helped increases, as he will need to protect the environment.
think he must help immediately..
B) More people live on the European continent than
anywhere else and for this reason they know that
A) Is an individual aware that others are present? they have a duty to protect the environment.

B) How did they carry out that research? C) It is necessary for Europeans to stop living in luxury
and start taking more responsibility for the
C) Do they offer any explanation as to why this protection of the European continent.
happens?
D) Of all the densely-populated continents on earth,
D) Is this finding true for all cultures? Europe is the one that is in most need of
environmental protection.
E) Who were the participants in this research?
E) The luxury of protecting the environment can no
longer be applied to earths continents, particularly
66. James: You must read this article on global in the case of Europe.
climate change.
Steve: Is it about carbon dioxide emissions
again? If so, Im really getting rather bored with 69. Reports about the ozone layer are as
the subject. maddeningly variable as the protective shield
James: So am I. The articles are so repetitive. itself..
Only this one says something different.
Steve: ----.
A) Reports on the ozone layer and its efficiency as a
protective shield are incredibly inconsistent.
A) The author is certainly well-known; this is the fifth
time hes been interviewed on the issue by TV B) There is no consistency in the reports concerning
channels the ozone layer and the protective shield itself,
which is very embarrassing.
B) Give it to Robert; hes already published something
on it. C) Reports on the ozone layer concentrate on the
irritating variability of the protective shield.
C) So, it focuses on renewable energy supplies rather
than fossil fuels. D) The frustrating lack of consistency in reports
concerning the ozone layer matches the state of
D) Id rather download a portable document from the the protective shield itself.
university library and give it to you as well.
E) The variability of the ozone layer is reflected in the
E) You mean it presents a different perspective on the lack of consistency in reports concerning its
subject. Ill take a look at it, then. protective functions.
70. He has retired, and he is now consultant for 73. Cycling is a slow but environmentally friendly
several businesses and this brings him in more way of getting around. ----. Besides, it is easy to
money than he ever used to earn in his full-time rent bicycles in many tourist areas, particularly
job.. at main train stations..

A) If he had wanted to retire, he could have acted as a A) If theres a lot of heavy rain, its not much fun
consultant for several agencies and this would have
given him a better income. B) On the other hand, manufacturers have improved
the quality of their products
B) Once he had retired, he became a consultant for
several organizations because he needed an C) Even so, a good map of the area is not necessary
additional income.
D) It enables you to combine sightseeing with physical
C) So that his income would not drop when he retired exercise
from full-time employment, he started to work in an
advisory capacity for several organizations. E) If youre travelling by bus, be sure to get an up to-
date timetable
D) Though he has been retired for some time, he now
earns twice as much as when he was in a full-time
job, because he is working as a consultant for quite 74. Ulcerative colitis is a disease in which extensive
a lot of firms. areas of the walls of the large intestine become
inflamed and ulcerated. Some clinicians believe
E) He earned less when he was in full-time that it results from an allergic or immune
employment than he does now that he is retired destructive effect, but it could also result from a
and acting as a consultant for two or three firms. chronic bacterial infection. ----..

71. Its not fair to put all the blame on him; he is not A) The person may also have a fever and a poor
the only one at fault.. appetite

B) Even then, the ulcers sometimes fail to heal, and


A) It wouldnt be right to punish those who, like him, the only solution is removal of the entire colon
werent involved.
C) Moreover, about 10 per cent of people who have
B) He deserves to be punished, but the others dont. the disease have only simple attacks

C) Only the ones who were involved should be D) Whatever the cause, there is a strong hereditary
punished. tendency for susceptibility to this disease

D) The fault is his only; so let him take the blame. E) Unlike Crohns disease, ulcerative colitis never
affects the small intestine
E) He doesnt deserve to be blamed for everything;
there were others involved.
75. Promotion is where marketing goes over into
selling. It involves communicating all the
information necessary to persuade the customer
72. - 75.sorularda, bo braklan yere, parada
to buy the product. ---- Advertising pulls it
anlam btnln salamak iin makes customers aware of the product and
getirilebilecek cmleyi bulunuz. prompts them to ask for it, but it can be
expensive. In a push strategy, the sales force
promotes the product to wholesalers and
retailers and push it through distribution
72. The United Arab Emirates (UAEs) poultry sector channels to the end user..
has lost $5.5 million because of a 50% cut in
consumption due to bird flu fears. Many small
farms have been forced to close, and production A) It was only after the Second World War that
has been cut by 30%, with surplus meat being marketing began to evolve to pull in customers.
frozen and sold off cheaply. The consumption of
eggs has not been affected. ----.. B) Price is a flexible element, since it can be changed
very quickly.

A) Nevertheless, the country claims to be free of the C) A key decision will be the selection of a distribution
disease channel.

B) Moreover, Kuwait and Saudi Arabia have D) Decisions to be made here will include what the
announced cases of bird flu on their territories product will look like.

C) In fact, the UAEs oil income has doubled over the E) Promotional strategies tend to be categorized as
last few years due to a sharp rise in its exports either push or pull.

D) Among the Gulf states, it is the UAE that most


aspires to become a major global investor

E) However, the UAE is facing big increases in the


price of consumer goods and transport equipment
imported from Europe
78. (I) British dominance of the slave trade in the
76. - 80.sorularda, cmleler srasyla
late seventeenth and early eighteenth centuries
okunduunda parann anlam btnln gave it decisive economic advantages over other
bozan cmleyi bulunuz. nations. (II) As one Englishman wrote in 1749,
the slave trade had provided an inexhaustible
fund of wealth to this nation. (III) But even apart
from the slave trade, the value of colonial
76. (I) Seismic waves are the vibrations from commerce was increasing dramatically during
earthquakes that travel through Earth. (II) The the eighteenth century. (IV) Like the Spanish
Richter magnitude scale was developed in 1935 colonies, the French colonies in the New World
by Charles F. Richter of the California Institute of were established and administered as direct
Technology to compare the intensity of crown enterprises. (V) For instance, British
earthquakes. (III) They are recorded on foreign trade increased in value from 10 million
instruments called seismographs. (IV) in the 1730s to 40 million in the 1750s..
Seismographs record a zigzag trace that shows
the varying amplitude of ground oscillations
beneath the instrument. (V) Sensitive A) I
seismographs, which greatly magnify these
ground movements, can detect strong B) II
earthquakes from sources anywhere in the
world.. C) III

D) IV
A) I
E) V
B) II

C) III

D) IV

E) V

77. (I) According to recent data, doctors sometimes


misdiagnose migraines as sinus headaches. (II)
Migraines are an inherited form of recurring
headaches. (III) This confusion in diagnosis
occurs because neurons in the brain stem can
activate the sinuses during a migraine, causing
them to secrete a clear fluid. (IV) Yet this
discharge differs from the cloudy fluid produced
by a sinus infection. (V) Therefore, it is advisable
for doctors to consider migraines more seriously
and to look for clear nasal secretions as
symptoms..

A) I

B) II

C) III

D) IV

E) V
79. (I) X-rays are a form of electromagnetic radiation
that readily penetrates human flesh. (II) As soon
as the existence of X-rays had been reported, the
medical applications with radioactive materials
were recognized. (III) However, it took quite a
while longer for people to detect the dangers of
these mysterious new rays. (IV) All X-rays are
not man-made; extremely hot stars, such as
neutron stars, also produce huge amounts of X-
rays. (V) For example, Major John Hall-Edwards,
who helped pioneer the use of X-rays in medical
treatments, lost an arm because of excessive
exposure..

A) I

B) II

C) III

D) IV

E) V
80. (I) Lakes have obviously played an important role
in the historical development of communities,
and modern life is dependent on the purification
facilities and agricultural benefits that lakes
provide. (II) As renewable energy becomes
increasingly important in the 21st century, so do
lakes and the possibilities of hydroelectric power
that they present. (III) Major threats to the
longevity of lake fertility are pollution, drainage
and faulty water-management practices. (IV)
Economically, lakes play an integral part in the
development of major waterways and travel
routes. (V) Fishing and aquaculture, and the jobs
their industry represents, are also principle
benefits of living in a lake community..

A) I

B) II

C) III

D) IV

E) V
SORU CEVAP SORU CEVAP

1 B 41 E

2 C 42 D

3 C 43 A

4 E 44 C

5 E 45 E

6 B 46 C

7 D 47 E

8 C 48 E

9 C 49 B

10 B 50 A

11 A 51 C

12 D 52 E

13 C 53 D

14 B 54 A

15 A 55 C

16 B 56 A

17 D 57 B

18 A 58 D

19 E 59 E

20 C 60 B

21 B 61 C

22 B 62 A

23 A 63 A

24 E 64 B

25 D 65 C

26 A 66 E

27 B 67 A

28 A 68 A

29 D 69 D

30 D 70 E

31 C 71 E

32 E 72 A

33 B 73 D

34 C 74 D

35 B 75 E

36 A 76 B

37 C 77 B

38 A 78 D

39 C 79 D

40 E 80 C
24
5. Since total sleeping time is likely to decrease
1. - 16.sorularda, cmlede bo braklan yerlere
with age, older people may find going to bed
uygun den szck ya da ifadeyi bulunuz. later or ---- earlier helpful..

A) getting up
1. Psychologists say that there is a genetic ---- to
will power, but that upbringing may also have an B) making out
effect..
C) breaking down

A) attachment D) keeping off

B) component E) taking away

C) description

D) reaction

E) selection

2. Most commercially available milk has been


pasteurized with heat to kill bovine tuberculosis
organisms and other ---- pathogens..

A) appealing

B) religious

C) possible

D) voluntary

E) profitable

3. Anaesthetics are drugs given to patients before


undergoing surgery to ---- the sense of feeling
either in a localized area or across the whole
body..

A) attract

B) reflect

C) reduce

D) approve

E) evaluate

4. Use of agricultural machinery ---- reduces the


amount of human labor needed for raising crops..

A) costly

B) exactly

C) chiefly

D) substantially

E) seriously
6. Most of our exposure to organic mercury ----
eating oily fish such as tuna..

A) plays up

B) turns into

C) comes from

D) finds out

E) puts up with

7. For several years now she ---- as private


secretary to the bank manager..

A) has been working

B) worked

C) was working

D) works

E) used to work

8. Since Taiwan ---- its recession the retail clothing


industry ---- very badly..

A) has entered / would have been doing

B) had entered / would do

C) would enter / has done

D) was entering / will be doing

E) entered / has been doing

9. Some scientists say that it is still not too late to


minimize the effects of climate change ----
enough money and effort are invested in correct
projects..

A) provided that

B) in case

C) even if

D) so that

E) whereas
10. Florence, in Italy, is famous among scholars ----
its cultural heritage and the major role it played
in the Renaissance and Humanist movements..

A) moreover

B) due to

C) so as to

D) because

E) despite
11. He invariably gets what he wants but 15. The processes of food production in Paleolithic
unfortunately this is usually ---- the expense of times ---- simple, ---- of gathering, hunting, and
others.. fishing..

A) out of A) were to be / to be consisting

B) from B) were / consisting

C) up to C) had been / consisted

D) at D) are / to consist

E) through E) have been / consists

12. ---- the world there has been a growing sense of


disillusionment with the United Nations and its
policies..

A) For

B) Among

C) On

D) From

E) Throughout

13. ---- the time a child reaches school age he ought


to be able to dress himself fairly quickly..

A) On

B) For

C) In

D) At

E) By

14. Industrialized nations can produce ----


manufactured goods ---- their people need or can
afford to buy..

A) such / as

B) no sooner / than

C) more / than

D) both / and

E) as / as
16. Unlike conventional computers, ---- operate in
linear fashion, taking on tasks one at a time,
DNA computers could perform calculations
simultaneously..

A) where

B) which

C) that

D) whose

E) in which
21. V.
17. - 21.sorularda, aadaki parada
numaralanm yerlere uygun den szck ya
da ifadeyi bulunuz. A) unless

B) because

Asilent auction is a fundraising technique frequently used C) so


at charity events. Itdiffers (I) ---- the typical
auctionconducted by an auctioneer (II) ---- callsout the D) even when
prices (III) ---- items andthen takes the raised hands or
paddles of people as bids at a certain price.Its often a E) but
preferred (IV) ---- forraising money at charitable events (V)
---- itdoes not detract from the entertainment provided at
the event.

17. I.

A) at

B) from

C) by

D) in

E) of

18. II.

A) whom

B) whomsoever

C) whoever

D) who

E) whomever

19. III.

A) at

B) off

C) with

D) from

E) for

20. IV.

A) means

B) appearance

C) insurance

D) quantity

E) reference
26. V.
22. - 26.sorularda, aadaki parada
numaralanm yerlere uygun den szck ya
da ifadeyi bulunuz. A) will communicate

B) is communicated

---- most peopleassociate telecommunications with C) having communicated


modern technologies, the strict definition ofthe term (II) -
--- primitive and (III) ---- ancient forms of D) would have communicated
telecommunication.Among these is the use of smoke
signals as a kind of visual telegraph. Puffs ofsmoke were E) could communicate
time-released (IV) ---- smotheringa fire with a blanket,
then quickly removing and replacing the blanket.
Widelyused by the American Indians, smoke signals (V) -
---short messages over long distances, assuming a clear
line of sight.

22. I.

A) When

B) While

C) Just as

D) However

E) If

23. II.

A) enhances

B) resigns

C) protects

D) encompasses

E) reacts

24. III.

A) even

B) just

C) yet

D) already

E) only

25. IV.

A) with

B) in

C) for

D) by

E) to
27. Until fairly recently, we viewed the ocean as a
bountiful, virtually limitless resource, ----..

A) so many countries are also taking steps to restore


and conserve wetlands

B) unless we are now seeing the effects of our


disregard for marine communities

C) yet seafood would become less plentiful

D) and we have harvested the ocean heavily and used


it as a dumping ground for wastes

E) regardless of the fact that laws in many countries


now prohibit disposal of sewage and other wastes
at sea

28. Because symptoms of sleep apnea occur during


sleep, ----..

A) severe apnea can result in headaches, excessive


daytime sleepiness and slow mental activity

B) they must be described by someone who observes


the person sleeping

C) snoring is associated with episodes of gasping and


choking

D) most people have been treated successfully and


can now sleep comfortably

E) people who snore are advised to sleep on their side


or face-down

29. When a rise in civil-servant numbers resulted in


economic problems in Tanzania, ----..

A) newly emerged leaders often offered civil service


jobs to their supporters in Africa

B) the payment of civil servants is still problematic

C) a painful cut in pay would be inevitable because of


staff numbers

D) the IMF forced the government to cut the total


wage bill

E) holding civil servants to account is another problem


30. As long as the inflation rate differs from what is
expected, ----..

A) there will always be winners and losers

B) the same is true for unanticipated inflation

C) it is called hyperinflation when rates exceed 50%


per month

D) the purchasing power of your wage would be less


than you anticipated

E) many people devoted their time to speculation in


real estate
31. Although it is not known exactly when pottery 34. ----, modern grand pianos have metal frames
making began in Cappadocia, ----.. with heavier strings and can be played much
more loudly..

A) early pottery was generally shaped by the


wrapping-rolling method A) Since the only instrument symbolizing the
Romantic period is the piano
B) it is still agreed that the art originated in
Mesopotamia B) Whereas some pianos have a third central pedal
like that of a tambourine
C) it dates back at least to Hittite times
C) Though many orchestral works are arranged for the
D) most archaeologists are keenly interested in the piano
excavations that are going on
D) While early pianos are largely made of wood and
E) pottery belonging to other cultures has been found have a delicate sound
in the townships of Hacbekta and Gzelyurt
E) As the glockenspiel piano requires years of training
to master
32. Patients with medial epicondylitis should be
started on physical therapy ----..

A) unless rest were of first importance

B) if surgical release of the flexor muscle had been


necessary

C) in which case surgical intervention is usually


successful

D) once the pain has subsided

E) as there was pain over the medial side of the elbow

33. It is doubtful that the proposal will be accepted


in the parliament ----..

A) unless there are more important matters to


consider

B) since it would be considered a tax increase by


many

C) after the parliament resumes the new years


budget-making

D) while federal programs must be paid for by federal


funding

E) whereas the proposal itself is never discussed by


the committee
35. Solar power stations can be an economical way
to generate power ----..

A) because the power generated by electricity is


insignificant

B) if a region has a steady amount of sunshine

C) even though they have built a demonstration plant


in Italy

D) although they already operate a power plant there

E) since power from solar power plants is about twice


as expensive

36. The most rapid growth in 2005 in African tourism


was recorded in Kenya, ----..

A) while it is difficult to overestimate the potential


benefit of tourism to the South African economy

B) even though Africa is an increasingly attractive


destination for tourists

C) whether tourism had a positive or negative impact


on a country

D) since there would certainly be plenty of room for


growth in the tourist sectors of many African
countries

E) where the number of visitors increased by 26%


37. - 42.sorularda, verilen ngilizce cmleye
anlamca en yakn Trke cmleyi, Trke
cmleye anlamca en yakn ngilizce cmleyi
bulunuz.

37. Satrn buz kapl saysz paracktan


oluan muhteem halka sistemi ile tannr..

A) What makes Saturn remarkable is its superb ring


system composed of countless-ice-covered
particles.

B) Saturn has a magnificent ring system, composed of


innumerable ice covered particles.

C) The special characteristic of Saturn is its fantastic


ring system made up of millions of ice covered
particles

D) The incredible ring system of Saturn is made up of


ice-covered particles

E) Saturn is noted for its magnificent ring system,


which is composed of myriads of ice-covered
particles.
38. Kresel snma hzlandka ve 39. Nicholas Gane'in yeni yaymlanan kitab
enerji gereksinimlerimiz artmaya devam ettike, Toplum Kuramnn Gelecei, dnyann nde gelen
bizim daha temiz ve daha srekli enerji toplum kuramclar ile yaplan bir dizi mlakat
kaynaklarna sahip olmamz gerekir.. bir araya getirmektedir..

A) Cleaner and more reliable sources of energy must A) The newly published book, The Future of Social
be found or global warming will accelerate, and it Theory by Nicholas Gene, draws heavily on a series
will be impossible to meet energy demands. of interviews conducted by the world\'s leading
social theorists.
B) As the process of global warming speeds up and
demands for energy rise, we need to find cleaner B) Nicholas Gane\'s newly published book The Future
and more sustainable sources of energy. of Social Theory brings together a series of
interviews held with the world\'s leading
C) If global warming continues and the demands for socialtheorists.
energy increase, we shall be forced to seek cleaner
and more sustainable sources of energy. C) A series of interviews with the world\'s leading
social theorists was the starting point for Nicholas
D) The demand for cleaner and more reliable sources Gane\'s newly published The Future of Social
of energy will increase if global warming continues Theory.
and energy requirement increase.
D) Nicholas Gane interviewed some of the world\'s
E) As global warming accelerates and our energy leading social theorists before writing The Future of
demands continue to rise, we have to have cleaner Social Theory which has recently been published.
and more sustainable source of energy.
E) Nicholas Gane\'s newly published book The Future
of Social Theory makes extensive use of interviews
between the writer and the world\'s leading social
theorists.

40. As we learn from the works of many biologists in


the past, since sponges did not move from one
place to another, they were thought to be
plants..

A) Gemiteki pek ok biyoloun, eserlerinde,


sngerlerden bitki olarak sz etmesinin nedeni, bu
canllarn bir yerden bir yere hareket etmemesidir.

B) Gemiteki biyologlarn eserlerinden rendiimiz


kadaryla, sngerler bir yerden bir yere hareket
etmemeleri dolaysyla bitkilerle ayn grupta ele
alnyordu

C) Gemiteki pek ok biyoloun eserlerinden


rendiimiz kadaryla, sngerler bir yerden bir
yere hareket etmedii iin onlarn bitki olduu
sanlyordu

D) Gemite, sngerler, bir yerden bir yere hareket


etmedikleri iin bitki olarak kabul edildii halde,
pek ok biyolog bu gre eserlerinde yer
vermemitir.

E) Sngerler, bir yerden bir yere hareket etmedikleri


iin, gemite bitki olarak snflanyordu; fakat
birok biyolog, eserlerinde bunun yanl olduunu
belirtmektedir
41. James Joyce, whom Samuel Beckett knew in Paris
in the 1920s, had a great impact on Becketts
thinking regarding the art of writing..

A) Samuel Beckettin 1920lerde Pariste tand


James Joyceun, Beckettin yazma sanatyla ilgili
dnceleri zerinde byk bir etkisi oldu.

B) James Joyceun yazma sanat konusundaki


dncelerinden ok etkilenen Samuel Beckett, onu
1920lerde Pariste tanmt.

C) Samuel Beckett 1920lerde James Joycela Pariste


tantktan sonra, yazma sanat konusundaki
dnceleri bir lde deiti.

D) James Joyce 1920lerde Pariste tand Samuel


Beckettin yazma sanat konusundaki dncelerini
byk lde etkiledi.

E) 1920lerde Pariste yaayan Samuel Beckettin


yazma sanat konusundaki dnceleri James Joyce
sayesinde tamamyla deiti.

42. When the Sirkeci train station went into service


on November 3, 1890, the waiting room was
heated with stoves brought from Austria and lit
by coal-gas lamps..

A) 3 Kasm 1890da hizmete giren Sirkeci tren


istasyonunun hava gaz lambalaryla aydnlatlan
bekleme salonu, ou Avusturyadan ithal edilen
sobalarla stlyordu.

B) Sirkeci tren istasyonu 3 Kasm 1890da hizmete


girdiinde, bekleme salonu Avusturyadan getirtilen
sobalarla stlyor ve hava gaz lambalaryla
aydnlatlyordu.

C) 3 Kasm 1890da, bekleme salonu hava gaz


lambalaryla aydnlatlan ve Avusturyadan ithal
edilen sobalarla stlan Sirkeci tren istasyonu
hizmete girdi.

D) Sirkeci tren istasyonu 3 Kasm 1890da hizmete


girdiinde, hava gaz lambalaryla aydnlatlan
bekleme salonunu stmak iin Avusturyadan soba
ithal edilmiti.

E) Sirkeci tren istasyonunun Avusturya sobalaryla


stlan ve hava gaz lambalaryla aydnlatlan
bekleme salonu, 3 Kasm 1890da hizmete girmiti
46. It is indicated in the passage that the
43. - 46.sorular aadaki paraya gre
disordersarising from faulty wiring in the brain -
cevaplaynz. ---..

A) have an adverse effect on depth perception


The pirate look is a time-honoured way to fix childrens
'lazy eye'. The patch over the good eye forces the weak B) are the consequences of the wrong treatment of
one to work, thereby preventing its deterioration. Playing the lazy eye
video games helps, too. The neural cells corresponding to
both eyes then learn tofire in synchrony so that the brain C) are not related to a neurodegenerative cause
wires itself for the stereo vision required for depth
perception. Left untreated past a critical age, lazy eye, or D) undermine the efficient functioning of the neural
cells concerning vision
amblyopia, can result in permanently impaired vision.
Newstudies are now showing that this condition, which E) cannot be related to brain plasticity at all
affects up to 5 per cent of the population, could
berepaired even past the critical age. What is more,
amblyopia may provide insights into brain plasticity that
could help treat a variety of other disordersrelated to
faulty wiring, including schizophrenia, epilepsy, autism,
anxiety, and addiction. The seailments are not
neurodegenerative diseases that destroy part of the
neuralcircuitry. So, if the defective circuits could be
stimulated in the right way, the brain could develop
normally.

43. According to the passage, the pirate look ----..

A) has been a disorder common in the world

B) is a video game for children

C) is a neurodegenerative disease

D) cannot be a cure as it cannot be detected at an


early age

E) is the treatment of amblyopia in children

44. According to the passage, the writer ----..

A) does not believe that new studies on amblyopia


make any contribution to the treatment of this
disorder

B) points out that through the new studies on


amblyopia, other mental disorders could be
understood

C) strongly believes that schizophrenia, epilepsy,


autism, anxiety, and addiction in children cannot
be treated past the critical age

D) suggests that pirate look speeds up the loss of


vision in children

E) argues that the neural circuitry is completely


ruined by mental diseases

45. According to the passage, the lazy eye ----..

A) is enabled by the pirate look to work harder

B) is a disorder that results from neurodegenerative


diseases

C) is observed in a large amount of the world


population

D) becomes much worse when a child plays video


games most of the time

E) has been a major medical concern among


physicians
49. The author points out that, along with the
47. - 50.sorular aadaki paraya gre
institution of a multiparty democracy, ----..
cevaplaynz.

A) both Mozambique and South Africa have focused


on their past and begun to bring to justice those
When Mozambique and South Africa ended their internal who are accountable for the crimes of the past
conflicts in the early 1990s, they enacted wide scale
amnesties, and in both countries the rule of law quickly B) the maintenance of the rule of law in Mozambique
improved. In each of them, political leaders opted to move and South Africa has failed due to a growing rate of
past the violence and injustices of the past and to focus political misconduct
on the tasks of social and political reconstruction. As part
of that reconstruction, each country became a multiparty C) one important change in Mozambique and South
Africa in the late 1990s was the new political
democracy in which the accountability of leaders and
principle that leaders were to be held accountable
other key norms of the rule of law could finally take root. for their policies
The restoration of public security, meanwhile, allowed the
provision of basic services. And though their criminal- D) violence and injustice in Mozambique have ceased
justice systems remained woefully underfunded, both completely while South Africa still struggles with
were finally able to start providing citizens with basic them
protections. While the legal, social and political
improvements in South Africa between 1994 and 2004 E) the question of public security has been of little
were impressive, in poorer Mozambique, the improvement concern for political leaders since they have
focused their efforts on party politics
was smaller but still marked.

50. It is emphasized in the passage that, in the post-


amnesties period, ----..
47. It is clear from the passage that both South
Africa and Mozambique ----..
A) radical political reforms in Mozambique are the
major reason for the impressive improvements in
A) underwent a very radical process of social, political the countrys economic performance
and legal transformation over a decade after the
mid-1990s B) Mozambique, which is economically
underdeveloped, has experienced some striking
B) are still extremely obsessed with their political past political improvements
and have failed to adapt themselves to the norms
of democracy C) political improvements in South Africa became the
model for political improvements in Mozambique
C) are noted for their indifference to the rule of law
and have introduced a series of non-democratic D) Mozambique has excelled South Africa in the
measures improvement of both its economy and political
institutions
D) have not solved their internal conflicts fully
although they have made their criminal-justice E) Mozambiques political leaders were concerned not
systems very efficient through radical reforms only with political reforms but also, more
importantly, with the economic well-being of the
E) have resorted to every conceivable means in their country
efforts to restore public security

48. It is pointed out in the passage that, compared


with Mozambique, South Africa has ----..

A) never been genuinely enthusiastic about the rule of


law in the country and invariably attempted to
undermine it

B) upgraded their system of criminal justice and, thus,


completely eliminated violence and injustice in
society

C) given priority to the cessation of internal conflicts


in the country and the establishment of various
political parties

D) made enormous progress in the social and political


reconstruction of their country

E) never been held accountable for the violence and


injustices they caused in the past
53. In the passage, the writer points out that,
51. - 54.sorular aadaki paraya gre
paradoxically, ----..
cevaplaynz.

A) even if America did not want the Soviet Union to


capture Berlin completely, it acted fast and moved
Today, the Berlin Crisis of 1961, in which the Soviet Union into West Berlin
demanded that Western powers cut their ties with Berlin,
may be the most forgotten crisis in the annals of the Cold B) since the Allies and the Soviet Union fought in
War. Even most Berliners who lived through the event World War II against Nazi Germany, in fact they
remember little about it. Yet this crisis over Berlin brought cooperated in many respects, including the capture
America and the Soviet Union, the two superpowers of the of Berlin
post-World War II period, close to war. In fact, since the
C) although the 1961 Berlin crisis almost led to a
very end of World War II, Berlin had been the centrepiece
military confrontation between America and the
of a struggle between these two superpowers. It was here Soviet Union, for most residents of Berlin today it
that World War II ended in 1945 when, following the has lost its significance
occupation and defeat of Nazi Germany by the allied
armies of the United States, Britain, and France from the D) despite their alliance during World War II, the
west, and the Soviet Union from the east, the city had United States, Britain and France each had
been captured and divided into the separate zones of East conflicting policies towards Nazi Germany
and West Berlin. Soon afterwards Germany had itself split
into East and West, and the border between the two had E) while Berliners experienced many hardships under
the Soviet occupation of their city, they did not
become the dividing line (the so-called iron curtain)
show any resistance
between Soviet controlled Eastern Europe and free,
capitalist Western Europe. Thus, situated behind this iron
curtain and stuck a hundred miles inside Eastern territory, 54. The point is made in the passage that, during the
West Berlin was claimed, protected and supplied by the early decades of the Cold War, ----..
Western powers. In 1948, Stalin imposed a blockade,
cutting West Berlin off from its Western suppliers. The A) the United States and its allies were totally
United States responded with an airlift, keeping the zone indifferent to the political implications of the
alive for more than 300 days before Allied access was socalled iron-curtain
restored.
B) the United States and the Soviet Union were locked
in political conflict over Berlin

51. As one learns from the passage, West Berlin ----.. C) the Soviet Union followed a policy of peace and
friendship towards the United States and its allies

A) received so much American aid during the Cold D) Berliners regarded the Allied and Soviet occupation
War that its people enjoyed a comfortable life of their city as a relief from the Nazi regime

B) was recognized by the Soviet Union during the Cold E) Berliners regarded the Allied and Soviet occupation
War as a separate American territory of their city as a relief from the Nazi regime

C) was, during the Cold War, far more prosperous and


secure than East Berlin

D) made an alliance during the Cold War with the


United States against a possible Soviet invasion

E) was able to survive the Soviet blockade in 1948


through relief provided by the United States

52. It is clear from the passage that the fall of Berlin


in 1945 to the Allies and the Soviet Union ----..

A) resulted from the establishment of the so-called


iron curtain between East and West Germany

B) was one of a series of political crises, the most


serious of which was the one in 1961

C) followed the division of Germany into East and


West

D) clearly represented the final defeat and collapse of


Nazi Germany

E) completely undermined Nazi Germanys political


and military ambitions to conquer Europe
57. It is clear from the passage that toothed whales-
55. - 58.sorular aadaki paraya gre
---..
cevaplaynz.

A) can hardly find where their prey is located


Baleen whales and toothed whales each have a unique B) navigate through the echoes of the sounds they
way of vocalizing. Only baleen whales produce long make
sequences of deep sounds known as whale songs. They
have a larynx, an organ at the top of the trachea, which C) often come up to the sea surface to inhale enough
may be involved in sound production. Researchers are air
unclear about the organs role in the songs as whale
larynxes are unlike those of humans, which have vocal D) generally swim close to the seabed while they are
hunting
chords. Toothed whales, on the other hand, rely on
sequences of high-pitched clicks and whistles for both E) are better hunters than baleen whales, especially in
echolocation and communication with their mates. Their deep waters
phonic lips, a structure analogous to human nasal
passages, press together when air is forced through them,
vibrating the surrounding tissue. The sound waves then
penetrate an oily organ in the whales head, called the
'melon', where they are focused in to a beam of sound.
When this beam strikes a fish, the seabed, or another
object, the sound is reflected back to the whale as an
echo. Toothed whales canthus locate prey and navigate in
total darkness.However, during their long, deep dives,
toothed whales cannot inhale air every time they want to
produce a sound. So they collect it in a sac at the back of
their head and reuse it.

55. It is pointed out in the passage that there is


some uncertainty as to ----..

A) why both baleen and toothed whales use sounds in


order to find their ways

B) whether toothed whales can travel long distances


in the sea without inhaling fresh air

C) how a baleen whales phonic lips function in the


production of sound

D) how baleen whales produce sound

E) how far whale songs travel in the sea when whales


communicate with their mates

56. According to the passage, baleen whales are


different from toothed whales because they ----..

A) use their trachea to produce sounds

B) communicate with other whales through whistles

C) can dive to immeasurable depths and find their


ways in full darkness

D) are much more efficient in locating feeding areas in


the sea

E) produce what is called the whale song


58. The passage ----..

A) doesnt explain clearly how toothed whales vocalize

B) gives a full account of the study researchers have


made of baleen whales

C) focuses more on toothed whales than on baleen


whales

D) points out the similarities of sound production in


whales and human beings

E) describes in detail how toothed whales hunt in total


darkness
61. It is clear from the passage that natural spider
59. - 62.sorular aadaki paraya gre
silk is produced ----..
cevaplaynz.

A) in large quantities for use in the production of


various materials
Scientists are exploring ways of producing spider silk
artificially, a process difficult to repeat effectively. A B) only when the spiders silk gland has stored enough
spiders silk gland is a very efficient chemical liquid proteins
factory.Inside its gland, the spider stores a mixture of
liquid proteins, which it is able to transform into light, C) shortly before the spiders silk gland undergoes a
strongfibres. Artificial spider silk could have many chemical reaction
applications, from lightweight and durable packing
D) through the transformation in the spiders silk
materials to parachutes, surgical sutures, and even bullet-
gland of liquid proteins into fibres
proof vests. Producing it synthetically is a two part
process: scientists must first manufacture the proteins E) even though the spiders silk gland fails to have an
and then find a way to form them into superfine threads. adequate amount of liquid protein mixture
They have had success with the first part, by producing
proteins through genetic modification. Binding proteins
into fibres as thin and strong as spider silk, however, has
proved to be a challenge. Recently, however, a group of
German scientists have attempted to solve that problem
by using a device modelled on a spiders glands. Like the
arachnid method, the proteins are mixed with potassium
phosphate, and then the pH is lowered before pressure is
applied as the mixture flows through tiny channels,
hardening and binding the proteins. So far, researchers
have been able to makefibres of only a fraction of an inch
long, but they hope to be able to produce longer, stronger
fibres in the future.

59. As stressed in the passage, with regard to the


production of artificial spider silk, the main
problem is ----..

A) the difficulty in mixing proteins with potassium


phosphate

B) that scientists are not seriously interested in it

C) how to make fibres as fine and long as natural


spider silk

D) whether the material can be used for various


purposes

E) whether it can be commercially and technologically


viable

60. According to the passage, artificial spider silk -


---..

A) could have various uses in industry

B) is produced only in chemical factories

C) is commonly known as a very profitable product

D) is a material that has already had a wide range of


applications

E) is currently being produced in threads longer than


an inch
62. It is pointed out in the passage that, in making
synthetic silk, scientists have so far been
successful in ----..

A) imitating the entire natural process of silk


production

B) producing fibres suitable for parachutes and bullet-


proof vests

C) understanding the importance of proteins

D) producing artificial fibres about an inch in thickness

E) obtaining the necessary proteins through genetic


modification

63. - 67.sorularda, karlkl konumann bo


braklan ksmn tamamlayabilecek ifadeyi
bulunuz.

63. Jane : Did you know that women's spines curve


up to 60% more by the end of their pregnancy
than before? John : ----. Jane : Such realignment
could stress their backs, but womens spines are
stronger because they have larger joints that go
farther down the spine than in men. John : Well, I
guess it's lucky that mens backs don't need to
support the weight of a baby..

A) How can pregnant women support such weight?

B) Researchers believe that these adaptations are


millions of years old.

C) Adaptations such as this reduce strain on the back.

D) How can pregnant women stand upright with such


pressure on their backs?

E) Doesn\'t that damage the spines of the pregnant


women?
64. Sam: I'm planning to have a holiday at a resort in
the Caribbean, but people say every few years
this spot is hit by a tropical storm. What should I
do? Tom: Officially, the Atlantic hurricane season
is from June 1 to November 30, but Mother
nature isn't reading calendars. Are you a risk
taker? Sam:----. Tom: You'd better pick another
destination then, because every once in a while a
tropical cyclone hits out of season..

A) Perhaps I should do a cruise instead, it would be


much safer.

B) There is nothing magical in these dates and


hurricanes have occurred outside of these six
months.

C) Not at all! I\'m not going anywhere near the


hurricane zone during peak months.

D) I think the odds of enjoying fine weather are very


good.

E) OK, I\'ll go but I\'ll be careful. I\'ll try to reduce the


risk by taking out insurance.
65. pek: What do you think about the Human Brain 66. Jenny: What are you reading? It's made you
Project? annoyed!
Norman: ----
Handan: ---- Jenny: And presumably nothing is being done
about it.
pek: Youre right. Scientists want to build a Norman: Nothing at all. But two superfluous
completely simulated human brain, but what sports centres are being constructed there..
about the obligations to the mind that has been
created?
A) I was reading about the new inner city traffic
Handan: Yes, imagine a scenario where a brain regulations.
has been created that has memories and is really
humanlike, but then the scientists decide to B) Yes, I suppose it does. Actually more sad than
switch it off.. annoyed.

C) True Theyve just caught some children of twelve


A) I agree that its a unique opportunity for us to learn taking drugs.
how the brain really works.
D) Indeed it has. The number of homeless people in
B) I think it has taken scientists far too long to come the capital now exceeds 5,000 and continues to
up with this idea. increase.

C) I dont really know enough about it. But Im sure E) Precisely. It says every effort is being made to
there must be some serious ethical considerations. ensure that the children\\\'s every need is being
met.
D) Im not very certain, as I havent read much about
it. Whats your opinion?
67. James: Have you been to the exhibition of
E) Id really like to know more about it before I say Ottoman manuscripts yet?
anything about the morality of the issue. Robin: Yes, last Sunday.
James: What did you think of it?
Robin: _____.

A) Some of my colleagues from the office are


interested in Ottoman culture.

B) Really! I really must find time to see it.

C) Fascinating! I only wish I could have understood


something of it!

D) Obviously the Ottomans had an efficient


administrative system.

E) By the way, you said it was open till 6 oclock and


they closed the doors at 5 oclock.
68. Meals at 'The Round Table' are undoubtedly the
best in town and prices are reasonable..

A) Considering the prices, the quality of food at 'The


Round Table' is not satisfactory.

B) At 'The Round Table' you get extremely good food,


but you certainly, pay for it.

C) Prices have gone up at 'The Round Table' but the


quality of the food has gone up accordingly.

D) They dont overcharge you at 'The Round Table'


and the food there is unrivalled in the town.

E) Considering how much they charge the food they


serve at 'The Round Table' is not as good as it
ought to be.
69. Experienced athletes and artists often complain
that they have lost touch with what made them
love what they do..

A) More and more experienced athletes and artists are


having issues about the areas that they are
passionate about

B) Most of the time, it is the less experienced athletes


and artists who stay connected to what they like to
do.

C) Losing interest in what enabled them to enjoy what


they were doing is a common complaint among
experienced athletes and artists.

D) Not being able to do what they have experience in


usually causes athletes and artists to complain.

E) Although experienced athletes and artists may


sometimes lose their excitement about what they
do, they still keep doing it.

70. 'The Rocket' incorporated the principal features


that dominated locomotive design from that time
on wards..

A) Later locomotive design did not differ noticeably


from that of \'the Rocket\'.

B) Many of the main features of \'the Rocket\' were to


reappear in later locomotive design.

C) The main features affecting all future locomotive


design were present in \'the Rocket\'.

D) Later locomotive design was in certain essentials


not dissimilar from that of \'the Rocket\'.

E) As regards the principal features of its design, \'the


Rocket\' was not dissimilar from later locomotives.

71. So many books have been written about Einstein


that you might expect there would be little left
to say..

A) You might think that Einstein is such a person that


needs to be the subject of many more books.

B) After hundreds of books have been written about


Einstein, you might expect there is still much more
to say.

C) Given the high number of books published on


Einstein, one would think there is hardly anything
left to say about him.

D) Seeing what has been said about Einstein, one is


forced to think what else is left to write.

E) The fact that so many books have been written


about Einstein leaves very little room for you to
write anything else about him.
72. - 75.sorularda, bo braklan yere, parada
anlam btnln salamak iin
getirilebilecek cmleyi bulunuz.

72. Poetry is one of the oldest forms of literature. -


---.They used rhythm and rhyme to help them
remember the stories better. Ballads are a good
example of this, for they are stories in poetic
form that were sung..

A) Before literature was written down, people told


stories

B) Each word and phrase in this poem is chosen with


great care

C) In poems, language is used in unusual and creative


ways

D) Prose is the language used in everyday life

E) Even so, there are certain basic similarities


between present-day poetry and that of the past

73. The importance of diet in cancer prevention is


pointed out in many reports. Soya is an
important component of a diet to prevent breast
cancer. Certain components in soya, the
isoflavones, may affect the hormone receptors in
a way that prevents cancer. ----. It has long been
recognized, for example, that Japanese women
have a very low breast cancer risk..

A) Soya can be consumed as soya milk, soya sauce, or


processed into textured vegetable protein.

B) Isoflavones act through these receptors.

C) There is much lower breast cancer rate in those


cultures eating a diet high in soya protein.

D) Our diet should be full of whole grains, fruits and


vegetables.

E) Soya bean, originally grown as a food crop for


animals, is increasingly used for human
consumption.
74. In its first case concerned with global warming, 75. When we talk about the shared customs of a
the US Supreme Court ruled in April that society, we are referring to a culture.
greenhouse gases such as carbon dioxide are air However, not all things shared generally by a
pollutants which the Environmental Protection population are cultural. For example, the typical
Agency (EPA) can regulate. As a consequence, hair colour of a population is not cultural
experts agree that greenhouse emissions from because it is genetically determined. ----..
automobiles and possibly power plants will face
regulation. The debate will now focus on how
strict or flexible those rules will be. ----. A) Culture has nothing to do with playing the piano or
Moreover, the agency also stated that even if it reading poetry
did have the power to regulate these gases, it
would not do so.. B) Culture refers to all these aspects of life

C) What and how human beings eat is learned and


A) If the agency were to refuse, there would be a varies from culture to culture
lawsuit against it
D) Every society has a culture, no matter how simple
B) However, the EPA had long claimed to have no this culture is
authority in regulating these gases
E) For something to be considered cultural, it must be
C) Siding with the EPA were several industry groups learned as well as shared
and ten states, a number of which rely heavily on
coal, electricity or motor vehicle production

D) The Supreme Court ruling may represent a


milestone in the legal battle over climate change

E) Notably, 12 states had been seeking waivers from


the Agency permitting them to pass laws requiring
reduced car emissions of greenhouse gases from
the 2009 model year onward
76. - 80.sorularda, cmleler srasyla
okunduunda parann anlam btnln
bozan cmleyi bulunuz.

76. (I) Life in Europe in the seventeenth and


eighteenth centuries was largely shaped by the
combined effects of war, trade, and a steadily
growing population. (II) Especially the practice of
free trade led to the establishment of overseas
colonies and opened up new markets for
European industry. (III) Additionally, agricultural
productivity increased, making it possible for
Europe to feed a population that had now
reached unprecedented levels. (IV) The dominant
artistic school of southern Europe from about
1600 until the early 1700s was that of the
Baroque, a school not only of painting but of
sculpture and architecture. (V) Moreover,
population growth in turn enabled European
governments to wage more frequent wars and
employ larger and larger armies..

A) I

B) II

C) III

D) IV

E) V

77. (I) The label homo sapiens was first attached to


man by Linnaeus in his classification of the
animal kingdom over two hundred years ago. (II)
That kingdom is now thought to include over
three quarters of a million species. (III) Still, the
physiology of the human body parallels, in a host
of different ways, that of the animals. (IV)
Though very many more species may be
discovered, it is not likely that anything will ever
shake our conviction that we belong to a very
special class. (V) This conviction is quite as
strong today as it was in the eighteenth century..

A) I

B) II

C) III

D) IV

E) V
78. (I) Antibiotics are drugs that disable or kill
infectious microorganisms. (II) However, for
nearly every antibiotic that has been developed,
a resistant strain of bacterium has appeared
within a few decades. (III) Penicillin, for example,
was originally isolated from a mold and has been
widely prescribed since the 1940s. (IV) A
revolution in human health rapidly followed its
introduction, rendering many previously fatal
diseases easily curable (such as strep throat and
surgical infections). (V) During the 1950s, some
doctors even predicted the end of human
infectious diseases altogether..

A) I

B) II

C) III

D) IV

E) V

79. (I) In the film Amistad, Steven Spielberg


expresses his outrage at the institution of
slavery. (II) That is, he makes something that
enables the film audience to feel the same kind
of outrage toward slavery that he feels. (III)
Clearly, the artist at the start felt deeply about
slavery. (IV) He then set out to give to his
audience a similar feeling. (V)On the contrary,
these configurations stimulate differing
emotional states in the audience..

A) I

B) II

C) III

D) IV

E) V

80. (I) Aristotle explored the apparent ties between


odour and memory in his work On Sense and the
Sensible. (II) Since then, people have speculated
that the memories elicited by smell are more
intimate and immediate than other recollections.
(III) When we experience certain smells, we often
find ourselves taken back in time to a specific
event or scene. (IV) Many movies of the 1980s
include scenes that trigger memories of
childhood and school years. (V) For example, the
smell of a salsa, a sauce eaten with Mexican
food, may remind a person of watching James
Bond movies on television with his or her father
while dipping chips in the spicy sauce..

A) I

B) II

C) III

D) IV

E) V
SORU CEVAP SORU CEVAP

1 B 41 A

2 C 42 B

3 C 43 E

4 D 44 B

5 A 45 A

6 C 46 C

7 A 47 A

8 E 48 D

9 C 49 C

10 B 50 B

11 D 51 E

12 E 52 D

13 E 53 C

14 C 54 B

15 B 55 D

16 B 56 E

17 B 57 B

18 D 58 C

19 E 59 C

20 A 60 A

21 B 61 D

22 B 62 E

23 D 63 E

24 A 64 C

25 D 65 C

26 E 66 D

27 D 67 D

28 B 68 D

29 D 69 C

30 A 70 E

31 C 71 C

32 D 72 A

33 B 73 C

34 D 74 B

35 B 75 E

36 E 76 D

37 E 77 C

38 E 78 B

39 B 79 E

40 C 80 D
25
6. The article ---- that Martin Luther
1. - 16.sorularda, cmlede bo braklan yerlere
King's fundamentalist views were foreign to
uygun den szck ya da ifadeyi bulunuz. his liberal allies..

A) holds up
1. Insight into the ---- by which the human brain
came to exist and acquire its remarkable abilities B) deals with
can be gained by studying animals that are far
simpler than us.. C) pulls through

D) points out
A) outcome
E) writes out
B) process

C) response 7. Doctors who had been studying longevity ---- an


assortment of genes that ---- life span in
D) result different organisms..

E) task
A) will identify / had influenced

2. Earthquakes can do ---- damage to buildings, B) have identified / will be influencing


bridges, pipelines, railways, embankments, and
other structures.. C) identified / will have influenced

D) had identified / could influence


A) significant
E) identify / could have influenced
B) satisfactory

C) objective 8. Despite the fact that computer misuse ---- as


computer use becomes more widespread, no one
D) exclusive ---- that computers make life easier with their
increasing efficacy..
E) incentive
A) is growing / can deny
3. In a recent press conference, the company ----
their plans to launch a new hybrid car in B) has been growing / denied
autumn..
C) had grown / denies

A) originated D) grew / will deny

B) revealed E) was growing / has denied

C) qualified
9. Discrimination and prejudice are both similar, ----
D) advised they are both based on stereotypes..

E) contributed
A) for

4. The bolero is considered a ---- easy dance to B) just as


learn, yet difficult to master properly. .
C) hence

A) strongly D) in order that

B) probably E) lest

C) merely
10. ---- suggestion he may make as regards the tax
D) relevantly rate, lets not give any response ---- we have
discussed the matter thoroughly together..
E) fairly
A) However / when
5. Apparently the drop in farm incomes had nothing
to ---- the introduction of modern B) Whatever / until
farming methods..
C) Whyever / after

A) close down D) Whatsoever / while

B) make out E) Whenever / before

C) sort out

D) force out

E) do with
11. In the 19th century, one could grasp a field with 16. The writers of this article seem to assume
a little reading, ---- in these days, breaking new that nuclear plants conform with safety
ground in science and literature is much harder requirements ---- the fact that violations are
and more demanding.. constantly being reported..

A) but A) in case

B) so B) although

C) as C) against

D) when D) concerning

E) once E) despite

12. Even if India ---- China in growth rates based on


IMF's projections, it ---- at least till 2032 to match
China's GDP in real terms..

A) had surpasses / has taken

B) will surpass / takes

C) has surpassed / took

D) surpassed / had taken

E) surpasses / will take

13. People in England have been writing books about


Turkey at least since the 15th century; but ----
were written in the 19th century..

A) the few

B) the least

C) the best

D) fewest

E) best

14. ---- anxiety, depression is the most common


psychiatric disorder and typically begins ---- the
20s, 30s or 40s..

A) In / through

B) Over / during

C) After / in

D) Through / by

E) By / for

15. An intracerebral haemorrhage begins abruptly -


--- a headache, followed ---- signs of steadily
increasing neurologic losses..

A) at / through

B) from / in

C) after / on

D) in / up

E) with / by
21. V.
17. - 21.sorularda, aadaki parada
numaralanm yerlere uygun den szck ya
da ifadeyi bulunuz. A) progress

B) blend

Capitalismis an economic and political system in which C) rest


individuals own economic resourcesand industry, (I) ----
under socialism,the state plans and produces goods, and D) advantage
either owns or redistributes resources (II) ---- its citizens.
In a capitalist economy,the political system emphasizes E) relationship
competition for resources (III) ---- increasing capital, or
wealth, anddeveloping personal success. In a socialist
economy, the emphasis is (IV) ---- distributing wealth so
that individualneeds are met with collective capital. There
are many different versions ofboth capitalism and
socialism, and most modern societies are a (V) ---- of the
two.

17. I.

A) because

B) unless

C) instead

D) whereas

E) as

18. II.

A) among

B) into

C) of

D) on

E) from

19. III.

A) in order to

B) in case of

C) as a means of

D) not only

E) with the help of

20. IV.

A) under

B) by

C) from

D) off

E) on
22. - 26.sorularda, aadaki parada 27. - 36.sorularda, verilen cmleyi uygun
numaralanm yerlere uygun den szck ya ekilde tamamlayan ifadeyi bulunuz.
da ifadeyi bulunuz.

27. Despite the fact that short-term interest rates


The poet Robert Browning was lucky in his parents.
have marched steadily upward since the early
Hisfather, who (I) ---- in a bank, wasa man (II) ---- 2000s, ----..
genuineintelligence, who owned a library of six thousand
volumes and seems (III) ---- them all. His mother, (IV) ----
was a devoutly religiouswoman, gave Browning a A) the result will be a sharp slowdown in consumer
tenderness and optimism he would need (V) ---- in life. spending

B) Americas overall inflation hit 4.7% in recent years

C) the cost of a tank of petrol would have fallen


22. III.
dramatically

A) reading D) they are still negative in real terms

B) to read E) Americans have been able to borrow more and


save even less
C) to have read
28. Unless you have Turkish citizenship, ----..
D) to be reading

E) have read A) you cannot be regarded as a foreigner

B) you can permanently reside and work


23. IV.
C) you are obliged to vote in Turkish general elections
A) which
D) you will be registered in Turkey as a voter after 18
B) who years of age

C) whose E) you will not be eligible to vote in the elections

D) that
29. Before transplants are performed, ----..
E) whom
A) these immunosuppressive drugs also make the
24. II. transplant patient more vulnerable to pneumonia

B) most organs to be transplanted, therefore, are


A) from removed from patients who have just died

B) about C) corneal transplants are highly successful because


the cornea has almost no blood and lymphatic
C) for vessels

D) in D) graft rejection is an immune response against


transplanted tissue
E) of
E) tissues from the patient and the potential donors
must be typed and matched as closely as possible
25. V.

30. When colorectal cancer has spread and isnt


A) never likely to be cured by surgery alone, ----..

B) hardly
A) chemotherapy after surgery may prolong the
C) scarcely persons life, but cure is still rare

D) frequently B) surgery to relieve the intestinal obstruction may


ease the symptoms
E) fairly
C) this treatment, though expensive, may provide
more benefit than ordinary chemotherapy
26. I.
D) this procedure may relieve symptoms and prolong
life
A) having been employed
E) about 70 per cent of patients with colorectal cancer
B) has been employed are good candidates for surgery

C) has employed

D) was employed

E) will be employed
31. ---- whether the universe will continue to expand 35. Such a health policy could be ideal for
forever.. developing countries ----..

A) From the discussions at the conference it has been A) until a better one had appeared
clear
B) if it is so expensive
B) There is evidence for the growing view
C) however practical it appeared to be
C) Many scientists have conclusively proved
D) whether it has the potential some people claim
D) A recent analysis of some galaxies has shown
E) where AIDS hits hardest
E) One important question is

32. All developed countries governments are


worried about unemployment, ----..

A) so that demand for labour is derived from the


demand for the product

B) if some employers pay good wages because of


market forces

C) and there is a good deal of international discussion


about it

D) so investment in new technology makes people


more productive

E) even if employers must be able to reduce the


number of people they employ

33. ----, journalists had to stand on their feet for over


two hours..

A) After the ceremony, the spectators left the stadium


in an orderly fashion

B) As the ceremony was too long to test the patience


of spectators in the stadium

C) While the invited guests enjoyed the ceremony


from the comfort of their seats

D) When the medias role gets the attention it


deserves from the organizers

E) Whereas Congos national team did not get any


media coverage during the event

34. ----, it has become the worlds best-selling


vehicle..

A) Even though she paid a lot of money

B) As long as more research is done to develop a new


type of vehicle

C) Just as companies have got into fierce competition


with one another

D) While most people were concerned about the rising


fuel prices

E) Since the new car recently developed by one of the


leading manufacturers is environmentally friendly
36. Primates enjoy good visual discrimination in all
lighting conditions ----. .

A) unless its cells are full of melanin granules

B) though the retina has lined the inside of the eyeball

C) after early mammals were able to generate their


own body heat

D) because they have a duplex retina

E) though their cone photometers would adapt to the


surrounding brightness

37. - 42.sorularda, verilen ngilizce cmleye


anlamca en yakn Trke cmleyi, Trke
cmleye anlamca en yakn ngilizce cmleyi
bulunuz.

37. William Butler Yeats, yazmaya adanm sakin bir


yaam srdren Thomas Hardynin tersine,
deiik trlerdeki etkinliklerle baarl bir ekilde
megul olmu ve bunlarn hepsini sanatyla
btnletirmitir..

A) William Butler Yeats engaged successfully in


different kinds of activities and integrated all of
them into his art, unlike Thomas Hardy, who lived a
quiet life dedicated to writing.

B) Although Thomas Hardy lived a quiet life dedicated


to writing, William Butler Yeats was successful in
many different activities, and integrated them all
into his art.

C) William Butler Yeats, unlike Thomas Hardy, did not


lead a quiet life dedicated to writing, but
participated in different kinds of activities, all of
which he included in his art.

D) While Thomas Hardy preferred to live a quiet life,


dedicated to writing, William Butler Yeats wanted
nothing of the sort, and therefore engaged in
different kinds of activities, integrating them
successfully into his art.

E) Engaging successfully in different kinds of


activities, William Butler Yeats was able to
integrate them into his art, whereas Thomas Hardy
preferred to live a solitary life dedicated to writing.
38. On dokuzuncu yzyldan beri tutulan kaytlar,
atmosferdeki karbondioksit miktarnn arpc bir
ekilde arttn aka gstermektedir..

A) The records kept so far clearly show that the


amount of carbon dioxide in the atmosphere has
reached a dramatic level since the nineteenth
century.

B) As the records kept since the nineteenth century


clearly show, there has been a dramatic increase of
carbon dioxide in the atmosphere.

C) Records have been kept since the nineteenth


century to show clearly that the amount of carbon
dioxide in the atmosphere has been dramatically
increasing.

D) It is clearly shown by records kept since the


beginning of the nineteenth century that the
amount of carbon dioxide in the atmosphere has
been dramatically increasing.

E) Records kept since the nineteenth century clearly


show that the amount of carbon dioxide in the
atmosphere has increased dramatically.
39. Tmr bymesine neden olan kanser kk 41. A study made by an American research centre
hcrelerinin varl, eitli kan kanseri ve tmr has found that the Internet is the third most
trlerinde tespit edilmitir; ancak, bu habis kk popular source for news among Americans,
hcrelerinin nasl olutuu hl belirsizdir.. behind local and national television..

A) It has been shown that a wide range of blood A) Amerikallar arasnda, nternetin, yerel ve ulusal
cancer and tumour types result from malignant televizyonun ardndan en sk yararlanlan nc
stem cells whose properties are not yet fully haber kayna olduunun, bir Amerikan aratrma
known. merkezince belirlenmesinden sonra, internetin
yaygnl artmtr.
B) Cancer stem cells that cause tumour growth have
been discovered in a number of blood cancer and B) Bir Amerikan irketinin yapt aratrmaya gre
tumour types, but it is not yet clear why they Amerikan toplumunun nc en gvenilir haber
become malignant. kayna, yerel ve ulusal televizyonun ardndan
gelen nternettir.
C) Different blood cancer and tumour types are
caused by cancer stem cells, although there is no C) Bir Amerikan aratrma merkezinin aratrmas,
evidence to show how these malignant stem cells yerel ve ulusal televizyonun, en ok tercih edilen
come into being and drive tumour growth. nc haber kayna olan nternetin nnde
olduunu ispat etmitir.
D) Even though it is not yet certain how malignant
stem cells form, they are often found in various D) Bir Amerikan aratrma merkezince yaplan
blood cancer and tumour types, causing them to alma, Amerikan halk arasnda, nternetin, yerel
grow. ve ulusal televizyondan sonra, nc en yaygn
haber kayna olduunu ortaya karmtr.
E) The existence of cancer stem cells that drive
tumour growth, has been established in several E) nternetin en sk bavurulan nc haber kayna
blood cancer and tumour types, but how these olmasna ramen, yerel ve ulusal televizyonu
malignant stem cells arise is still uncertain. geemedii, bir Amerikan aratrma merkezince
kantlanmtr.

40. Sweden, which occupies the eastern part of the


Scandinavian peninsula, is the fourth-largest
country in Europe..

A) sve, hem skandinav Yarmadasnn dou


kesiminde yer alr hem de Avrupann en byk
drt lkesinden biridir.

B) Avrupada drdnc en byk lke olan sve,


skandinav Yarmadasnn dou kesiminde yer
almaktadr.

C) Dou kesimini svein kaplad skandinav


Yarmadas Avrupann drdnc en byk
blgesidir.

D) Avrupann drdnc en byk blgesi olan


skandinav Yarmadasnn dousunda sve yer
almaktadr.

E) skandinav Yarmadasnn dou ksmn kaplayan


sve, Avrupada drdnc en byk lkedir.
42. The tablets of the Epic of Gilgamesh were first
found in the 19th century among the ruins in
Nineveh, which was once the capital of the
Assyrian Empire..

A) Glgam Destannn tabletleri ilk kez 19. yzylda,


bir zamanlar Asur mparatorluunun bakenti olan
Ninevehteki harabeler arasnda bulunmutur.

B) Arkeologlar Glgam Destanna ait ilk tabletleri 19.


yzylda, eskiden Asur mparatorluunun bakenti
olan Ninevehteki harabeler arasnda bulmulardr.

C) Eskiden Asur mparatorluunun bakenti olarak


bilinen Ninevehteki harabeler arasnda 19. yzylda
bulunan tabletlerin Glgam Destanna ait olduu
anlalmtr.

D) 19. yzylda Glgam Destannn ilk tabletleri,


arkeologlarn almalar srasnda, eskiden Asur
mparatorluunun bakenti Ninevehteki harabeler
arasnda bulunmutur.

E) Glgam Destanna ait olduu sanlan ilk tabletler,


bir zamanlar Asur mparatorluunun bakenti
Ninevehteki harabeler arasnda arkeologlar
tarafndan 19. yzylda bulunmutur.
45. It is suggested in the passage that drug
43. - 46.sorular aadaki paraya gre
deprivation ----..
cevaplaynz.

A) usually results in an increased level of metabolic


activities
Many drugs, whether prescribed or abused, affect the
nervous system. While about 25% of all prescribed drugs B) makes the user more aggressive and intolerant
are taken to alter psychological conditions, almost all the towards others
commonly abused drugs affect mood. In particular, levels
of serotonin and dopamine arethought to influence mood. C) increases instantly the sense of tolerance in the
For example, when excessive amounts of norepinephrine user
are released, people feel energetic and stimulated, where
D) stimulates the production of enzymes that
as low concentrations of this neurotransmitter
contribute to the users psychological well-being
reduceanxiety. Habitual use of almost any mood-
alteringdrug can result in psychological dependence, in E) leads to psychologically and physically negative
which the user becomes emotionally dependent on the effects
drug. When deprived of it, the user craves the feeling of
euphoria (well-being) that the drug induces.Some drugs
46. According to the passage, drug tolerance ----..
induce tolerance after several weeks.This means that
response to the drug decreases, and greater amounts are
required to obtain the desiredeffect. Tolerance often A) is a temporary case that can be treated effectively
occurs because the liver cellsare stimulated to produce
more of the enzymes that metabolize and inactivate the B) is on the whole limited to the use of heroin and
drug. Use of some ofthe drugs, such as heroin, tobacco, alcohol
and alcohol, mayalso result in addiction (physical
C) means that the more a drug is used, the more the
dependence), in which physiological changes occur that
need for it increases
make the user dependent on the drug. Addiction can also
occurbecause certain drugs, such as morphine, have D) brings about significant change in the structure of
components similar to substances that body cellsnormally body cells
manufacture on their own. The continueduse of such a
drug causes potentially dangerous physiological effects. E) generally has known adverse effects on the users
mood

43. It is indicated in the passage that a regular use


of drugs ----..

A) makes the user feel dizzy and exhausted

B) has no effect on the levels of the hormones

C) may result in addiction which greatly affects the


psychology of the user

D) usually increases anxiety in the user

E) can be toxic and lead to a number of psychological


disorders

44. According to the passage, the cause of the


dependence on drugs ----..

A) is the fact that they can be easily purchased

B) is the excessive release of some hormones which


modify the psychology of the user

C) is generally related to the users background

D) can be understood through a comprehensive study


of various drug cases

E) can be explained with reference to the stability of


enzymes
49. As one learns from the passage, it was before
47. - 50.sorular aadaki paraya gre
1721 that ----..
cevaplaynz.

A) the Russians re-captured Riga from the Swedes and


made it the capital of Latvia
Narva is a quiet northeastern Estonian town bathed in sea
breezes. Though small, with a population of just over 72, B) Russia reached an agreement with Austria and
000, it occupies a large place in Russian history. It was Prussia for the partition of the whole Baltic region
here in 1700 that, by attacking the Swedes, who were
then in control of much of the Baltic coast, Russia C) Russia became a major European power which
launched its final campaign in a centuries-long quest to completely dominated the Baltics
become a European power. The battle ended in defeat for
D) the Swedes and the Russians agreed on their
the Russians, but the war did not; by 1721 Russia had
respective control of the Baltic territories
conquered the Baltic territories as far southwest as Riga,
the capital of present-day Latvia, and had built a new E) the city of Saint Petersburg was founded by the
capital, Saint Petersburg, on the Gulf of Finland. Later in Russians on the Gulf of Finland
that century, Russia, through a partition agreement with
Austria and Prussia, gained control of the rest of the
Baltics, and would retain them until the fall of the Soviet
Union in 1991.

47. According to the passage, the Russian conquest


of the Baltics ----..

A) forced both Austria and Prussia to give up their


centuries-old claims on the Baltic territories and
sign a treaty

B) was finalized in the eighteenth century partly


through war and partly through a multi-lateral
treaty

C) included only the region between Narva in


northeastern Estonia and Saint Petersburg on the
Gulf of Finland

D) was several times resisted by the Swedes, whom


the Russians were unable to defeat and drive out of
the region

E) involved a very long war with the Swedes on the


one hand, and with Austria and Prussia on the other

48. As can be understood from the passage, the


writer ----..

A) points out that Narva is an attractive and


wellknown centre for tourism in Estonia

B) describes the hardships which the Baltic countries


endured under the Russians in the eighteenth
century

C) stresses how the Russian expansion in the Baltics


in the 1720s was prevented by Austria and Prussia

D) draws a contrast between the present-day romantic


appearance of Narva and its historic significance

E) compares Riga with Narva and concludes that the


Latvian capital is historically more important
50. It is pointed out in the passage that ----..

A) Russian possession of the Baltic territories lasted


until the late twentieth century

B) Prussian control of the Baltic territories lasted


throughout the eighteenth century and after

C) no European power in the eighteenth century was


concerned about the Russian invasion of the Baltics

D) the Swedes and the Russians were constantly at


war with each other for the possession of the
Baltics

E) the Russian conquest of the Baltic territories has


always been a controversial issue in Russian history
53. It is clear from the passage that Annas study of
51. - 54.sorular aadaki paraya gre
art ----..
cevaplaynz.

A) was supported by her father, who, as a political


writer, often wrote against the cultural policies of
Set in 1941 in Leningrad, Helen Dunmores novel The Stalins regime
Siege opens with deceptively gentle scenes of Chekhovian
melancholy. After the death of her mother, 23-year-old B) was interrupted as, following her mothers death,
Anna Levin, the protagonist, gives up her artistic studies she had to take care of her family
to look after her 5-year old brother and her politically
suspect father Abraham, who, as a writer, has fallen out of C) was carried out under the most favourable
favour with Stalins cultural police. So she jumps at the circumstances, and her fame as a promising young
artist soon spread throughout Leningrad
chance to make a drawing of the retired actress Marina
Petrovna, with whom Annas father might once have had a D) was a great achievement for her, despite the fact
romantic relationship. But Annas worries about art and that her artistic creativity had not been recognized
romance are soon swept away as the Germans besiege at first
her native city. At this point, Dunmores novel transforms
abruptly as well, shifting from a romantic narrative into a E) took so long that, in the end, she gave it up to join
study of survival under most extreme hardships. Annas the people of Leningrad in their defence of the city
abundant artistic creativity is put to use providing food against the Germans
and fuel for her helpless family, and her drawing skills are
called on to sketch a neighbours starved baby so that the 54. It is pointed out in the passage that, when the
grieving mother might remember her lost child. Indeed, German siege of Leningrad began, ----..
the novel presents a striking contrast between the gentle
display of human emotions and the rude dictates of
survival under the most inhuman circumstances. A) Annas concern about her own artistic future was
totally wiped out

B) Annas father Abrahams love affair with Marina


Petrovna came to an end
51. According to the passage, Helen Dunmores
novel The Siege is mainly the story of ----.. C) Anna had already become famous because of her
portrait of the actress Marina Petrovna
A) how the children of Leningrad in particular suffered
D) Stalins cultural police arrested Abraham because
extreme hardship when the Germans besieged the
of his subversive ideas
city in 1941
E) the inhabitants made various arrangements to
B) the famous actress Marina Petrovna, who was living
overcome the serious shortage of food and fuel in
in retirement when the Germans besieged
the city
Leningrad in 1941

C) the strong armed resistance that the people of


Leningrad put up against the Germans when the
city was besieged

D) how a young artist, Anna Levin, struggled hard with


her family to survive during the German siege of
Leningrad in 1941

E) the romantic relationship between the actress


Marina Petrovna and the writer Abraham Levin,
who was a political activist

52. It is emphasized in the passage that the novel -


---..

A) never loses its romantic tone, even though some of


the scenes described can be very depressing and
upsetting

B) shows how Helen Dunmore differs from the Russian


writer Chekhov in her detailed portrayal of
characters

C) is not only a narrative of romantic scenes, but also


the depiction of a cruel reality in which human
survival was almost impossible

D) primarily focuses on the complexity of human


emotions and tries to demonstrate this through the
depiction of a wide range of characters

E) is embedded with a number of inconsistencies as


regards style and characterization, which make
Dunmores literary creativity rather superficial
57. It is stressed in the passage that Britain ----..
55. - 58.sorular aadaki paraya gre
cevaplaynz.
A) was the only colonial power in Europe that objected
to the practice of slavery
Following World War II, European countries largely gave B) was faced soon after World War II with an
up their colonial possessions and, by the 1950s and enormous influx of immigrants
1960s, had already begun to receive growing numbers of
immigrants from their former colonies. In many instances, C) had already possessed a large community of
these included the descendants of the slaves in the Africans under Roman rule
colonies, who had been forced to work. In this respect,
Britain is a case in point. Though in small numbers, D) began to receive huge numbers of colonial
immigrants from the 1960s onwards
Africans and Indians had come to Britain long before the
tens of thousands who came as colonial immigrants in the E) always welcomed large numbers of immigrants
1960s and thereafter. The first Africans who came to from its colonies in Africa and elsewhere
Britain were probably soldiers during the Roman
possession of that country in antiquity. In modern times,
especially in the eighteenth and nineteenth centuries, 58. As stated in the passage, many of the
immigrants that arrived in Europe after World
African and Indian princes and scholars visited Britain.
War II ----..
Others coming to Britain were in service positions; for
instance, in the eighteenth century, black African and
Indian young men were fashionable as servants in the A) were actually fleeing from the slave labour
homes of the wealthy. Africans and Indians also came to invariably practised in the colonies
Britain as sailors and traders, and port towns, such as
London, Glasgow, Bristol, Cardiff and Liverpool, developed B) constituted small minorities that became the target
small black populations in the early nineteenth century, of white native hatred
some of which persisted into the twentieth century.
C) were the relatives of the former colonial slaves
Relations between these populations and the native white
population were varied, historians citing instances both of D) were interested in service positions and, hence,
hostility and solidarity. were mostly employed as servants

E) were in fact the descendants of the slaves


especially in India
55. It is pointed out in the passage that the black
communities living in some British port towns in
the early years of the nineteenth century ----..

A) were not always treated in a friendly way by the


white population

B) consisted solely of sailors who came mainly from


Britains colonies in Africa

C) were completely constituted by merchants who


were involved in overseas trade with India

D) were made up of travellers from Africa and India,


who visited Britain for various reasons

E) were essentially formed by colonial immigrants


from Africa as well as India

56. As one finds out from the passage, it would be


wrong to maintain that ----..

A) many of the immigrants to Britain came from India


and the other colonies

B) Africans arrived in Britain for the first time in the


twentieth century

C) European colonial rule in Africa ended in the post-


World War II period

D) Britain was occupied and ruled by the Romans

E) the domestic staff of the wealthy in Britain often


included Africans and Indians
61. It is clear from the passage that what makes the
59. - 62.sorular aadaki paraya gre
newly-found organism unique is that it ----..
cevaplaynz.

A) lives in an environment that lacks any kind of


organic life
An organism discovered deep in the ground has taken
astrobiologists by surprise. The organisms unique ability B) can survive only through photosynthesis
to live in complete isolation from other species, or even
light or oxygen, suggests it could be the key to life on C) feeds on the remains of dead organisms
other planets. It was discovered in fluid-filled cracks in a
South African gold mine, nearly three kilometres beneath D) does not have to produce proteins
the Earths surface. When US scientists analyzed the fluid,
E) exists only in the depths of the Earth
they expected to find genes from a mix of species.
Instead, they found that 99.9 per cent of the DNA
belonged to just one bacterium, a previously unknown
species. Such a self-sufficient organism is virtually
unheard of. It means that this organism extracts
everything it needs from an otherwise dead environment.
Almost all other known organisms on the Earth that do not
use sunlight directly do use some product of
photosynthesis. However, this newly-found organism gets
its energy from the radioactive decay of uranium in the
surrounding rocks. It also has genes to extract carbon and
nitrogen from its environment, both of which are essential
for making proteins. Scientists believe that this organism
is just the type that could survive on a planet other than
the Earth.

59. As suggested in the passage, for scientists, the


newly-discovered organism ----..

A) can be useful in preventing environmental


deformation

B) is unique only to Africas ecosystem

C) can provide clues about life on other planets

D) seems to reveal the beginnings of life on the Earth

E) can survive only in an environment where there is


plenty of protein

60. According to the passage, upon the analysis of


the fluid found in a South African gold mine,
scientists have learned that ----..

A) proteins discovered in it were made through carbon


and nitrogen

B) it contained only one type of organism, unlike any


of those already known

C) the Earth and other planets have similar


environmental conditions

D) the radioactive effects of uranium can be seen very


deep down in the Earth

E) all kinds of organisms can be found not only on the


Earth but also on other planets
62. One understands from the passage that
astrobiologists ----..

A) have focused all their efforts on the search for life


on other planets

B) regard organisms as indispensable for the solution


of environmental problems

C) were astonished by the discovery of so unusual an


organism in the depths of the Earth

D) have now shifted their attention to the study of


organisms deep in the Earth

E) have been particularly interested in the DNA


structure of a large variety of organisms

63. - 67.sorularda, karlkl konumann bo


braklan ksmn tamamlayabilecek ifadeyi
bulunuz.

63. Jane: - What is meant by the 'body clock'?


Malcolm: - Well, the body adopts a routine for
waking, sleeping, eating and so on. And that's its
clock. Jane: - ---- Malcolm: - I suppose the usual
reason is when you go into a different time zone
and suffer from what is called 'jet lag'..

A) So people can go to bed very late one night but still


wake at the usual hour in the morning.

B) Is it possible to reset one\'s body clock?

C) Is that why people get so depressed in dark


weather?

D) Then why ever should anyone want to reset their


body clock?

E) Does a long walk in the sunshine really help to


overcome

64. Interviewer: Did you get the overwhelming


support of the women in your constituency?
Newly-Elected Woman Representative: ----
Interviewer: How do you account for that?
Newly-Elected Woman Representative: I suppose
the issues I talked about were not of primary
concern to them..

A) Probably. There are more women than men in the


constituency.

B) I really have no idea.

C) Yes, I suppose so. But the men voted, too.

D) Yes, and I have already thanked them.

E) No, I dont think I did.


65. Jack : In the British empire, India was the first
68. - 71.sorularda, verilen cmleye anlamca en
colony to win its independence soon after World
War II, wasnt it? yakn cmleyi bulunuz.
Robin : Yes, it was. Actually, the extraordinary
Indian nationalist Mahatma Gandhi had been at
work in India since the 1920s and had pioneered
anti-colonial ideas. 68. The stocks of bluefin tuna, the most valuable fish
Jack : ---- in the world, have plummeted to such paltry
Robin : Principally, his strategy was nonviolent levels that many scientists speculate that the
non-cooperation. So he urged the Indian people fish could be headed for extinction..
to withdraw from the imperial economy and
develop their own means of resistance such as
going on strike, refusing to pay taxes, or A) Scientists believe that the excessive demand for
boycotting British goods.. the valuable bluefin tuna fish has risen to such a
level that there is speculation about the fish
becoming extinct.
A) In the face of colonial domination, how did he set
about the anticolonial movement for the B) Stocks of the worlds most desirable bluefin tuna
independence of India? fish have reached such a low level that many
scientists are convinced that they are about to
B) Gandhis strategy for independence provided a become extinct.
model for many liberation struggles in other
colonies. C) There is some speculation among scientists around
the world as to how far the stocks of the valuable
C) Hadnt he already led a nonviolent campaign for bluefin tuna fish can be allowed to fall before they
the political rights of the Indian community in South become extinct.
Africa between 1894 and 1914?
D) Stocks of the bluefin tuna, the most expensive fish
D) By 1947 Gandhi had gained such widespread in the world, have dropped to such a low level that
support that the British found it impossible to scientists are predicting that they might become
continue in power in India. extinct.

E) Yet, in June 1947, British India was partitioned into E) The most expensive fish in the world is the bluefin
the nations of India and Pakistan, and the process tuna, but scientists fear that stocks will soon reach
of partition brought brutal religious and ethnic a paltry level and the fish will become extinct.
warfare.

69. Wouldnt it be better to let them know about the


66. Beverly :- Heres an article about a 300-metr- alterations to the plan?.
-tall wind turbine that was built in Denmark.
Cam :- Three hundred metres? Thats unusually
tall, isnt it? A) Couldnt the plan be changed and made better?
Beverly :- ----
Cam :- Unbelievable!. B) Why havent they been informed about the new
developments?

A) Indeed, it is, and it will be able to provide enough C) Shouldnt they have been cosseted before the
electricity for 20,000 homes. scheme was changed?

B) More of these huge turbines are going to be built in D) Wed better ask them to change the plan, hadnt
Germany. we?

C) Yes, it is. I wonder why the US isnt doing more to E) Dont you think they should be informed about the
produce renewable energy like this. changes in the plan?

D) I dont know. Was it?


70. Because doctors are unlikely to know the amount
E) I agree. What about wind turbines in other of radiation a person has received, they usually
countries? predict outcome based on the persons
symptoms..

67. Patient: - The pain in my stomach still hasn't


gone away. A) Doctors often base their predictions on a persons
Doctor: - ---- symptoms rather than on the amount of radiation
Patient: - Well, I did for a day or two. But it the person has received.
didn't seem to help..
B) It is unlikely that doctors will predict the amount of
radiation a person has received based on the
A) It will, in time. Don\'t worry. persons symptoms.

B) Have you been taking the medicine as I prescribed? C) Doctors usually predict outcome based on a
persons symptoms, but they may also know the
C) In that case I\'ll give you some more medicine. amount of radiation the person has received.

D) How long has this been going on? D) The amount of radiation a person has received is
highly unlikely to lead to a prediction of the
E) I should have come to see you earlier. outcome based on the persons symptoms.

E) Looking at a persons symptoms, doctors often


predict outcome, as they probably will not know the
amount of radiation to which the person has been
exposed.
71. Birds, despite sharing common reptilian 73. During World War II, the Allies had started to
ancestors with pterosaurs, evolved quite think of ways in which a new world order could
separately and have been much more successful replace the failed League of Nations. Even before
in their dominance of the air.. it joined the war against Germany and its allies,
the US had agreed on an Atlantic Charter with
Britain. The basis of this new charter was US
A) In spite of having the same genetic codes, birds\' President Roosevelts Four Freedoms: freedom
evolution was totally distinct from that of of speech, freedom of worship, freedom from
pterosaurs, and birds dominate the air. fear, and freedom from want. At the end of the
war, the victorious Allies created the United
B) Birds have succeeded in dominating the air since Nations, which expressed the ideas of the
they evolved differently even if they share common Atlantic Charter. ----..
ancestors with pterosaurs.

C) Birds and pterosaurs are both descended from the A) In the early 1960s, Britain was increasingly
same reptilian ancestors, but birds had a rather interested in joining the newly-formed European
different change of direction in the evolution and Community
have become far better at flying.
B) In 1948, the Soviet Union tried to capture West
D) Pterosaurs couldn\'t catch up with the evolution of Berlin by stopping all road and rail traffic to it
birds although they come from the same reptilian
family, and thus air is under birds\' domination C) Thus, they hoped that the success of the wartime
today. alliance could be carried into peacetime

E) Birds are much more successful predators when D) All these military and scientific developments drew
compared to their reptilian ancestors and have Britain more closely to the US
evolved to dominate the air.
E) During the post-war period, Britain was seriously
concerned about the danger from the Soviet Union
72. - 75.sorularda, bo braklan yere, parada
anlam btnln salamak iin 74. Piri Reis was a Turkish mariner who lived in the
getirilebilecek cmleyi bulunuz. Ottoman Empire during the 16th century.
Besides his skills as a great seaman, he was also
interested in drawing maps of the places he
visited. The details in his map of the coasts of
72. In the 1990s, most psychiatrists in the US started Africa, America, and Northern Antarctica caused
to treat children and teens with antidepressants great excitement when it was discovered in
designed for adults, since they wanted 1929.What is most striking was that Reiss map
something more for them than talk therapy. ----. showed Antarctica as it looks underneath ice-
Now, however, studies have found that some cap. ----.
antidepressants might interfere with normal
patterns of growth in childrens still developing
brains.. A) Antarctica was the last continent to be fully
discovered and mapped.

A) Although these drugs had not been tested in the B) The last time the area was ice-free, however, was
young brains of children, they hoped the benefits at least 6,000 years ago.
would outweigh the risks
C) Piri Reis did his best to let the future generations
B) Several brain areas are commonly associated with know how the world looked in his time.
depression such as those involved in mood, sleep,
appetite, desire and memory D) The map that belonged to Piri Reis was discovered
in a museum in Istanbul.
C) Finding these effects in mice is a long way from
proving that the same thing happens in humans E) The discovery of Antarctica by Piri Reis was a very
important event.
D) Adults with depression usually recognize that they
have a problem and they want to fix it, but children
75. Hippopotamuses can be irritable and aggressive
rarely ask to see a psychiatrist
when it comes to defending their territory and
their young. ----. They have trampled or gored
E) Concerns over antidepressant use led the FDA to
people who came too near, dragged them into
issue a warning label for these medications
lakes, tipped over their boats, and bitten off
their heads..

A) Hippos are led by dominant males, which can


weigh 6,000 pounds or more

B) Agricultural irrigation systems and other


developments have depleted the hippos wetland,
river and lake habitats

C) Although hippos occasionally fight with crocodiles,


a growing number of their attacks are on humans

D) A decade ago there were about 160,000 hippos in


Africa, but the population has dwindled to between
125,000 and 148,000 today

E) In countries beset by civil unrest, where people are


hungry and desperate, hippos are hunted for their
meat
78. (I) In his article The Future Doesnt Need Us,
76. - 80.sorularda, cmleler srasyla
the scientist Bill Joy describes advances in three
okunduunda parann anlam btnln fields: genetic engineering, nanotechnology and
bozan cmleyi bulunuz. robotics. (II) The first has created the possibility
of gene therapy that could bring diseases like
cancer under control. (III) These technological
advances carry a strong potential for improving
76. (I) Female killer whales live long after their our quality of life in the not-too-distant future.
reproductive years are over. (II) A recent study (IV) The second refers to technologies that
showed that adult sons of mother whales live manipulate matter on the extremely small scale
longer and produce more offspring when the of nanometres, allowing the creation of novel
mother has a long post-menopausal phase. (III) plant species or new viruses. (V) Finally, robotics
The same trend, however, was not observed for will eventually raise the possibility of intelligent
daughters of the same whales. (IV) The and self-replicating machines that are barely
mechanism by which mothers increase their distinguishable from humans..
adult sons survival are unclear, but
hypothesized reasons include assistance to
search for food and support during dangerous A) I
situations. (V) Whale research has become an
increasingly popular field over the past ten B) II
years, which was actually unexpected.
. C) III

D) IV
A) I
E) V
B) II

C) III

D) IV

E) V

77. (I) One of the oldest methods of preserving food


is drying. (II) It slows down the proliferation and
activity of the bacteria that cause spoilage and
decay, but it considerably alters the appearance
of food due to the loss of water. (III) With the
success of freezing and its characteristics for
retaining the food value of ingredients, drying is
no longer an essential means of preserving food
for times when it may be out of season or
expensive. (IV) Since prehistoric times, cereals
and fruits have been dried in the sun before
being stored. (V) The drying of fruits and
vegetables has been widely practiced for so long;
in Greece for grapes, in Turkey for apricots, and
in Iran and Spain for tomatoes..

A) I

B) II

C) III

D) IV

E) V
79. (I) Noam Chomskys ethical analyses are based
on what he calls the 'principle of universality'.
(II) Good politicians must analyze their political
actions rigorously, instead of blinding the public
with rhetoric. (III) At root, this principle is
relatively simple. (IV) It says that at the very
least we should apply to ourselves the same
standards that we apply to others. (V) This is a
principle that Chomsky claims has always been
central to any responsible system of ethics..

A) I

B) II

C) III

D) IV

E) V

80. (I) Why do people pursue self-esteem? (II) Most


people would answer that having self-esteem
makes you feel good. (III) They found that
participants who had their self-esteem raised
had lower psychological arousal and reported
less anxiety. (IV) There is probably a grain of
truth here, but on the other hand, there are
causality issues to be addressed. (V) So actually,
rather than self-esteem producing happiness,
feeling happy may inflate self-esteem..

A) I

B) II

C) III

D) IV

E) V
SORU CEVAP SORU CEVAP

1 B 41 D

2 A 42 A

3 B 43 C

4 E 44 B

5 E 45 E

6 D 46 C

7 D 47 B

8 A 48 D

9 A 49 E

10 B 50 A

11 A 51 D

12 E 52 C

13 C 53 B

14 C 54 A

15 E 55 A

16 E 56 B

17 D 57 D

18 A 58 C

19 C 59 C

20 E 60 B

21 B 61 A

22 C 62 C

23 B 63 D

24 E 64 E

25 D 65 A

26 D 66 A

27 D 67 B

28 E 68 D

29 E 69 E

30 A 70 E

31 E 71 C

32 C 72 A

33 C 73 C

34 E 74 B

35 E 75 C

36 D 76 E

37 A 77 C

38 E 78 C

39 E 79 B

40 E 80 C

Das könnte Ihnen auch gefallen